PAeasy Genitourinary

Lakukan tugas rumah & ujian kamu dengan baik sekarang menggunakan Quizwiz!

A 12-year-old boy presents with a 3-hour history of extreme, severe pain in the right testis. It started suddenly, is 8/10 in intensity, and does not radiate. It is associated with nausea and scrotal swelling. He never had such pain in his lifetime, and he denies any problem in urination. He has never been operated on, and he denies any history of trauma. He is allergic to penicillin. On physical exam, the child is in visible distress. Temperature is 37.0°C, heart rate is 95, blood pressure is 120/70 mm Hg, and respiratory rate is 20 per minute. Genital examination reveals enlargement and edema of the entire scrotum. The right testicle is erythematous and tender to palpation; it appears to sit higher and lies horizontally in the scrotal sac relative to the left side. The cremasteric reflex is absent ipsilaterally, and there is no relief of pain upon elevation of the scrotum (Prehn's sign). Abdomen is non-tender and tympanic to percussion in all 4 quadrants. Bowel sounds are audible. Chest auscultation shows normal vesicular breathing with mild crepitations over the lower lung fields. Cardiac exam reveals normal S1 and S2, without rubs, murmurs, or gallop. His initial labs show a hemoglobin of 14.5 g/dL, WBC of 13,000/mm³, platelets of 210,000/mm3, sodium of 140 mmol/dL, potassium of 3.8 mmol/dL, chloride of 95 mmol/dL, urea of 25 mg/dL, and creatinine of 0.9 mg/dL. Question What sign or symptom is the most sensitive for the diagnosis of this condition? Answer Choices 1 Tenderness 2 Edema 3 Horizontal lie 4 Prehn sign 5 Loss of cremasteric reflex

Explanation Testicular torsion is a true urologic emergency and needs to be differentiated from other causes of testicular pain (e.g., trauma, epididymitis/orchitis, incarcerated hernia, varicocele, idiopathic scrotal edema, and torsion of the appendix testis). The finding of an ipsilateral absent cremasteric reflex is the most accurate and sensitive sign of testicular torsion. This reflex is elicited by stroking or pinching the medial thigh, causing contraction of the cremaster muscle which elevates the testis. The cremasteric reflex is considered positive if the testicle moves at least 0.5 cm.

3 types of Dialysis

Dialysis started CKD stage 5 or symptoms of Uremia: Three options for dialysis- Hemodialysis: • MC type; Usually done in center, 3x week 4hrs at a time Peritoneal Dialysis: • MC complication is infection/peritonitis Renal transplantation: • Most satisfactory treatment for ESRD • Living related donor is best match

A 39-year-old male was out on a ranch with his friends for the weekend. He indulged in horseback riding daily, stretched out over several hours in the afternoon. On his return home, he experienced high fever with chills and malaise, myalgias, dysuria, perineal pain, and cloudy urine. Examination in the ER revealed a temperature of 101.5°F, pulse 110/min, BP 120/80 mmHg, and respiratory rate of 16/min. There was no pallor, jaundice, or lymphadenopathy. Lungs were clear and no murmurs appreciated. Abdominal exam showed no tenderness, masses, ascites, or hepatosplenomegaly. Bowel sounds were active, and rectal exam showed exquisite tenderness. Significant labs included WBC 13,400/uL and urinalysis with 15 WBC and 4 RBC. Question What would be a provisional diagnosis with blood and urine cultures pending? Answer Choices 1 Acute pyelonephritis 2 Acute urethritis 3 Rectal abscess 4 Anal fissure 5 Acute prostatitis

Explanation Acute prostatitis is defined as an inflammation of the prostate gland that develops suddenly and is common in men, likely due to reflux of infected urine into intraprostatic ducts. This can happen after instrumentation, catheterization, or trauma, like horseback riding, biking, etc., and worsened by dehydration, as in this patient. The National Institutes of Health classification of inflammatory conditions of the prostate is as follows: I Acute prostatitis II Chronic bacterial prostatitis III A Chronic prostatitis/pelvic pain syndrome, inflammatory III B Chronic prostatitis/pelvic pain syndrome, noninflammatory IV Asymptomatic inflammatory prostatitis Gram negative organisms are the main culprit, including E.coli, proteus, klebsiella, enterobacter, and pseudomonas. Symptoms of dysuria, fever, perineal pain, and tender prostate are typical. Treatment is with trimethoprim-sulfamethoxazole or quinolones for 4 weeks. In sicker patients, hospitalization may be needed, in which case IV antibiotics with aminoglycoside and ampicillin should be given until the patient is afebrile for 24-48 hours, then oral antibiotics continued for total of 4-6 weeks to avoid complications such as abscess formation or chronic prostatitis. Acute pyelonephritis presents with fever, flank pain, tender renal angle, and normal rectal exam. Treatment includes oral fluoroquinolone or trimethoprim-sulfamethoxazole for mild to moderate disease and IV ceftriaxone or a fluoroquinolone for hospitalized patients, to be substituted with oral antibiotics after improvement in symptoms. Total duration of antibiotics should be 10-14 days. Acute urethritis is associated with dysuria and urethral discharge with pruritus at urethral meatus. Fever, chills, frequency, urgency, and hematuria are uncommon. It may be gonococcal, which is the most common cause of urethritis in men or nongonococcal urethritis (NGU). Although most cases of NGU are due to chlamydia trachomatis, other etiologies include T. vaginalis, Mycoplasma genitalium, and Ureaplasma urealyticum. Gram stain and culture or the urethral discharge should be done. Treatment is with ceftriaxone 125mg IM, cefixime 400mg PO, ciprofloxacin 500mg PO, or ofloxacin 400mg PO, all in a single dose in gonococcal urethritis and azithromycin 1gm PO or doxycycline 100mg BID for 7 days or ofloxacin 400mg PO BID for 7 days for NGU. Rectal abscess is a distant possibility in this patient. It presents with constant pain in the rectal area and perhaps fever and malaise but no dysuria or cloudy urine. Rectal exam will be tender and reveal a fluctuant mass. UA, however, will not be abnormal. Treatment is with incision, drainage, and perhaps antibiotics for anaerobic coverage. Anal fissure presents with excruciating pain with the passage of bowel movements and is associated with constipation. The passage of stool may be accompanied by bright rectal bleeding usually limited to a small amount on the toilet paper but sometimes more profuse bleeding. Treatment aims at relaxing the sphincter, keeping bowel movements soft and smooth, and pain control.

The most common cause of tender testicular swelling is: Answer Choices 1 Tuberculosis. 2 Hematocele. 3 Neoplasm. 4 Orchitis. 5 Hydrocele.

Explanation Acute orchitis is the most common cause of tender testicular swelling. It may result from mumps and other infectious diseases, and may be confused with spermatic cord torsion. Tuberculosis, hematoceles, neoplasms, hydroceles, chyloceles, and gummas all may produce nontender testicular swellings.

A 59-year-old patient with end-stage renal disease presents via ambulance to the emergency department. Initially complaining of severe weakness, he quickly developed respiratory failure, for which he was intubated shortly after arrival. Stat chemistry labs reveal the following: Na 131 mEq/L; K 6.1 mEq/L; Cl 98 mEq/L; CO3- 19 mEq/L; BUN 42 mEq/L; Cr 6.6 mEq/L. Question Which of the following electrocardiographic (ECG) findings is most likely to be present? Answer Choices 1 Flat P waves 2 Peaked T waves 3 Shortened QT interval 4 Shortened PR interval 5 ST segment elevation

Explanation The patient is presenting with hyperkalemia. This is relatively common in patients with end-stage renal disease, especially among those who are noncompliant with medications and/or dialysis therapy. Signs and symptoms of hyperkalemia include weakness, which may escalate to muscle paralysis and respiratory failure. A number of characteristic electrocardiographic (ECG) findings are present in the setting of hyperkalemia. Tall, so-called "peaked" T waves are a classic early finding. As potassium levels rise, there is flattening of P waves, prolongation of the PR interval, and widening of the QRS complex. Untreated, a sinus wave pattern may develop followed by life-threatening dysrrhythmias and/or cardiac arrest.

A 38-year-old male with a history of frequent urinary tract infections (UTIs) and kidney stones is following up with the urology physician assistant. His urine today continues to reveal persistent hematuria, which he had the last few times he saw his primary care physician. His vitals are 120/80, P = 72, T = 96.7˚F. He denies any discomfort at this time. GFR is normal. You review a urogram, which reveals a paint-brush appearance of the papillae. The kidneys are normal in size. What is the most likely diagnosis? A chronic renal insufficiency B renal cell carcinoma C end stage renal disease D medullary sponge disease E polycystic kidney disease

Medullary sponge disease The correct answer is (D). The patient's history of hematuria, recurrent UTIs, and kidney stones are a common presentation of this inherited disease with medullary cysts. The findings on the urogram of a paint-brush appearance of the papillae are pathognomonic of the disease. Choices (A) and (C) are unlikely due to the normal GFR. The patient's history is not suggestive of choice (B), renal cell carcinoma. A solitary solid renal mass would suggest renal cell carcinoma.

A 6-year-old boy presents with his mother, who is concerned because he wets his bed 2 or 3 times a week at night. He has no urinary symptoms during the daytime. He is the second child of 3. He is in the 1st grade and is struggling with his performance. He has had no medical problems; there is no history of developmental delay, and he was the product of a normal uncomplicated pregnancy and delivery. Since the birth of the 3rd child, his behavior has been poor. The vital signs are normal, and examination of other body systems is unremarkable. Question What investigation is most important in this child's workup? Answer Choices 1 Complete cystometric evaluation 2 Intravenous pyelography (IVP) 3 Renal ultrasound 4 Urine culture 5 Urinalysis

Urinalysis Enuresis means any involuntary loss of urine. If it is used to denote incontinence during sleep, it should always be qualified with the adjective "nocturnal." Nocturnal enuresis is defined as involuntary urination in sleep without urological or neurological causes after the age of 5 years, at which time bladder control would normally be expected. The classification of enuresis is based on whether the child has ever achieved bladder control. Primary enuresis refers to a child who has never been dry; whereas, secondary enuresis means the child has been dry for a period but becomes enuretic later. Another useful classification is based on a period when the child does not have bladder control: • Nocturnal enuresis: Enuresis at night only. • Diurnal enuresis: Enuresis during the day only. • Nocturnal and diurnal enuresis: Enuresis during both day and night. The pathophysiology of enuresis is not completely clear; however, immaturity on the part of the autonomic nervous system that controls the bladder is present in the vast majority of cases. Only 20% of children with enuresis have a psychodevelopmental disorder (lower intelligence quotient or behavioral disorder). Secondary enuresis is often associated with a stressful environmental event. Urinalysis is the only mandatory investigation for nocturnal enuresis. Treatment: the therapeutic approach is still based on empirical data. Therapy is aimed at alleviating the symptoms of nocturnal enuresis rather than at curing the condition. A behavior-modification program is the treatment of choice, including buzzer/bell and pad, positive reinforcement, charting progress to increase confidence and self-esteem, urinating before bedtime, avoiding liquids after the evening meal, and avoiding psychological trauma through blame or belittling the child. Complete cystometric evaluation is incorrect. Cystometric analysis is used to evaluate the bladder's capacity to contract and expel urine. Intravenous pyelography (IVP) is incorrect. IVP refers to a series of X-rays taken of the kidneys, their collecting or drainage system (the ureters), and the bladder. It is done to locate a suspected obstruction to the flow of urine through the collecting system. Renal ultrasound is incorrect. If enuresis also occurs in the daytime or if urinary flow is small or interrupted, a renal ultrasound and a careful neurologic examination are indicated. Urine culture is incorrect. Urine culture may be appropriate, but is not most important. Urine culture is useful mainly when history, physical examination, or both suggests infection.

A 7-year-old male is suspected to have a Wilms' tumor (nephroblastoma). What is the most common symptom at presentation? A Abdominal mass B Hypertension C Hematuria D Coagulopathy E Fever

abdominal mass The diagnosis of a Wilms' tumor is most commonly made after the discovery of an asymptomatic mass, by a family member or a physician, during a routine physical examination. The most common sign is an abdominal mass (A). Other symptoms at presentation include abdominal pain and distention, anorexia, nausea and vomiting, fever (E) and hematuria. Hypertension (B) is seen in 25-60% of cases and is caused by elevated renin levels. As many as 30% of patients demonstrate hematuria (C) and coagulopathy (D) can occur in up to 10%.

A 32-year-old woman presents with a solitary thyroid nodule. A biopsy confirms that she has thyroid cancer. She subsequently has a thyroidectomy. However, during the procedure, the surgeon accidentally removes most of her parathyroid glands as well. As a consequence, she develops hypoparathyroidism. Her laboratory results are in the chart. What symptom would be consistent with her history and findings? TEST RESULTS REFERENCE RANGE RBC 4.8 x 106/µl 4.5 - 5.7 x 106/µl (male) 3.9 - 5.0 x 106/µl (female) Hematocrit 42 40 - 50 (male) 36 - 44 (female) Hemoglobin 15 gm/dL 13.8 - 17.2 gm/dL (male) 12.1 - 15.1 gm/dL (female) Platelets 252,000/mm3 130,000 - 400,000/mm3 Potassium 4.1 mEq/L 3.5 - 5.0 mEq/L Sodium 144 mEq/L 135 - 145 mEq/L Calcium 7.8 mg/dL 8.5 - 10.5 mg/dL Question What is a sign of her secondary condition? Answer Choices 1 Chvostek's sign 2 Kernig's sign 3 Babinski's sign 4 Kussmaul's sign 5 Quincke's sign

chvostek's sign This patient has hypocalcemia secondary to hypoparathyroidism. Paresthesias can be seen with hypocalcemia due to the increase in neuromuscular irritability. Common sites for the paresthesias are around the mouth and on the fingertips. Tetany can also be seen. Chvostek's sign can be seen with tetany. When tapping on the facial nerve produces contraction on that side of the face, it is called Chvostek's sign. Kernig's sign is a sign of meningeal irritation. The patient lies on his back and his leg is raised and knee bent at a 90-degree angle. When the examiner straightens the knee, if there is pain or resistance to further extension, Kernig's sign is said to be positive. Babinski's sign is an abnormal response to stimulation on the sole of the foot; it results in dorsiflexion of the big toe and fanning of the other toes. Babinski's sign is seen with pyramidal disease. Kussmaul's sign is an increase in venous pressure during inspiration; it can be seen with cardiac tamponade. Quincke's sign (Quincke's pulse) is an alteration seen in nails with each heartbeat. Quincke's sign can be seen with aortic insufficiency.

Upon inspection of an 18-year-old man's scrotum, you note that the left side is underdeveloped, and a testis is not palpable. There is no scrotal tenderness, swelling, or nodularity. What is the patient's condition most likely due to? Answer Choices 1 Cryptorchidism 2 Acute epididmitis 3 Hydrocele 4 Testicular cancer 5 Strangulated inguinal hernia

cryptorchidism Explanation An underdeveloped scrotum, either unilateral or bilateral, is likely due to undescended testicles, which is also known cryptorchidism. Acute epididymitis and strangulated inguinal hernia are usually tender to the touch and cause swelling. Hydroceles are commonly accompanied by scrotal swelling. If painless nodules are palpated in the testes, testicular cancer should be ruled out, especially in men between the ages of 15 to 35.

A 33-year-old African American woman develops a loss of appetite and has a few episodes of vomiting. She reports constipation and lethargy. She thinks that her urine output has increased. Her history is significant for the presence of sarcoidosis. On physical examination, some muscle weakness is present. Laboratory tests are ordered and are pending. An EKG is done and shows a shortened QT interval. Question Based on the EKG changes, what electrolyte abnormality would be expected? Answer Choices 1 Hypercalcemia 2 Hyperkalemia 3 Hypocalcemia 4 Hypokalemia 5 Hyponatremia

hypercalcemia Hypercalcemia refers to an elevated calcium level. A shortened QT interval is seen with hypercalcemia. The time from the beginning of the QRS complex to the ending of the T wave is the QT interval. Sarcoidosis can be associated with hypercalcemia. Symptoms of hypercalcemia include anorexia, vomiting, constipation, and polyuria. Muscle weakness can be seen. None of the other electrolyte abnormalities listed are associated with sarcoidosis. Hyperkalemia refers to an elevated potassium level. Hyperkalemia can result in peaked T waves on EKG. The T wave is a reflection of ventricular repolarization. There can be widening of the QRS complex as well. The QRS complex is seen when the ventricle depolarizes. Hypocalcemia refers to a depressed calcium level. There would be a prolonged QT interval with hypocalcemia. The time from the beginning of the QRS complex to the ending of the T wave is the QT interval. Hypokalemia refers to a depressed potassium level. Hypokalemia can result in a depressed ST segment and the presence of U waves. The time from the end of the QRS complex to the beginning of the T wave is the ST segment. Hyponatremia refers to a depressed sodium level. Hyponatremia is not associated with EKG changes.

A 22-year-old man presents with fever and a transient maculopapular rash of 1 week duration. His serum creatinine and blood urea nitrogen (BUN) are elevated. The urinalysis is significant for hematuria, pyuria, white blood cell casts, and eosinophiluria. What is the most likely diagnosis? Answer Choices 1 Acute tubular necrosis 2 Diabetic nephropathy 3 Hypertensive nephrosclerosis 4 Interstitial nephritis 5 Lupus nephritis

interstitial nephritis The clinical picture of fever, recent etiological exposure (e.g., drugs, infection), maculopapular rash with serum and urinalysis findings of elevated creatine, elevated BUN, hematuria, pyuria, white blood cells casts and eosinopiluria is suggestive of interstitial nephritis. Interstitial nephritis typically occurs following medication administration, but can also occur in response to viruses or bacterial infections. It is an allergic reaction of the kidney that results in fevers, rash, arthralgias, hematuria, and eosinophilia. The eosinophils in the urine are pathognomonic for this disease. In acute tubular necrosis, the BUN and creatinine are elevated but the urinalysis may show a brown color. On microscopic examination, muddy brown casts (pigmented granular casts), epithelial cell casts, and renal tubular cells would be seen, which is not the case in this patient. There is no history of diabetes or hypertension in this patient; therefore, diabetic nephropathy and hypertensive nephrosclerosis are ruled out. Lupus nephritis is a complication of systemic lupus erythematosus. It is an autoimmune inflammatory disorder that affects many organs. 85% of patients are women. On urinalysis, hematuria and proteinuria are common findings.

Normal Urine Output

0.5- 1 mL/kg/hr

Glomerular damage inflicted in a patient with Goodpasture's disease is best described by which of the following? Answer Choices 1 Auto-antibodies are produced against glomerular basement membrane 2 Antibodies are produced against planted antigen 3 Auto-antibodies against Heymann antigen 4 Deposition of circulating immune complex in the glomeruli 5 C3Ne deposition in the glomeruli

1 Auto-antibodies are produced against glomerular basement membrane In Goodpasture's disease, autoantibodies (IgG) are produced against basement membrane antigen (type IV collagen). Simultaneous pulmonary hemorrhage and glomerulonephritis due to autoantibody deposition in pulmonary and glomerular basement membrane is known as Goodpasture's syndrome.

Normal GFR

120

BP goal in CKD patients

140/90 • Use a ACEI/ARB if HTN in- 1. DM pts 2. Non-DM with protein/cr ratio of 0.2 mg/mg or 200 mg/g AceI and ARBs decrease proteinuria (watch for hyperkalemia though!)

CKD stage 4/Severe Kidney disease GFR

15-29

CKD stage 3/ Moderate kidney disease GFR

30-59

Oliguria Urine Output

50-400 mL/day

CKD stage 2 GFR

60-89

CKD stage 5/kidney failure GFR

<15

Anuria Urine Output

<50 mL/day

Non Oliguria Urine Output

>400 ml/day

Absolute indications for dialysis

Absolute indications for dialysis!!!! Uremic pericarditis Hyperkalemia, severe Diuretic unresponsive fluid overload (pulmonary edema) Intractable metabolic acidosis

A 6-year-old boy presents to the office with a fever, malaise, and dark urine. His mother states he missed school earlier this month with a sore throat. On examination blood pressure is 120/88 mm Hg, pulse is 82/min, temperature 100.6°F, and respirations 16/minute. On physical assessment, the patient appears ill with only mild costovertebral angle tenderness noted. Urinalysis reveals the following: Urinalysis Result Specific gravity 1.00 pH 5.2 protein + 1 blood + 2 glucose negative ketones negative bilirubin negative urobilinogen negative nitrates negative leukocyte esterase + 1 Microscopic examination reveals: RBCs, renal tubular epithelial cells, RBC casts, and granular casts. Question What test will determine the most likely etiology? Answer Choices 1 Urine for culture and sensitivity 2 Urine cytology 3 Erythrocyte sedimentation rate 4 Anti DNAase B serology 5 Urine protein electrophoresis

Anti DNAase B serology Explanation The correct answer is anti-DNAase B serology to identify post-streptococcal glomerulonephritis. Group A β-hemolytic streptococci pharyngitis may result in the delayed complication of post-streptococcal glomerulonephritis 10-14 days after the infection. Patient presentations may range from subclinical symptoms to acute nephritic syndrome as streptococci may produce streptolysin, DNAase, and hyaluronidase that lead to tissue destruction and disseminate infection. Serology testing to identify antibodies to these exoenzymes can aid in the diagnosis by demonstrating indirect evidence of infection. Confirmation may require serial antibody draws that reveal a rise in titer levels above baseline. Urine culture and sensitivity would be appropriate if the clinical picture only entailed fever and flank tenderness in the presence of pyuria and hematuria suggesting a urinary tract infection. That diagnosis does not explain the presence of proteinuria with renal tubular epithelial cells and casts. These indicate intrinsic kidney damage, which is not seen in urinary tract infections. Further serology testing is indicated in the post pharyngitis period. Urine cytology is ordered in the presence of gross or microscopic hematuria, which is often painless, to identify malignant cells in the urinary tract. This patient does not fit the epidemiologic profile or clinical presentation of malignancy to warrant cytology testing. Erythrocyte sedimentation rate can aid in detecting an inflammatory response, such as nephritis, but it lacks specificity to identify the infectious process. Urine protein electrophoresis is indicated to identify abnormal levels of free monoclonal light chains (Bence Jones protein) from immunoglobins in cases of myeloma.

Your patient is a 55-year-old man presenting with a strong, sudden need to urinate. He feels his bladder spasms and sometimes has involuntary loss of urine. The problem started several months ago and seems to be worsening. He is very upset because it sometimes happens during his business meetings. His urinalyses were always normal. He takes no medications. Careful examination shows that he has urge incontinence. Bladder training and behavioral techniques were unsuccessful, and you decided to introduce medication. Before suggesting a medication for this condition, you will tell him that this drug may have some side effects: dry mouth, difficulty in urination, constipation, blurred vision, tachycardia, drowsiness, and dizziness. Question This may happen because you will be prescribing what type of drug? Answer Choices 1 Cholinergic 2 Anticholinergic 3 Epinephrine 4 Antibiotic 5 Botulinum toxin

Anticholinergic Urge incontinence is defined as involuntary loss of urine occurring for no apparent reason together with a feeling of urinary urgency (a sudden need or urge to urinate) that represents a hygienic or social problem to the individual. The most common cause of urge incontinence is involuntary and inappropriate detrusor muscle contractions. The drug you will suggest is anticholinergic (like Oxybutynin). It will relieve urinary and bladder difficulties, including frequent urination and urge incontinence by decreasing muscle spasms of the bladder, increasing the capacity of the bladder, and delaying the initial urge to void. It is a competitive antagonist of M1, M2, and M3 muscarinic acetylcholine receptors and in higher doses can act as spasmolytic on bladder smooth muscle. Anticholinergic side effects are dry mouth, difficulty in urination, constipation, blurred vision, tachycardia, drowsiness, and dizziness. Cholinergic drug will cause slowing of the heartbeat and increases in normal secretions. For this reason, patients who already have a problem with incontinence should not be advised to use these drugs. Epinephrine is not indicated in this patient. Epinephrine stimulates the ends of the sympathetic or inhibitory nerves of the bladder, with the effect of relaxation of the bladder muscles and the increase in tone and rate of contraction of the ureter. The secretion of urine is increased synchronously with the rise in arterial pressure. It will also cause overacting heart, palpitation, and vomiting. There is no need for antibiotics in a patient with normal urine analysis for a problem that lasts several months. Botulinum toxin is given as intradetrusor injection in patients who have failed pharmacological therapy. It has been shown to decrease episodes of urinary leakage by preventing the release of acetyl choline from presynaptic membrane. It is also indicated for urinary incontinence in patients with neurologic conditions (e.g., spinal cord injury, multiple sclerosis). It sometimes can cause urinary retention given as intradetrusor injections and occasionally headache, light-headedness, fever, abdominal pain, and diarrhea (not necessarily a direct result of Botox).

A 78-year-old Caucasian woman is admitted with a provisional diagnosis of diverticulitis, with complaints of acute abdominal pain and lack of eating or drinking well for several days. Her past medical history is significant for diabetes mellitus, diagnosed 12 years ago, and hypertension (though she was hypotensive at admission). Both conditions were reported to have been under good control. Her medications include regular and long-acting insulin and hydrochlorothiazide/lisinopril 25/20 mg QD. Her son had recently been giving her 800mg ibuprofen BID-TID for her abdominal pain during the last week. She has no known allergies. While in the hospital, the patient's laboratory results are followed daily. 2 days after the CT with contrast, it is noted that her serum creatinine has risen to a level of 3.5 mg/dL. Records from 1 month ago at her family physician showed her labs to include a hemoglobin A1C of 6.8%, creatinine of 1.8 mg/dL, and her blood pressure was 107/68 mm Hg. Question What measure would have been the best prevention for this patient's sudden decline in renal function? Answer Choices 1 Begin volume expansion prior to the contrast study 2 Change her insulin to metformin prior to the contrast study 3 Initiate a loop diuretic prior to the contrast study 4 Start IV mannitol immediately after the contrast study and continue for 72 hours 5 Use high-osmolar iodinated radiocontrast material during the contrast study

Begin volume expansion prior to the contrast study Explanation Mild volume expansion prior to the CT with IV contrast is recommended to prevent contrast nephropathy in higher-risk patients.1 Hydration should begin an hour or more before the procedure and continue several hours after the CT. There is evidence showing better results with NaHCO3 over regular saline (NaCl).2 This patient had several indicators of volume depletion - she was taking a diuretic and had decreased oral intake, along with potentially some fluid/blood loss with the diverticulitis, and her BP was hypotensive. A change to metformin is not recommended for this patient. Metformin, while not nephrotoxic directly, can increase the likelihood of lactic acidosis.3 She likely has some pre-existing renal disease (diabetic nephropathy) and metformin is relatively contraindicated in this patient. Some experts recommend cessation of metformin when patients undergo contrast studies in order to decrease likelihood of contrast nephropathy. Loop diuretics are also not recommended. These drugs can increase risk of contrast nephropathy and should be discontinued, preferably a few days prior to any contrast studies.4 Though mannitol has a role in treatment and prevention of oliguria, it is an osmotic diuretic and can increase risk of renal damage after exposure to contrast.3,5 Traditional high-osmolar iodinated contrast material carries a higher risk of contrast nephropathy than the newer low osmolar contrast materials. Risk is also dose-dependent, so a single study with a shorter exposure to a lower dose has less chance of inducing contrast nephropathy compared to repeat/multiple studies, with longer administration of high-dose.2,3,4

Complications of Dialysis

Complications in hemodialysis: ◼ Hypotension most common acute complication Estimation of "dry weight" Effects of dialysate composition Effects of medications Usually feel dizzy Manage by re-calculating dry weight or hold off on BP medication and keep well hydrated ◼ Cramps: muscle cramps in 20% Improved by reducing ultrafiltration rate ◼ Arrhythmias and angina SVT, PVC's Effects of K removal ◼ Hypoxia: supplemental oxygen should be given if angina occurs ◼ Hemorrhage: Heparin is used to prevent clotting in extracorporeal circuit. If too much heparin is given can cause hemorrhage Light pressure over hemorrhage and give Protamine to reverse heparin overload ◼ Small amounts of blood remain in artificial kidney and tubing after each treatment

Metformin Cr Cl level

CrCl must be < 1.3

Creatinine

Creatinine is a by product of msl breakdown and it is a good indicator of kidney function • If kidney function is declining Cr becomes elevated because the kidney is not filtering creatinine

A 55-year-old man with a history of chronic renal failure, 6 months status post renal transplant, presents with chest pain, productive cough, and low-grade fever. He reports generalized malaise as well. Current medications include only those related to the transplant. He has no known allergies. Examination reveals a temperature of 102°F, unremarkable HEENT (head, ears, eyes, nose, throat), and few crackles anteriorly in the upper right lung field. Chest X-ray reveals a solitary nodule in the right upper lobe. The most likely etiology for his symptoms is A Streptococcus pneumoniae B Pneumocystis jiroveci C cryptococcosis D Candida E influenza A

Cryptococcosis Cryptococcal species are opportunistic organisms responsible for infections in immune-compromised hosts. With the rise of HIV infections in the past few decades in the United States, cryptococcosis is becoming increasingly prevalent. It is also a common infection in those who have undergone solid organ transplantation. The two most common areas for infection are the lungs and the central nervous system. Pulmonary involvement includes fever, productive cough, chest discomfort, and weight loss. Pleural effusions, lymphadenopathy, and solitary or multiple nodules can all be seen on chest x-ray. Central nervous system manifestations include meningitis and meningoencephalitis. Diagnosis is confirmed with India ink prep of cerebrospinal fluid showing yeast or histologic stains of tissue from the involved organs. Treatment is with oral or parenteral antifungal agents. (

Elevated Serum Phosphate

Elevated Serum Phosphate: *Phosphorus goes up and Calcium goes down • Elevated serum Ph →Increases PTH! →Decreases Serum Ca→ Decreases Vit D • The role of PTH is to increase serum calcium and stimulates osteoclasts to breakdown bone to release more calcium in the serum (decalcification), osteosclerosis, osteoporosis, growth retardation in children • Secondary hyperparathyroidism causes Renal osteodystrophy • Tx- reduce phosphate by Rx phosphate binders; Selevamir Hydrochlroide (Renagel) • If calcidiol 25 (OH) D level is <30 ng/mL add supplemental Vit D

A 17-year-old boy experienced an increasing sensation of left scrotal fullness and heaviness after a snowboarding injury 2 weeks prior to presentation. Over the past 2 days, he has experienced worsening pain without dysuria, penile discharge, back or flank pain, fever, or chills. He denies any sexual activity. On exam, heart and lungs are normal; there is no gynecomastia. Genitourinary exam reveals Tanner stage 5 and circumcision. There are no skin lesions or lymphadenopathy. Right scrotum and testicle appear normal and without any tenderness, swelling, or erythema. The epididymis and spermatic cord are easily palpable. The left testicle is also palpable and normal in size and shape without any swelling, but it is tender to palpation over the inferior pole, and does not transilluminate. Pain does not decrease with elevation of the testis. Urinalysis is unremarkable. Question Based on these findings, what is the most likely diagnosis? Answer Choices 1 Testicular torsion 2 Testicular cancer 3 Varicocele 4 Epididymitis 5 Trauma

Explanation Testicular cancer is the most common cancer of young men between 15 and 34 years of age and is 4.5 times more common among Caucasian men then African Americans. 12% will have a history of cryptorchidism. Tumor presents commonly as a circumscribed, nontender area of induration within the testes that does not transilluminate. However, sometimes pain may be associated with torsion of the tumor, infarction, or bleeding into the tumor. The most common complaint is a feeling of fullness or heaviness in the scrotum. In most cases the epididymis and spermatic cord feel normal. Advanced tumors are diffusely enlarged and can be rock hard. Ultrasound is a sensitive and specific test that can discriminate between testicular neoplasm and other nonmalignant disorders. Additional tests for tumor markers and staging are indicated. All patients undergo orchiectomy with close observation for early stage tumors and chemotherapy and radiation for others. Either way, the prognosis is generally excellent. Testicular torsion will usually present with abrupt onset of acute testicular pain, swelling, and erythema. Torsion occurs because the attachments between the epididymis and the tunica vaginalis are loose, allowing the testis to twist on its vascular pedicle compromising blood flow. This is also known as the bell-clapper deformity. The cremasteric reflex is absent and lack of pain relief with gentle elevation of the affected testis is suggestive. Treatment consists of rapid surgical detorsion and fixation of the testis to the scrotal wall (orchiopexy) before irreversible testicular necrosis occurs. Varicocele is a dilatation of the venous plexus of the spermatic cord and affects about 20% of adolescents and male young adults. A palpable mass, which feels like a 'bag of worms', can be felt above the testis. Varicoceles rarely cause pain. It is most evident in a standing position; it collapses while supine. If there is progressive discomfort or a discrepancy in testicular volume with possible effects on fertility, a urological referral for surgical repair is indicated. Epididymitis is usually associated with a urinary tract anomaly; however, it can be caused by a virus or bacteria, it is commonly caused by Chlamydia trachomatis. Acute epididymitis is characterized by severe pain, tenderness, and swelling of the testes. Urinalysis and culture will typically be abnormal. Trauma may cause an overlying hematocele, swelling of the scrotum, and poor transillumination; in general, it will cause ecchymosis. Although this patient experienced precipitating trauma, there is no evidence of swelling or ecchymosis to indicate traumatic hematocele alone. Testicular trauma is usually an event that is indelibly inscribed in the memory of the victim, and it can be recited with much precision. Traumatic hematoceles are best managed with surgical exploration, evacuation, and repair of the testis.

Proteinuria can be falsely positive in:

Fever, exercise, prolonged standing or infection

Hematuria other causes include:

Hematuria other causes include: • Stones, UTI, kidney or bladder cancer

Metabolic Alkalosis

High HCO3 and High pH

Respiratory Alkalosis

High pH and Low CO2

A 5-year-old boy presents with history of low-grade fever, headache, and intermittent colicky pain in the abdomen, which has been localized mainly around the umbilicus since yesterday. The child has vomited once. In between the episodes of pain, the child is playful. His symptoms are also accompanied by a maculopapular rash that is more confluent over the lower extremities and the buttocks. There is no itching. Some areas of the rash are turning purple and red, which is suggestive of a hemorrhagic rash. Both knees and ankles are swollen and tender, and there is edema of the hands and feet mainly in the dependent areas. Examination of the cardiovascular, respiratory system, and abdominal examination are essentially normal. Laboratory investigations show: Hb. - 10gm%, WBC. 11,000/cm Platelet count - 550,000/cmm. Serum IGA - 500 mg /dL (normal 14-159 mg/dLfor 2-5 years age group) Urine - Proteinuria++, RBCs++ Stool - RBC+ Question What is the most likely diagnosis? Answer Choices 1 Kawasaki disease 2 Systemic onset Juvenile rheumatoid arthritis 3 Systemic lupus erythematosis 4 Henoch-Scholein purpura 5 Polyarteritis nodosa

HSP Explanation The most likely diagnosis is Henoch-Scholein purpura, which is also known as anaphylactoid purpura. It is the most common cause of non-thrombocytopenic purpura in children. Boys are affected twice as frequently as girls. It is a common vasculitis of small vessels, with cutaneous and systemic manifestations. The systems primarily involved are the skin, gastrointestinal tract (GIT), and kidneys. The characteristic manifestation of the disease is the rash, which presents initially as a pink maculopapular rash, but progresses to petechiae and purpura, which typically is characterized clinically as palpable purpura. The rash may continue to appear intermittently for 3 or 4 months, or even up to 1 year. Edema and vasculitis of the GIT may lead to GI hemorrhage, manifesting with colicky pain in abdomen, vomiting, and hematemesis. There may be enlargement of mesenteric lymph nodes. Stool is positive for occult blood. Swelling of knee and ankle joints is frequently seen due to serous effusion. There may be edema of the dependent areas. Renal involvement, which is the most important cause of morbidity and mortality, manifests as hematuria, proteinuria, and hypertension. CNS and cardiac involvement may rarely occur. Laboratory findings include thrombocytosis, leukocytosis, and elevated ESR. Serum IgA levels are elevated. Urine examination shows albuminuria, hematuria, and presence of white blood cells and casts in the urine. Renal biopsy may show mesangial deposition of IgA. Diagnostic criteria of Kawasaki disease are fever of more than 5 days duration and presence of at least 4 of the following conditions: (1) Strawberry tongue (protuberance of tongue papillae) suggestive of streptococcal infection (2) Diffuse reddening of the oral and pharyngeal mucosa, dry and cracking lips. (3) Conjunctivitis without any discharge. (4) Edema/erythema of the hands and feet and later desquamation of the skin of the fingers and toes. (5) Polymorphous rash. (6) Cervical lymphadenopathy (at least one lymph node >1.5 cm). These features are not present in the above child. Systemic lupus erythematosis (SLE) is a multisystem disease involving nearly all the organs. It is an autoimmune disorder that causes inflammation of the blood vessels and connective tissue, resulting in multisystem involvement. It is seen more commonly in girls in contrast to HSP, which is more common in boys. Joints may be merely stiff or there may be active inflammation. Cutaneous manifestations include malar, or butterfly, rash involving the cheeks and nasal bridge. Rash may be photosensitive and may involve all sun exposed areas. This rash is quite different from the rash of Henoch-Schonlein purpura. Hepatosplenomegaly and lymphadenopathy are often present. Cardiac involvement may include pericarditis, valvular thickening, myocarditis, conduction abnormalities, and congestive cardiac failure. Pulmonary involvement includes pulmonary hemorrhage and fibrosis. This is in contrast to the index case. Renal involvement may manifest as hypertension, edema, electrolyte abnormalities, nephrosis, or acute renal failure. Systemic onset juvenile rheumatoid arthritis (JRA) may be characterized by spiking fevers, arthritis, hepatosplenomegaly, lymphadenopathy, and serositis leading to pericardial effusion. Fever is accompanied by a faint transient, evanescent salmon-colored macular rash more commonly over the trunk and proximal limbs. It is non-pruritic and may last for a few hours. Heat, even that of a warm bath, may cause reappearance of the rash. Lab investigation includes raised ESR, leukocytosis, thrombocytosis, and C-reactive proteins (CRP) and anemia of chronic disease. JRA is the most common chronic rheumatologic disease in children, with a minimum duration of 6 weeks. The new nomenclature juvenile idiopathic arthritis (JIA) is being increasingly used to better define various subgroups. Clinical manifestations of Polyarteritis nodosa (PAN) is a necrotizing vasculitis involving small and medium sized arteries. Boys and girls are equally affected. It is believed to be a post-infective autoimmune response in susceptible individuals commonly occurring after upper respiratory infection by group A streptococcal infection, chronic hepatitis B infection, infectious mononucleosis, and tuberculosis. Common features include fever, weight loss, and abdominal pain. Skin manifestations include purpura, edema, and painful nodules along the course of arteries. Cardiac involvement occurs as myocarditis, pericarditis, and arrhythmias. Angiography may show aneurismal dilatation and segmental stenosis.

Hyperkalemia in CKD

Hyperkalemia Tx: • Usually seen in stage 3 in association with ACEI • Low potassium diet, avoid additional drugs that can raise K+ (like NSAIDs) • Stop ACEI or ARB if K is consistently high >5 mEq/L

A 9-year-old boy presents with headache and dark cola colored urine that appeared 2 days after a respiratory tract infection, for which he was given amoxicillin. He has microscopic hematuria and proteinuria. The rest of his past medical history and family history are non-contributing. Physical examination reveals elevated blood pressure (120/80), and the rest of the examination is normal. Urinalysis shows hematuria (the presence of erythrocytes and erythrocytes casts) and protein loss of 2.8 grams. Question What is the most probable diagnosis? Answer Choices 1 IgA nephropathy 2 Alport Syndrome 3 Amoxicillin side effect 4 Acute poststreptococcal glomerulonephritis 5 Hemolytic-uremic syndrome (HUS)

IgA nephropathy Both gross and microscopic hematuria a couple of days after nonspecific upper respiratory tract infection and hypertension in male patients are highly suggestive on IgA nephropathy (Bergers disease). IgA nephropathy is the most common chronic glomerular disease worldwide. It may also be associated with gastroenteritis, acute or chronic renal failure, or may be asymptomatic when erythrocytes (RBCs), RBC casts, and proteinuria are discovered on urinalysis. Some patients also have hypertension. Pathophysiological mechanisms are subendothelial deposits of amorphous material that lead to vascular occlusions, mechanical RBC, and platelet damage, resulting in prothrombotic state. Alport syndrome is hereditary X-linked dominant hereditary nephritis that will also present with hematuria (asymptomatic or gross) 1-2 days after upper respiratory infection. This progressive hereditary nephritis will, however, be accompanied with bilateral sensorineural deafness and visual problems (patognomonical extrusion of central part of lenses into anterior ocular chamber). Amoxicillin side effects are not probable. Amoxicillin side effects include nausea, vomiting, rashes, antibiotic-associated colitis, and diarrhea, in addition to more rare side effects such as mental changes, lightheadedness, insomnia, confusion, anxiety, sensitivity to lights and sounds, and unclear thinking. Even allergy to amoxicillin presents with a change in mental state initially, followed by itching skin rash, fever, nausea, and vomiting any time during the treatment up to a week after treatment has stopped. Acute overdose of amoxicillin may manifest with renal dysfunction, lethargy, and vomiting, but this usually happens in very young children. Acute poststreptococcal glomerulonephritis can present with the same clinical picture: sudden hematuria, edema, and hypertension, usually together with non-specific constitutional symptoms. However, there is always a latent period between the streptococcal infection and the onset of signs and symptoms of acute glomerulonephritis. Latent period is 1-2 weeks after a throat infection and 3-6 weeks after a skin infection. Hemolytic-uremic syndrome (HUS) is acute renal failure associated with non-immune (Coombs-negative) microangiopathic hemolytic anemia and thrombocytopenia. It is the most common cause of acute renal failure in children (though it may occur in adults as well). In HUS, there is usually a prodromal gastroenteritis, fever, or bloody diarrhea for 2-7 days before the onset of renal failure, sometimes with central nervous system signs (irritability, lethargy, even seizures). Acute renal failure with anuria follows. Physical findings may reveal hypertension, edema, fluid overload, and severe pallor.

A 51-year-old woman who has been in a road traffic accident presents with multiple injuries. She has a urine output of 350 mL over the previous 24 hours. Her serum creatinine level has increased from 4.2 mg/dL to 4.8 mg/dL over the same period. What drug can be used to convert this patient's oliguric renal failure to the non-oliguric type, and to facilitate further management? Answer Choices 1 Potassium sparing diuretics 2 Thiazide diuretics 3 Carbonic anhydrase inhibitors 4 Loop diuretics 5 Osmotic diuretics

Loop diuretics Loop diuretic drugs produce diuresis in patients with an acute renal failure (ARF), congestive heart failure, and acute pulmonary edema. Loop agents can increase the rate of urine flow and enhance potassium excretion in cases of acute renal failure. They are used to convert oliguric renal failure to non-oliguric renal failure, and to facilitate fluid and electrolyte management. They do not, however, shorten the duration of renal failure or reduce mortality. Potassium sparing diuretic drugs are mild efficacy drugs with a relatively weak diuretic action; they are often used in combination with loop diuretics or thiazides in patients with congestive heart failure. Thiazide diuretics are used to treat hypertension, congestive cardiac failure, and edema. Carbonic anhydrase inhibitors produce a mild diuretic effect; they are used to treat glaucoma. Osmotic diuretics may be used to reduce intracranial pressure in cerebral edema.

Metabolic Acidosis

Low HCO3 and Low pH

Respiratory Acidosis

Low pH and High CO2

MCC of CKD

MC cause of CKD is DM!!! HTN is 2nd leading cause!!!!

Tx of Metabolic Acidosis

Metabolic Acidosis: Low serum bicarb and low pH • Tx is alkali therapy 0.5 meq/kg per day to maintain normal serum bicarb; give them sodium bicarb or sodium citrate o Be aware of volume overload because these contain sodium

Which one of the following organisms is the cause of epididymitis in young men under 35 years Answer Choices 1 N. gonorrhea 2 P. aeruginosa 3 C. acuminata 4 T. pallidum 5 H. ducreyi

N gonorrhea Explanation The cause of epididymitis in young men under 35 years is usually sexually transmitted organisms such as N. gonorrhea and C. trachomatis. T. pallidum causes syphilis and H. ducreyi causes chancroid.

Hyponatremia evaluation

Most often, hyponatremia is due to excessive water retention rather than a true sodium deficiency. The first step in evaluating hyponatremia is to determine serum osmolality. Knowing whether the serum is isotonic (normal osmolality), hypotonic (low osmolality), or hypertonic (high osmolality) can help determine the etiology of the hyponatremia, and therefore, treatment. The most common causes of isotonic hyponatremia are hyperproteinemia and hyperlipidemia. The most common causes of hypertonic hyponatremia are hyperglycemia, presence of radiocontrast agents, and the presence of inactive metabolites, that is, mannitol, sorbitol, glycerol, and maltose. Treatment is aimed at correcting the underlying disorder. Most commonly, hyponatremia occurs in the setting of low osmolality (hypotonic). To further evaluate the etiology of the hyponatremia, it must be determined if the patient is hypovolemic, euvolemic, or hypervolemic. Hypovolemic hyponatremia is usually due either to extrarenal or intrarenal sodium losses. Extrarenal losses occur from dehydration, diarrhea, and vomiting. Urinary sodium measures <10 mEq/L (normal, >20 mEq/L), as the kidneys are avidly retaining sodium in an attempt to restore volume. Treatment is directed at restoring volume. Intrarenal sodium losses occur from the use of diuretics and ACE inhibitors, nephropathies, and mineralocorticoid deficiency. Urinary sodium measures >20 mEq/L. Treatment is directed at reversing the underlying cause. The most common causes of euvolemic hyponatremia are SIADH, postoperative hyponatremia, hypothyroidism, psychogenic polydipsia, and endurance exercise. In these cases, electrolyte-free water is retained, which results in a true physiologic hyponatremia. Treatment is directed at correcting the underlying abnormality and replacing sodium losses. Hypervolemic hyponatremia is caused by congestive heart failure, liver disease, nephrotic syndrome, and advanced CKD in general, anything that causes fluid retention. Treatment is directed at treating the underlying disease, restricting water intake, and facilitating excretion of water.

24 hour urine collection for protein, If 3-3.5 g protein=

Nephrotic syndrome! Send to nephrologist immediately!

A 42-year-old African American male is admitted to the hospital with heme positive urine and anemia. He recently completed a course of trimethoprim/sulfamethoxazole for a urinary tract infection. Treatment for his condition should include which of the following? A Begin vancomycin 1 GM IV q 12 for untreated infection B No treatment C Restart bactrim IV for an undertreated UTI D Transfuse packed red cells E Transfuse platelets

No treatment No treatment is necessary, except to avoid known oxidant drugs. The patient is not exhibiting signs of an untreated infection, so A is wrong. The anemia is transient so no transfusions are needed.

CKD stage 1 GFR

Normal GFR of 120 to a GFR of 90

Anemia of CKD and tx

Normocytic normochromic anemia caused by low epo production: • Tx erythropoetic agents like Epogen or Aranesp • Target Hgb 10-11.5 (normal is 13 g/dL) • Make sure the patient is not iron deficient anemic aswell because Epogen will not work if IDA is not treated

A 56-year-old African American man with a history of hypercholesterolemia and obesity presents with a 6-week history of reduced urinary stream. Upon further questioning, he also admits to generalized weakness, loss of appetite, and a dull lower backache during this time, which he attributes to being "out of shape." He denies fever, chills, chest pain, shortness of breath, abdominal pain, hematuria, frequency, hesitancy, flank pain, dysuria, abnormal penile discharge, a history of trauma, alcohol use, or smoking. He states that he had an extramarital affair about 2 months ago and did not use any barrier methods during sexual intercourse. The physical exam is notable for a male in no acute distress with normal vital signs. He is found to have a distended bladder on abdominal exam, vertebral tenderness of the lumbosacral spine, and firm, nontender nodularity of prostate upon digital rectal exam. Question What is the most likely diagnosis in this patient? Answer Choices 1 Acute bacterial prostatitis 2 Benign prostatic hypertrophy 3 Bladder cancer 4 Prostate cancer 5 Urolithiasis

Prostate Cancer Explanation This patient's presentation is most consistent with prostate cancer. Risks associated with prostate cancer include a high-fat diet, family history, and African American ethnicity. Upon digital rectal exam (DRE), it may manifest as focal nodules or areas of induration within the prostate. Obstructive voiding symptoms can occur with prostate cancer or benign prostatic hypertrophy; however, the prostate is more likely to demonstrate the absence of the median sulcus in BPH, not nodularity, as is observed in cancer. Manifestations of metastatic and advanced prostate cancer may also include weight loss and loss of appetite, anemia, bone pain (with or without pathologic fracture, most likely of the lumbar spine), neurologic deficits from spinal cord compression, and lower extremity lymphedema secondary to lymph node metastasis. Acute bacterial prostatitis typically presents with fever, chills, malaise, arthralgias, myalgias, perineal or prostatic pain, dysuria, and obstructive and irritative urinary tract symptoms, including frequency, urgency, dysuria, nocturia, hesitancy, weak stream, and incomplete voiding. There may also be lower abdominal or back pain and spontaneous urethral discharge. The prostate will be tender, nodular, hot, boggy, or normal-feeling on digital rectal examination in acute prostatitis. Suprapubic abdominal tenderness and an enlarged tender bladder due to urinary retention may also be present. Absence of systemic symptoms and persistence of pain for at least 3 months indicates chronic prostatitis. Risk factors that favor the development of bladder cancer include cigarette smoking and exposure to industrial dyes or solvents. Common presenting findings include gross or microscopic hematuria and irritative voiding symptoms such as frequency and urgency. Metastasis may cause hepatomegaly, lymphadenopathy, and lymphedema associated with involvement of pelvic lymph nodes. A form of nephrolithiasis, ureterolithiasis is caused by calculi in the ureters. It presents as abrupt, severe, colicky pain in the flank and ipsilateral lower abdomen. There is often radiation to testicles or vulvar area with intense nausea with or without vomiting. There is significant costovertebral angle tenderness; pain can move to the upper/lower abdominal quadrant coinciding with the migration of the ureteral stone. Patients typically are constantly changing body positions, such as writhing and pacing about. Tachycardia, hypertension, and microscopic hematuria are common.

Role of Kidneys

Role of the kidneys: • Homeostasis o Maintenance of body fluids by ADH o Regulation of Na, K, Ph concentrations o Excretion of metabolic waste products- urea, creatinine, uric acid o Acid-base balance o Drug and toxin metabolism and elimination • Hormonal o EPO production- hormone that stimulates RBC production o Vit D and calcium metabolism o RAAS (renin angiotensin system)

An 82-year-old woman living in a nursing home has her routine biannual CBC, sequential multiple analysis-7 (SMA-7), and urinalysis (UA). She is a widowed nonsmoker and nondrinker with 3 grown children who visit her weekly. Her CBC and SMA-7 come back normal. Her UA shows several WBCs and 105 colony-forming units per mL of urine. The patient has no symptoms and no known drug allergies. Question What is the most appropriate next step? Answer Choices 1 Send specimen for culture and sensitivity and base treatment on results 2 Treat with ciprofloxin b.i.d. for 7 days 3 Treat with ciprofloxin b.i.d. for 3 days 4 Observation without antibiotic treatment 5 Treat with ciprofloxin, trimethoprim, orsulfamethoxazole for 3 days then repeat UA to confirm resolution

The correct response is observation without antibiotic treatment. In 2004, the United States Preventive Services Task Force recommended against screening for asymptomatic bacteriuria in men and non-pregnant women. This recommendation was based on studies showing a lack of benefit in treating these patients, coupled with the potential harm in treating (in the form of adverse effects from antibiotics and development of bacterial resistance). The USPSTF updated this recommendation in 2008 based on further studies between 2004 and 2008; it corroborated previous studies showing a lack of benefit in treating asymptomatic bacteriuria. It is still recommended to screen pregnant women between 12 - 16 weeks gestation, or at the 1st prenatal visit. This recommendation is based on studies that show the detection and treatment of asymptomatic bacteriuria in pregnant women reduces the incidence of symptomatic maternal urinary tract infections and low birth weight.

A 29-year-old Caucasian man with a prior history significant for right cryptorchidism presents for a routine physical examination. His prior cryptorchidism was corrected by orchiopexy at the age of 6 months. He has no complaints at present. His physical exam reveals no abnormalities other than bilateral gynecomastia and a right testicular mass which is painless and firm; it measures approximately 1.5 cm in diameter. The right testicular mass does not transilluminate, nor does it disappear when the patient lies supine. There is no femoral or inguinal lymphadenopathy, nor are there palpable hernias. Question What is the most likely diagnosis? Answer Choices 1 Testicular malignancy 2 Scrotal hydrocele 3 Epididymitis 4 Orchitis 5 Varicocele

Testicular malignancy Explanation This patient's physical exam findings suggest testicular malignancy. In the United States, the incidence of testicular cancer in African-Americans is approximately 1/4 of that in Caucasians. Within a given race, individuals in the higher socioeconomic classes have approximately twice the incidence of those in the lower classes. Testicular cancer is slightly more common on the right side than on the left, coinciding with higher rates of right cryptorchidism. Of all acquired and congenital risk factors, cryptorchidism is the strongest associated risk factor associated with testicular cancer. Placement of the cryptorchid testis into the scrotum (orchiopexy) does not alter the malignant potential of the cryptorchid testis; however, it does facilitate examination and tumor detection. The most common symptom of testicular cancer is a painless enlargement of the testis. Enlargement is usually gradual, and a sensation of testicular heaviness is not unusual. Acute testicular pain is seen in approximately 10% of cases, and may be the result of intratesticular hemorrhage or infarction. Approximately 10% of patients are asymptomatic at presentation, and the tumor may be detected incidentally following trauma or by the patient's sexual partner. On physical exam, a testicular mass or diffuse enlargement is found in most cases. The mass is typically firm and nontender, and the epididymis should be easily separable from it. A hydrocele may accompany the testicular tumor and help to camouflage it. Transillumination of the scrotum can help to distinguish between these entities. Palpation of the abdomen may reveal bulky retroperitoneal disease; assessment of supraclavicular, scalene, and inguinal nodes should be performed. Gynecomastia is present in 5% of all germ cell tumors, but may be present in 30 - 50% of Sertoli and Leydig cell tumors. Its cause seems to be related to multiple complex hormonal interactions involving testosterone, estrone, estradiol, prolactin, and hCG. Hemoptysis may be seen in advanced pulmonary disease. Findings consistent with epididymitis are an enlarged, tender epididymis associated with fever, urethral discharge, and irritative voiding symptoms. A translucent, fluid-filled hydrocele should be visualized by transillumination of the scrotum. Orchitis presents with inflamed, painful, tender, and swollen testes. A varicocele, which is an engorgement of the pampiniform plexus of veins in the spermatic cord, should disappear when the patient is in the supine position. It is frequently described as a 'bag of worms' that is separate from the testes.

A 70-year-old man presents with a 3-month history of urinary leaking. He notes that he suddenly feels the need to urinate. He notes symptoms when at rest and when asleep; he has not had any symptoms when coughing or bearing down. He has not experienced urinary dribbling, dysuria, foul smelling urine, or fever. He has daily bowel movements. He has no other known medical problems, and he takes no medications. His temperature is 98 degrees Fahrenheit. His bladder is not distended, and he walks without difficulty. His post void residual is 30 ml. His rectal examination is normal, and his urinalysis is unremarkable. Question What is the most likely diagnosis? Answer Choices 1 Stress incontinence 2 Urge incontinence 3 Overflow incontinence 4 Mixed incontinence 5 Fecal impaction

Urge Incontinence Explanation This patient most likely has urge incontinence. In urge incontinence, patients typically have involuntary leaks, increased urinary frequency, and nocturnal incontinence either during or just after the sensation of needing to void. Symptoms are not exacerbated by increased abdominal pressure or the stress of coughing/sneezing. Bladder detrusor muscles may be overactive, leading to the unexpected release of urine. Treatments include scheduled voiding and anticholinergic medications (oxybutynin, etc.). Stress incontinence is characterized by the involuntary leaking of urine during stress or increases in abdominal and bladder pressure, such as coughing and sneezing. Bladder pressure at these times exceeds urethral pressure, allowing urine to leak through the urethra. Urinary tract deficits are found commonly in older patients; both men and women have decreased bladder sensation, decreased contractility, and involuntary bladder contractions, which predispose them to incontinence. Obesity, pregnancy, and vaginal births may increase the risk for stress incontinence. In such cases, the pelvic floor muscles may be insufficiently strong to support the urethra and overcome pressure of urine flowing from the bladder. Mixed incontinence refers to the presence of symptoms of both stress and urge incontinence. It may be seen in 1/3 of patients. Overflow incontinence refers to urinary leaks that occur due to an obstruction of urine flow. In the absence of urinary retention, post void residuals are typically elevated; normal post-void residuals in the absence of retention are less than 200 ml. His 30 ml post-void urine volume is not consistent with urinary retention. Initially, patients with urinary retention may experience dribbling after voids, straining, the sensation of a full bladder, and a constant urge to void. Prostatic hypertrophy, atonic bladders, etc. can impede urine flow. Once urine volume exceeds bladder capacity, it may spill out, causing a leak. Overflow incontinence may be distinguished from urge incontinence by urodynamic testing, and it may be treated with terazosin and finasteride. Many medical problems and medications can contribute to incontinence. Delirium, restricted mobility, urinary infection, fecal impaction, polyuria, and medications that decrease urethral pressure (e.g., alpha blockers, neuroleptics, and benzodiazepines) or increased bladder pressure (e.g. anticholinergics, beta blockers, anti-Parkinson's medications, and bethanechol) may contribute to incontinence. The normal rectal examination and the history of regular bowel movements make fecal impaction a less likely diagnosis in this case.

Volume Overload tx

Volume overload treatment: • Restrict sodium intake < 2g day • Daily loop diuretic (furosemide) if edema o Thiazide diuretics ineffective if GFR <30 ml/min o If diuresis is not achieved with a loop diuretic, then try Metolazone

What is Chronic Kidney Disease

What is CKD? • > 3 months with Low GFR <60 ml/min

A 59-year-old female with history of non-insulin-dependent diabetes mellitus (NIDDM), hypertension, and chronic kidney disease (CKD) returns for follow-up of her labs. You note that she her GFR has decreased from 40 to 36 ml/min/1.73m 2 . What stage of chronic kidney disease is she currently? A 1 B 2 C 3 D 4 E 5

c 3 There are five stages of CKD. Stage 3 Chronic kidney disease is referred to as a moderately decreased GRF between 30-59 ml/min/1.73m 2 . All other choices reflect different ranges of GRF above or below stage 3.

A 45-year-old woman presents as a new patient. She was recently seen in the emergency department for right flank pain, and a CT scan revealed a right-sided ureteral stone. The stone was 4mm, a passable size, and she was sent home with analgesics and advised to hydrate well and strain her urine. She was straining her urine and noticed a small, dark fleck. She brought the sediment to the urologist's office to undergo a stone analysis, as this is her first episode of a renal or ureteral stone. Question What is the most likely composition of this stone? Answer Choices 1 Uric acid 2 Struvite 3 Cystine 4 Magnesium 5 Calcium

calcium The correct answer is calcium. Calcium stones are the most common type of renal calculi and can be further characterized as either calcium oxalate or calcium phosphate stones. Calcium oxalate stones make up about 60% and calcium phosphate stones make up about 20%, which results in calcium stones being about 80% of stones overall. Geography, fluid intake, and diet can all influence stone formation, but metabolism and genetics can also play a role. Absorptive hypercalciuria, renal hypercalciuria, and resorptive hypercalciuria can all result in calcium stone formation. Hyperuricosuria, gout, hyperoxaluria, and hypocitraturia are all other causes of calcium calculi. Uric acid is the not the correct answer, as this is not the most common type of renal calculus. Uric acid stones make up about 10% of stones in the United States. Hyperuricosuira and/or a urinary pH less than 5.5 are the 2 most common causes of uric acid stones. Gout, increased turnover of nucleic acids (such as in polycythemia or psoriasis), increased purine intake, and alcohol consumption are all causes of hyperuricosuria and can therefore put a patient at risk for uric acid stones. Struvite is not the correct answer, as this is not the most common type of renal calculus. Struvite stones make up about 7% of total renal calculi. Urinary tract infections secondary to the presence of urea splitting organisms can result in the formation of struvite stones. Klebsiella, Proteus, Staphylococcus, and Pseudomonoas produce urease. Urease breaks down urea and aids in the formation of ammonia. The ammonia then undergoes hydrolysis, which results in alkaline urine and reduced solubility of struvite, as well as urine that is supersaturated in struvite. Cystine is not the correct answer, as this is not the most common form of renal calculi. Cystine stones make up about 3% of total renal calculi. Cystine stones form due to an autosomal recessive disorder in the metabolism of cystine, which leads to cystinuria. Once the urinary saturation of cystine is more than 250 mg/L, cystine stones can start to form. Magnesium is not the correct answer, as this is not a type of renal calculi. In fact, magnesium has been known to be preventative of stone formation and is, therefore, a component in a lot of renal calculi prevention measures. Hypomagnesuria, usually dietary in nature, is a known risk factor for renal calculi formation. Maintaining urine magnesium about 50 mg/day is preventative.

The mother of a 2-year-old Caucasian boy explains that the boy has had a 4-day history of fever and cries during urination. She has been giving him acetaminophen, but the fever keeps coming back. She also says that his urine "smells funny." Family history is positive for a 4-year-old brother with Down syndrome. His temperature is 39°C. Urinalysis and culture reveal urinary tract infection. A VCUG and a renal bladder ultrasonography reveal vesicourethral reflux (VUR). Question What is a risk factor for VUR in this boy? Answer Choices 1 His gender 2 Acetaminophen treatment 3 His age 4 His ethnicity 5 Family history of Down syndrome Your Answer: Correct His ethnicity

his ethnicity Ethnicity is a significant risk factor for vesicourethral reflux (VUR) because it is 10 times more common in Caucasians than African-Americans. VUR is the retrograde flow of urine from the bladder towards the kidneys due to a dysfunctional vesicoureteric junction. It may occur alone or in association with other urological conditions like posterior urethral valves. Although prevalence is more common in male neonates, VUR is 5 times more common in female children compared to male children after the age of 1 year. Acetaminophen treatment and family history of Down syndrome have no association with VUR.

Normal amount of Protein Excretion by the kidneys

no more than 150 mg/day

What category of pharmaceutical agents is commonly used to treat benign prostatic hypertrophy (BPH)? Answer Choices 1 Alpha-1 adrenergic antagonists 2 Alpha-1 adrenergic agonists 3 Alpha-2 adrenergic antagonists 4 Alpha-2 adrenergic agonists 5 5-alpha reductase agonists

Alpha-1 adrenergic antagonists Explanation The correct response is alpha-1 adrenergic antagonists. Benign prostatic hypertrophy (BPH) is a condition that arises as a result of proliferation of stromal cells in the periurethral glands of the prostate. Consequently, whereas cancer most commonly arises in the outer layer of the prostate, BPH is a condition associated with the inner portion or transition zone. BPH is a condition that accompanies aging. By age 60, approximately 50% of men have microscopic evidence of BPH; by age 85, about 85% of men will have BPH. The exact cause of BPH remains unclear; however, it is most likely the result of changes in the local hormonal milieu of the prostate. Dihydrotestosterone (DHT) is produced in the prostate via conversion of testosterone by the 5-á reductase enzyme. DHT is known to promote stromal proliferation. It is hypothesized that elevated estrogen levels that accompany aging increase the density of DHT receptors in the prostate, leading to hyperplasia. In addition, the density of alpha-1 adrenergic receptors, which contract smooth muscle cells in the prostate, also appears to increase with age, contributing to the obstructive urinary symptoms that typically bring patients with BPH to clinical attention. Microscopic prostatic hyperplasia in and of itself is not dangerous; there is no known association between BPH and prostate cancer. However, prostatic enlargement around the urethral canal can cause urinary outflow obstruction. Clinically, patients report urinary hesitancy, decreased force of urination, interrupted urinary stream, and a feeling of incomplete bladder emptying. Recurrent urinary tract infections may also occur because of residual urine pooling. In severe cases, hydronephrosis may result. In patients with BPH, digital rectal examination reveals lateral and/or median lobe prostatic enlargement. However, prostate size correlates poorly with obstructive symptoms and urodynamic studies are the most useful means of quantifying the severity of BPH. Peak flow rates under 10 ml/sec and residual urine volumes of greater than 150 ml are indicators of significant obstruction. There are 2 main categories of pharmacological agents used to treat BPH: alpha-1 adrenergic antagonists and 5-alpha reductase inhibitors. Alpha-1 adrenergic antagonists (e.g., terazosin, doxazosin, tamsulosin) relax prostatic smooth muscle by blocking the action of alpha-1 adrenergic agonists, reducing obstructive symptoms. By contrast, 5-alpha reductase inhibitors (e.g., finasteride) reduce DHT levels by blocking enzymatic conversion of testosterone. It should be noted that finasteride also lowers prostate specific antigen (PSA) levels, which may complicate the use of PSA as a screening test for prostate cancer.

The patient is a 68-year-old male retired steel worker and a past smoker with a 36-pack-year history. He has been under the care of a urologist for about 5 years, ever since an episode of gross hematuria led to the diagnosis of bladder cancer. He is s/p transurethral resection of bladder tumor (TURBT) on 2 occasions now for noninvasive carcinoma in situ. He still follows with surveillance cystoscopy every 3 months, and a recent appointment revealed another bladder tumor. Another TURBT is scheduled, but he will also need adjunctive treatment. Question What medication can be administered intravesically to treat the bladder cancer in this patient? Answer Choices 1 Bacillus Calmette-Guérin 2 Ciprofloxacin 3 Potassium chloride 4 Sulfamethoxazole-trimethoprim 5 Pentosan polysulfate sodium

Bacillus Calmette-Guérin Explanation The correct answer is bacillus Calmette-Guérin (BCG), as this is a common intravesical treatment for patients who have either recurrent bladder cancer or multiple tumors at one time. BCG is administered to patients with bladder cancer on an outpatient basis, typically weekly for 6 weeks, but can also be given as maintenance monthly or every 6 months after initial treatment at the urologist's discretion. After treatment, bladder surveillance is extremely important, as bladder cancer recurrence rates can be as high as 50% over 5 years. Bladder cancer surveillance consists of outpatient cystoscopy and urinary cytology every 3 months for 18-24 months, every 6 months for 2 years, and then annually. Ciprofloxacin is not the correct answer, as this is an antibiotic that is typically used to treat urinary tract, upper respiratory, and skin infections. Ciprofloxacin is available in oral and intravenous forms, but not intravesical. Potassium chloride (KCL) is also not the correct answer. While KCL could, in fact, be administered intravesically, it is not used to treat bladder cancer. It is sometimes administered intravesically as a "potassium sensitivity test" to help with the diagnosis of interstitial cystitis. 40ml KCL solution is administered through a catheter, and then the patient's pain and urgency are rated compared to administration of plain water. Sulfamethoxazole-trimethoprim is also not the correct answer, as this is an antibiotic that is typically used to treat urinary tract, upper respiratory, and skin infections. Sulfamethoxazole-trimethoprim is available in oral and intravenous forms, but not intravesical. Pentosan polysulfate sodium is also not the correct answer. This medication is available in an oral form and can also be administered intravesically as a bladder instillation. It is used in both forms for the treatment of interstitial cystitis, not bladder cancer.

A 61-year old female patient who is a smoker is undergoing a routine physical examination in your family practice. She is otherwise asymptomatic but her urinalysis reveals microscopic hematuria. Your next definitive step would include which of the following? A Refer her for a spiral CT scan of the kidneys to ascertain where her renal calculi may be and to rule out hydronephrosis. B Repeat her urinalysis in the morning after asking the patient to hydrate with at least eight glasses of water in the interim. C Send her urine for cytology and refer her to a urologist to rule out bladder cancer. D Send her urine for a C&S and depending upon the result, start her on antibiotics E Start a course of levofloxacin to resolve her occult urinary tract infection.

C Send her urine for cytology and refer her to a urologist to rule out bladder cancer. C Painless hematuria must always include bladder cancer in the differential diagnosis. Without another reasonable explanation—something that is not the case in this patient—bladder cancer must be ruled out, beginning with cytology and subsequent referral to an urologist.

A 45-year-old woman presents as a new patient. She was recently seen in the emergency department for right flank pain, and a CT scan revealed a right-sided ureteral stone. The stone was 4mm, a passable size, and she was sent home with analgesics and advised to hydrate well and strain her urine. She was straining her urine and noticed a small, dark fleck. She brought the sediment to the urologist's office to undergo a stone analysis, as this is her first episode of a renal or ureteral stone. Question What is the most likely composition of this stone? Answer Choices 1 Uric acid 2 Struvite 3 Cystine 4 Magnesium 5 Calcium

Calcium Explanation The correct answer is calcium. Calcium stones are the most common type of renal calculi and can be further characterized as either calcium oxalate or calcium phosphate stones. Calcium oxalate stones make up about 60% and calcium phosphate stones make up about 20%, which results in calcium stones being about 80% of stones overall. Geography, fluid intake, and diet can all influence stone formation, but metabolism and genetics can also play a role. Absorptive hypercalciuria, renal hypercalciuria, and resorptive hypercalciuria can all result in calcium stone formation. Hyperuricosuria, gout, hyperoxaluria, and hypocitraturia are all other causes of calcium calculi. Uric acid is the not the correct answer, as this is not the most common type of renal calculus. Uric acid stones make up about 10% of stones in the United States. Hyperuricosuira and/or a urinary pH less than 5.5 are the 2 most common causes of uric acid stones. Gout, increased turnover of nucleic acids (such as in polycythemia or psoriasis), increased purine intake, and alcohol consumption are all causes of hyperuricosuria and can therefore put a patient at risk for uric acid stones. Struvite is not the correct answer, as this is not the most common type of renal calculus. Struvite stones make up about 7% of total renal calculi. Urinary tract infections secondary to the presence of urea splitting organisms can result in the formation of struvite stones. Klebsiella, Proteus, Staphylococcus, and Pseudomonoas produce urease. Urease breaks down urea and aids in the formation of ammonia. The ammonia then undergoes hydrolysis, which results in alkaline urine and reduced solubility of struvite, as well as urine that is supersaturated in struvite. Cystine is not the correct answer, as this is not the most common form of renal calculi. Cystine stones make up about 3% of total renal calculi. Cystine stones form due to an autosomal recessive disorder in the metabolism of cystine, which leads to cystinuria. Once the urinary saturation of cystine is more than 250 mg/L, cystine stones can start to form. Magnesium is not the correct answer, as this is not a type of renal calculi. In fact, magnesium has been known to be preventative of stone formation and is, therefore, a component in a lot of renal calculi prevention measures. Hypomagnesuria, usually dietary in nature, is a known risk factor for renal calculi formation. Maintaining urine magnesium about 50 mg/day is preventative.

A 66-year-old man presents to the office with polyuria and erectile dysfunction. He denies any other symptoms or significant past medical history. Physical examination reveals Tanner stage 5 of the external genitalia, balanitis of an uncircumcised penis, and slightly enlarged, symmetrical and smooth prostate. His condition is most likely the result of: Answer Choices 1 Benign prostatic hypertrophy 2 Diabetes insipidus 3 Diabetes mellitus 4 Hypogonadism 5 Prostate cancer

Explanation The correct answer is diabetes mellitus since the presence of polyuria would indicate hyperglycemia and the associated erectile dysfunction and/or balanitis may be the only other presenting symptom or sign of diabetes mellitus in a male patient. Erectile dysfunction is a common vascular and neurological complication of diabetes and occurs in up to 75% of male diabetics. Elevated blood sugars result in autonomic neuropathy of the cavernous nerve of the penis so that erectile dysfunction serves as one of the earliest indications of neuropathy. Likewise, hyperglycemia results in microvascular damage to the dorsal and cavernous arteries, in the same way retinopathy, nephropathy, and neuropathy develop, further contributing to poor perfusion and erectile dysfunction. Hyperglycemia also results in the colonization of skin organisms, commonly Candida, resulting in typical superficial yeast infections seen in diabetics such as balanitis in men and vulvovaginitis in women. Benign prostatic hypertrophy (BPH) typically occurs in the periurethral zone of the prostate and usually presents with lower urinary symptoms (LUTS) that suggest obstruction (i.e. hesitancy, weak stream, straining, post-void leaking) or irritation (i.e. nocturia, frequency, urgency). Digital rectal examination of prostatic hyperplasia typically reveals a smooth, firm enlargement of the gland which may be asymmetrical or indurated. Early BPH is not typically associated with erectile dysfunction or Candidaskin infections. Prostate cancer most often develops in the peripheral zone of the prostate and is usually asymptomatic. Locally advanced prostate cancer may encroach on the central transition zone of the prostate and present with irritative urinary symptoms. Prostate cancer that extends outside the prostate capsule may result in erectile dysfunction. Carcinomas in the peripheral zone are often palpable and typically a hard, irregular nodule or induration. Prostate cancer is not typically associated with Candidaskin infections. Hypogonadism may present with fatigue, decreased libido, diminished erections, gynecomastia, or decreased testicular size, muscle mass, or hair growth associated with secondary sexual characteristics. It is typically not associated with an enlargement of the prostate, urinary complaints, or Candidaskin infections. The characteristic presentation of diabetes insipidus (DI) is abnormally large amounts of dilute urine - insipidus means tasteless. Polyuria is massive, often associated with nocturia and enuresis, and results in dehydration, which is often not evident due to a compensatory increase in thirst and polydipsia. DI is the result of the posterior pituitary's failure to secrete antidiuretic hormone (ADH) resulting in central diabetes insipidus (DI) or the kidney's resistance to ADH resulting in nephrogenic DI. DI is not typically associated with Candidaskin infections.

A 47-year-old Caucasian woman presents for evaluation of acute abdominal pain. She was brought in by her son, who reports the patient had not been eating or drinking well for several days. Further history, exam, and imaging studies were performed. The patient was pre-hydrated with sodium bicarbonate, had an abdominal CT with IV contrast, and was later admitted to the medical floor with a provisional diagnosis of diverticulitis. Her past medical history is significant for diabetes mellitus diagnosed 12 years ago and hypertension. Both conditions were reported to have been under good control. Her medications include regular and long-acting insulin and hydrochlorothiazide/lisinopril 25/20 mg QD. She has recently been taking 800mg ibuprofen BID-TID for her abdominal pain during the last week. She has no known allergies. While in the hospital, the patient's laboratory results are followed daily. 2 days after the CT with contrast, it is noted that her serum creatinine has risen to a level of 3.5 mg/dL. Records from 1 month ago at her family physician showed her labs to include a hemoglobin A1C of 6.8%, creatinine of 1.2 mg/dL, GFR of > 60 mL/min/1.73 m2, and blood pressure of 127/78. Question Which of the following is a major risk factor for this patient's sudden decline in renal function? Answer Choices 1 Baseline GFR > 60 mL/min/m2 2 Caucasian race 3 Diabetes mellitus 4 Pre-hydration with sodium bicarbonate 5 Younger age

DM Explanation Diabetes mellitus is a major risk factor for renal impairment after administration of IV contrast. In one study, DM had an odds ratio of 5.47 for development of contrast nephropathy. A baseline GFR > 60 mL/min/m2 estimates nearly normal creatinine clearance. Individuals with significantly reduced GFRs are at high risk for contrast nephropathy. Because creatinine clearance is inversely related to the creatinine level, this patient's lower creatinine level also supports less risk for the contrast nephropathy. However, it was not enough to offset the risk from her diabetes. Caucasian race is not considered a major risk factor in developing contrast nephropathy. Race is not a major factor in the development of contrast nephropathy. However, if any race is implicated for higher risk, it would be African Americans. Pre-hydration with sodium bicarbonate is a means of reducing risk, rather than a major risk factor, for contrast nephropathy. Pre-hydration seems to confer a protective effect, preventing hypotension and decreased renal blood flow. The sodium bicarbonate has been suggested in many studies to be more effective than traditional saline (sodium chloride). Younger age is not a risk factor for contrast nephropathy. Elderly age is a risk factor, with particular concern arising in patients 75 years of age and older.

Your 52-year-old male patient has chronic renal failure on dialysis for 1 year. He is suffering from anemia and hypoproteinemia. He complains of bone pains, which have developed recently. Renal osteodystrophy is diagnosed after appropriate tests and you plan vitamin D therapy. What is the role of the kidneys in the synthesis of vitamin D? Answer Choices 1 Hydroxylation of 25-hydroxy vitamin D3 at the number 1 position 2 Hydroxylation of 1,-hydroxy vitamin D3 at the number 25 position 3 Activiation of cholesterol to cholecalciferol 4 Conversion of calcitriol to cholecalciferol 5 Providing the cholesterol for synthesis of vitamin D

Hydroxylation of 25-hydroxy vitamin D3 at the number 1 position Explanation Activation of Vitamin D is completed by the kidneys when they hydroxylate it at the number 1 position. The 25-hydroxy cholecalciferol is converted to 1, 25 dihydroxycholecalciferol (calcitriol) by the kidneys by 1-alpha-hydroxylase. This activates vitamin D for use in calcium and phosphorus uptake from the intestinal lumen. Hydroxylation of vitamin D3 at the number 25 position is accomplished by the liver. Cholesterol is activated to cholecalciferol by sunlight (UV light) in the skin. Once active, vitamin D3 is placed in the intestinal epithelial cells. Excess can be lost through the lumen. Cholecalciferol cannot be converted to calcitriol without the intermediate hepatic step. Calcitriol is not converted to cholecalciferol. Cholesterol is synthesized by the liver or from dietary sources.

A 3-year-old girl presents with progressive abdominal enlargement associated with abdominal pain and occasional vomiting. Physical examination shows a palpable mass over the right upper quadrant extending to the right flank. She looks pale and the BP is slightly elevated. Urinalysis shows microscopic hematuria. What is the most likely diagnosis? Answer Choices 1 Wilm's Tumor 2 Neuroblastoma 3 Nephroblastomatosis 4 Renal Cell Carcinoma 5 Mesoblastic Nephroma

Wilm's tumor Explanation The clinical picture is suggestive of Wilm's Tumor. It accounts for most renal tumors in childhood during the first 5 years of life. It affects both sexes equally. It is a solitary growth that affects either part of the kidneys. There are congenital anomalies associated with it, most commonly the GUT anomalies, hemihypertrophy, sporadic aniridia, and mental retardation. It is commonly manifested by an abdominal mass that is described as generally smooth, firm and rarely crosses the midline, and it causes abdominal pain and vomiting. Hypertension is seen in 60% of the patients either due to elaboration of renin by the tumor cells or due to compression of the renal vasculature by the tumor. Hematuria is also uncommon and mostly microscopic. CT scan confirms the diagnosis which will show an intrarenal tumor, therefore ruling out Neuroblastoma. Treatment is by surgical removal. Chemotherapy is indicated post-operatively for the residual tumor. Neuroblastoma is a malignancy of the neural crest. It is the most common solid tumor in children outside the CNS. It is slightly more common in males and whites and median age of diagnosis is 2 years old. It arises mostly in the abdomen either in the adrenal gland or retroperitoneal sympathetic ganglia followed by the thoracic area mostly seen in the posterior mediastinum. Other sites are the head, neck, and epidural area. Tumors in the head and neck region are sometimes associated with Horner's Syndrome (Mioisis, Ptosis, Anhidrosis, and Enophthalmos). Diagnosis is by CT scan or MRI but pathologic diagnosis is made by biopsy. Tumor markers such as VMA and HVA (Homovanillic Acid) help confirm the diagnosis. Treatment is surgery, chemotherapy, and radiation depending on the stage of the tumor. Nephroblastomatosis are immature renal elements called Nephrogenic rest. It is a Wilm's tumor precursor lesion that is both unifocal and deep within the Renal parenchyma (intralobar rest) or multi-focal (perilobar rest). Subsequent development of Wilm's tumor in the other kidney is more likely in patients with this feature; therefore prompt inspection of the contralateral kidney is necessary during surgery of the neprhogenic rest. CT scan follow-up should also be done. Renal Cell Carcinoma is rare during the first decade of life but can occur occasionally in teenagers. Initial presentations are abdominal mass and hematuria. Surgical resection may offer cure, but prognosis is poor with post-operative residual disease. Mesoblastic Nephroma is a massive, firm, solitary renal mass and is generally thought to be benign. It resembles Leiomyoma or low-grade leiomyosarcoma grossly and microscopically. It also accounts for the majority of congenital renal tumors. It is more often seen in males and noted to produce renin. Treatment is surgical resection.

An 8-year-old male presents to your office with his parents for an evaluation of his bedwetting. He is being teased about his bedwetting on overnight outings and his parents are concerned about his self-esteem. He has never had a prolonged period of dryness at night. He has no wet accidents during the day and has not had difficulties with stool incontinence. His parents report that his mother wet the bed until age twelve. They state that he appears to be a very sound sleeper. He is on no medications and his past medical history is unremarkable. The physical examination is within normal limits. Question Which of the following is considered first line intervention due to proven evidenced success rates? Answer Choices 1 Bed-wetting alarm system 2 Imipramine administration 3 Desmopressin acetate administration 4 Behavioral therapy-repeated wakening at night 5 Fluid restriction

Bed-wetting alarm system Explanation Bedwetting, or enuresis, is a common problem in the pediatric age group. Prevalence at the age of 5 years is 7% in males and 3% in females. At the age of 10 years the prevalence is 3% for males and 2% in females. Enuresis is divided into primary enuresis, where the child has never been dry at night, and secondary enuresis, where the child who has been continent for at least 6 months starts to wet the bed again. Primary nocturnal enuresis is associated with a smaller bladder capacity, abnormal arousal patterns during sleep, and inappropriate or inadequate toilet training. Secondary enuresis is generally precipitated by situations of psychological stress, such as a recent move, marital conflict, or a new sibling in the household. Only rarely does enuresis have an organic component and usually urological procedures are not warranted. General treatment guidelines that patients and especially the parents should be educated on include voiding before retiring, limiting fluids prior to bedtime, and expecting older children to clean their own bedding. More often than not the first line intervention is considered bed-wetting alarms. These have been found in multiple studies to be extremely effective in producing a long-term cure. The bed-wetting alarms are moisture-sensing devices that are placed near the child's genitals, and are activated to trigger an alarm when the child voids in bed. These alarms go off at the initial first few drops of voiding; thus awakening the child to get out of bed and finish voiding in the toilet or hold urine until later. This evokes a conditioned response of waking and inhibiting urination. They are generally recommended in children older than seven years. Therapy is recommend for at least 3 months and used every night. Parents must get up with the child to ensure the child does not just turn off the alarm and go back to sleep. If used appropriately, bed-wetting alarms are successful in 2/3 of the patients that utilize them. Alarm therapy requires a cooperative, motivated child and family. Parental involvement plays an essential role when using alarm devices due to the consistency that is necessary. Treatment must involve education and avoidance of being judgmental and even shaming the child due to most children feeling ashamed; the goal of treatment is to help the child establish their continence and at the same time maintain or gain self-esteem. It is recommended that children use these alarm devices until they experience three weeks of complete dryness. For complete resolution of nocturnal enuresis, the bed-wetting alarm may be needed to be used for up to 15 weeks. Relapse rates are higher when the alarm system is discontinued after shorter dry periods. Studies have shown that compared with other skill-based or pharmacologic treatments, the bed-wetting alarm has a higher success rate (75%) and a lower relapse rate. Desmopressin and imipramine are the primary drugs used in the treatment of nocturnal enuresis but should not be considered first line interventions. Imipramine administration is only slightly less effective than the alarm systems in bringing about dryness, usually within 2 weeks. Long-term results are less promising as the enuresis tends to return while off the medication. Desmopressin acetate typically has an excellent response over the short-term, but again, the enuresis tends to recur when the child is taken off the medication. Fluid restriction and wearing diapers at night may resolve the consequences of the issue but will not create the behavior which will lead to the actual bedwetting from stopping; behavioral modification/training is a main key to treatment.

A 62-year-old woman presents with bright red gross hematuria for the past 2 months. She states it is painless, but persistent. She denies other symptoms. She has no chronic medical problems. Upon further questioning, she admits to a 50-pack year smoking history, and she states she is currently retired from her job in a rubber factory. Vital signs are within normal limits, and physical exam is normal. Urine dipstick only shows too numerous to count RBCs, and urine cultures are negative. Question What is the most likely diagnosis? Answer Choices 1 Acute cystitis 2 Bladder cancer 3 Renal cell carcinoma 4 Urethritis 5 Ureteral calculi

Bladder cancer Explanation Hematuria is the most common presenting sign of urinary tract cancer, and bright red gross hematuria is usually of lower urinary tract origin. Silent or painless hematuria suggests tumor or renal parenchymal disease; therefore, the clinical picture points to cancer, with bladder cancer as the most likely diagnosis. Smoking and exposure to industrial dyes or solvents (like in a rubber plant) are risk factors for bladder cancer. Bladder cancer is the 2nd most common urologic cancer, and the mean age at diagnosis is 65 years. It is more common in men than women (2.7:1), and 98% of primary bladder cancers are epithelial malignancies (majority urothelial cell carcinomas). Ordering cytology of the urine sample is often helpful with higher grade and stage bladder cancers. Patients can become anemic with chronic blood loss, so a CBC is justified. Diagnosis is made by cystoscopy with biopsy. Acute cystitis typically presents with irritative voiding symptoms (frequency, urgency, dysuria) and suprapubic discomfort in addition to possible hematuria. Urinalysis will show pyuria, bacteriuria, and varying degrees of hematuria. Urine cultures will show specific organisms. Renal cell carcinoma can present with flank pain, hematuria, persistent back pain, and an abdominal mass; also, it can be found incidentally on CT scan. RCC is more common in men than women (2:1), and it has a peak incidence in the 6th decade of life. This could be a possible option for diagnosis, but the significant history of smoking and previous work history points more toward bladder cancer. Urethritis is inflammation of the urethra that presents with urethral discharge, dysuria, and itching. Urethritis is most often caused by a STD. This patient does not have a history of unprotected sexual intercourse or any other symptoms that would indicate this diagnosis. Ureteral calculi can present with hematuria, but it also typically presents with flank or abdominal pain as well. If the stone is in the ureter, it often causes some hydroureter with or without hydronephrosis, both of which cause some pain or discomfort. The patient has no past history of forming stones.

Hypocitraturia is associated with: Answer Choices 1 Cystine nephrolithiasis 2 Uric acid nephrolithiasis 3 Struvite nephrolithiasis 4 Calcium oxalate nephrolithiasis

Calcium oxalate nephrolithiasis Explanation Various substances have been proposed to inhibit renal stone formation, and in this regard, citrate is well studied. Hypocitraturia (<470mg/24 hours) is present in up to 60% of patients with nephrolithiasis. Hypocitraturia may have secondary causes, including chronic diarrheal states, renal tubular acidosis, hypomagnesemia, urinary tract infection, and thiazide-induced hypokalemia. Hypocitraturia is associated with idiopathic calcium oxalate stone formation, and after its correction, there is a notable reduction in stone forming events.

A 73-year-old male presents to clinic with a history of blood in his urine for the past month. He has mild irritation with voiding but denies any other symptoms except an unintentional weight loss of 20 pounds in the past 6 months. His past medical history includes hyperlipidemia, seasonal allergies, and fibromyalgia. He denies alcohol use and has a 22 pack-year history of tobacco use. Which risk factor supports your diagnosis? A Fibromyalgia B Hyperlipidemia C Seasonal allergies D Cigarette smoking E Age

Cigarette smoking D Risk factors for bladder cancer include cigarette smoking (D) and exposure to industrial dyes and solvents. Age (E) is not a risk factor but the mean age of diagnosis is 73 years old and is more common in men than women (3.1:1). Fibromyalgia (A), hyperlipidemia (B), and seasonal allergies (C) have not shown any correlation to predisposing a person to bladder carcinoma.

A 13-year-old girl is drowsy and unable to answer questions; her mother says that she has been extremely thirsty lately and urinates frequently. Her father notes that the patient has also been fatigued. There is a fruity odor to the patient's breath. Blood gases are drawn and reveal the following: pH 7.3 CO2 32 HCO3 17 Question What is the most likely cause of the patient's condition? Answer Choices 1 Frequent vomiting 2 Diabetes mellitus 3 Myasthenia gravis 4 Ethylene glycol ingestion 5 Salicylate ingestion

Diabetes mellitus Explanation The patient's blood gas results indicate metabolic acidosis. Given her presentation of polydipsia, polyuria, fatigue, and fruity odor to her breath, she is likely suffering from diabetic ketoacidosis from undiagnosed diabetes mellitus. Frequent vomiting would lead to metabolic alkalosis from hydrogen loss. Myasthenia gravis would be associated with respiratory acidosis due to the associated decrease in pulmonary function and therefore decreased clearance of CO2. Ethylene glycol ingestion is associated with metabolic acidosis; however, the patient's presentation is classic for diabetic ketoacidosis. Salicylate ingestion is associated with respiratory alkalosis.

A 37-year-old man has a 5-hour history of left-sided flank pain. He was previously healthy and takes no medications or supplements. He passed several grainy pieces in his urine 2 hours ago and brought them to the emergency room. Calcium oxalate crystals were identified. What dietary modifications will help reduce chances of future episodes of nephrolithiasis? Answer Choices 1 Decreased intake of citric acid foods 2 Increased intake of oxalate rich foods 3 Increased intake of sodium 4 Increased intake of fluids 5 Increased intake of proteins

Increased fluid intake Explanation This patient has passed calcium oxalate stones in his urine. Increasing his intake of fluids will help prevent future episodes of nephrolithiasis. At least 2 liters/day of fluid intake is suggested to prevent recurrent nephrolithiasis. Increased intake of water, coffee, and beer will all lead to decreased urine concentration of stone-forming solutes; alcoholic beverages inhibit the action and secretion of anti-diuretic hormone, leading to decreased urine solute concentration. In addition, reducing the intake of oxalate-rich foods, such as rhubarb, green leafy vegetables, chocolate, tea, liver, nuts, and seeds, is recommended. Crystals form in urine when the urine is supersaturated with crystal-forming solutes such as calcium, phosphate, and uric acid. Some patients overexcrete solutes, while others drink inadequate amounts of fluids to keep solutes dissolved. Stones also occur when the urine is infected with urea-splitting bacterium (Proteus). Here, urea is broken down into ammonia and bicarbonate, which then forms ammonium hydroxide and bicarbonate, the components of struvite stones. Struvite stones consist of a triple phosphate of calcium, magnesium, and ammonium Certain stone inhibitors, such as pyrophosphate, citrate, and magnesium, prevent crystal growth. In patients who have low levels of these inhibitors, stones are more likely to form. We do not know if this patient is deficient in urinary stone inhibitors, only that he passed calcium oxalate stones. 24-hour urine collections, preferably 2 samples drawn 6 weeks apart, are recommended to assess urinary citrate excretion. Decreased intake of oranges (being a type of citrus fruit) will likely decrease urinary citrate levels. This modification will be of little benefit if his urine citrate levels are normal and may be detrimental if his levels are low. Increased intake of liver is not recommended; liver is an oxalate-rich food. Increased sodium is not advised in a patient who is prone to developing urinary calcium stones. Increased sodium intake leads to increased calcium excretion, which may promote further calcium stone formation. Increased protein intake is not recommended for patients with risk or recurrent renal stones and should be limited to less than 80/g day. Nephrolithiasis is a common problem, affecting some ~ 2 - 9% of the population. Without preventative treatment, 50% of patients having an episode of nephrolithiasis will likely have recurrent episodes over the next 10 years. Because of its likely recurrence, measures to prevent recurrent nephrolithiasis (such as increasing water intake, decreasing sodium intake, and moderating calcium intake) are all important in a patient such as this one who has documented calcium phosphate stones.

A 15-year-old boy is seen by his primary care physician for delayed sexual changes during puberty. Upon physical exam, the physician notes an overall normal appearance, with small testes and some gynecomastia. Behaviorally, the patient seems to be somewhat shy and reserved, with occasional social outbursts. Although he is tall for his age, he is uncoordinated and inactive. Measured testosterone levels are decreased. A karyotype is done, and found to contain an extra chromosome. The most probable diagnosis for this child is which of the following? Answer Choices 1 Klinefelter syndrome 2 X-linked adrenoleukodystrophy 3 X-linked MR hypotonic facies syndrome 4 Fragile X syndrome 5 XYY syndrome

Klinefelter syndrome Explanation Fragile X syndrome results from a mutation in the FMR1 gene, located on the long arm of the X chromosome (Xq27). It is characterized by moderate mental retardation. The phenotypic manifestations of fragile X syndrome vary, but often include developmental delay, hyperactivity, abnormal craniofacials, and macro-orchidism (in post-pubertal males). Greater than 99% of affected individuals have what is known as a "full mutation" in the FMR1 gene. This mutation is caused by an increased number of CGG trinucleotide repeats in the 5' end of the gene (> 230 CGG repeats), which causes aberrant methylation of the gene, and aberrant expression of the gene product. Mothers of affected children that have this full mutation are obligate carriers of a "premutation" in the FMR1 gene. This premutation or "intermediate" allele has between 55 and 230 CGG repeats, and can expand upon transmission to offspring. (The normal allele has between 6 and 54 CGG repeats). These women, and their family members, are at an increased risk to have children affected with fragile X syndrome. Molecular genetic testing is available on a clinical basis to determine the status of the FMR1 gene allele(s). Klinefelter syndrome affects males. It is diagnosed by an abnormal karyotype. Individuals with this syndrome have an extra X chromosome, with the karyotype being 47, XXY (Variations occur, but this karyotype is the most common). The extra X chromosome seems to affect the functioning of the testes and testosterone production. Adolescent boys with this disorder may undergo gynecomastia. Most are tall, but not particularly coordinated. The penis is of normal length; however the testes are small. Treatment with male sex hormones can be helpful. X-linked adrenoleukodystrophy is caused by a mutation in the ALD gene, located on the long arm of the X chromosome (Xq28). The childhood form most commonly presents between the ages of four and eight years. It begins with symptoms of attention deficit and hyperactivity disorder, but gets progressively worse, with symptoms including difficulty with previously mastered subjects such as speech and reading. The affected individual also becomes clumsy and has visual disturbances. Brain MRI is often abnormal, even while symptoms are still mild. The rate of progression of the disorder varies, but leads to death in a matter of years. Along with the childhood form of the disease, there are several other types. Type two usually presents during middle age (possibly as early as in the twenties), and includes leg stiffness and weakness and sexual dysfunction. This, too, is progressive, usually over decades. Type three, which is found in approximately 10% of cases, is characterized by adrenal insufficiency. Presentation can be anywhere between two years of age to adulthood. X-linked mental retardation hypotonic facies syndrome is caused by a mutation in the XNP gene, also located on the X chromosome (Xq13.3). Affected individuals have a distinct phenotype that includes genital abnormalities, a common set of facial features, and severe developmental delays with mental retardation. All patients have a normal 46, XY karyotype; however their appearance at birth can range anywhere from a male with hypospadius to a normal appearing female. Common craniofacial features include a small head circumference, small triangular nose, tented upper lip, prominent lower lip and open mouth. Short stature is common. Developmentally, milestones are delayed to a marked degree. Hypotonia is commonly present. Interestingly, the mutated gene appears to down regulate expression of the alpha-globin gene, leading to a microcytic and hypochromic anemia in some affected individuals. Individuals with XYY syndrome have an abnormal karyotype - 47, XYY. Physical development is normal, although these individuals have an increased likelihood for learning problems and tend to be tall.

A 17-year-old female athlete presents with easy fatigability and weakness. She has been training daily, and she has no significant medical history. Her pulse is 55/min, regular, BP 120/80 mmHg. There are no remarkable findings in a physical examination. Lab reports show the following: Serum Na+ 140 mEq/L Cl- 86 mEq/L K+ 2.3 mEq/L HCO3- 34 mEq/L pH 7.50 Urine Na+ 82 mEq/24 h K+ 168 mEq/24 h What is the most likely cause of her condition? Answer Choices 1 Bulimic vomiting 2 Hyperventilation induced by anxiety 3 Misuse of diuretics 4 Somatoform disorder 5 Use of anabolic steroids

Misuse of diuretics The patient has hypokalemia and alkalosis. Even though her serum potassium is low, she is still losing potassium in the urine. This is consistent with the action of a diuretic drug. Surreptitious use of diuretics by athletes is a common occurrence, as they use it to reduce weight. Abuse of diuretics is also a form of bulimia nervosa. Bulimic vomiting can also cause a hypokalemic alkalosis, but in that case urinary excretion of potassium would not be elevated. Additional sign of induced vomiting, like eroded dental enamel, would be evident on physical examination. Hyperventilation would lead to a respiratory alkalosis with a compensatory mild metabolic acidosis. In case of a somatoform disorder, there would not be any abnormal lab values. Anabolic steroids are unlikely to produce a picture of electrolyte imbalance.

A 44-year-old diabetic woman presents with a 2-week history of lower extremity edema. She has no other symptoms. Physical examination of her lower extremities reveals bilateral edema with 2+ pitting. Urinalysis: Appearance Pale yellow and cloudy Glucose Negative Bilirubin Negative Ketones Negative Specific gravity 1.025 Blood Negative pH 7.0 Protein 4+ Urobilinogen Normal Nitrite Negative Leuk. esterase Negative Microscopic examination: RBCs 0 - 2 cells/hpf WBCs 0 - 2 cells/hpf Few squamous epithelial cells noted Moderate oval fat bodies noted Question What is the most likely diagnosis? Answer Choices 1 Cirrhosis 2 Cystitis 3 Pyelonephritis 4 Nephrotic syndrome 5 Nephritic syndrome

Nephrotic syndrome Explanation The clinical picture is suggestive of nephrotic syndrome. Peripheral edema is the hallmark of nephrotic syndrome; this occurs when serum albumin levels drop to below 3 g/dL. Other findings include positive urinary protein (large amounts); oval fat bodies may be seen on microscopic urine exam. Clinical findings associated with cirrhosis include symptoms of weakness, easily fatigability, disturbed sleep, muscle cramps, and weight loss. In 70% of cases the liver is palpable. Jaundice is not an early sign, but rather a later symptom occurring when the disease progresses. Urine findings include darkened urine color, positive bilirubin and variable urobilinogen, and negative protein. These findings are not present in this patient. Clinical findings in cystitis include irritative voiding symptoms, being afebrile, and positive urine cultures. On urinalysis, leukocyte esterase is usually present with positive or negative nitrites and a cloudy appearance; it may have a noxious smell. Microscopic exam may reveal WBCs and bacteria. These symptoms are not present in this patient. Findings with pyelonephritis include fever, flank pain, and symptoms similar to cystitis. These findings are not present in this patient. Nephritic syndrome can present with proteinuria, hematuria, azotemia, RBC casts, oliguria, and hypertension. These findings are not present in this patient.

A 28-year-old man with no significant past medical history is rushed to the local trauma center following a stab wound to his chest. Paramedics report that there was significant blood loss. The patient has lost consciousness, is oliguric, and extremities are cool and moist to touch. His physical exam is also remarkable for tachycardia, tachypnea, a depressed systolic pressure, an immeasurable diastolic blood pressure. Question What is the preferred initial pharmacologic agent of choice for this patient? Answer Choices 1 Epinephrine 2 Hypertonic saline and dextran 3 Normal saline 4 Somatostatin 5 Vasopressin

Normal Saline Explanation This patient is presenting with signs and symptoms consistent with hypovolemic shock due to acute traumatic blood loss. 3 goals exist in the emergency department treatment of the patient with hypovolemic shock as follows: (1) maximize oxygen delivery - completed by ensuring adequacy of ventilation, increasing oxygen saturation of the blood, and restoring blood flow, (2) control further blood loss, and (3) fluid resuscitation. Current recommendations are for aggressive fluid resuscitation with lactated Ringer solution or normal saline in all patients with signs and symptoms of shock, regardless of underlying cause. If a patient is moribund and markedly hypotensive (class IV shock), both crystalloid and type O blood should be started initially. These guidelines for crystalloid and blood infusion are not rules; therapy should be based on the condition of the patient. Epinephrine is indicated as initial resuscitation management in cardiac arrest, anaphylactic shock, symptomatic bradycardia, and hypotension refractory to volume replacement. Epinephrine is associated with a host of adverse effects such as induction of pulmonary hypertension, tachyarrhythmia, myocardial ischemia, lactic acidosis, hyperglycemia and compromise hepatosplanchnic perfusion, oxygen exchange, and lactate clearance. The combination of hypertonic saline and dextran also has been studied because of previous evidence that it may improve cardiac contractility and circulation. Studies in the US and Japan have failed to show any difference when this combination was compared with isotonic sodium chloride solution or lactated Ringer solution. Somatostatin and octreotide infusions have been shown to reduce gastrointestinal bleeding from varices and peptic ulcer disease. These agents possess the advantages of vasopressin without the significant side effects. In the patient with GI bleeding, intravenous vasopressin and H2 blockers have been used. Vasopressin commonly is associated with adverse reactions, such as hypertension, arrhythmias, gangrene, and myocardial or splanchnic ischemia. Therefore, it should be considered secondary to more definitive measures. H2 blockers are relatively safe but have no proven benefit.

A 35-year-old woman just found out she is pregnant. She is experiencing polyuria, but she denies dysuria and incontinence. Her urinalysis is unremarkable. Her fetal ultrasound is normal, and her renal ultrasound shows normal physiological hydronephrosis of pregnancy. Her pre-pregnancy weight was 155 lbs, and she is 5 feet tall. Her calculated body mass index (BMI) is 30.3 kg/m2. She takes no medications. She smokes ½ pack of cigarettes/day. Question Assuming that the patient has a normal vaginal delivery and no episiotomy, how can you best prevent urinary incontinence post partum? Answer Choices 1 Normalize weight 2 Avoid episiotomy 3 Vaginal childbirth 4 Prescribe Duloxetine 5 Prescribe oxybutynin

Normalize weight Explanation This patient will benefit from lifestyle modification to normalize her body weight post-pregnancy. Dietary modifications, exercise, breastfeeding, and nutritional counseling may be helpful. She was overweight pre-pregnancy, with a BMI > 30 kg/m2. Obesity is a known risk factor for incontinence. It is also a risk factor for insulin resistance and diabetes, which may also contribute to various forms of incontinence. None of the other options listed below will help prevent incontinence; in fact, they may be risks for incontinence. Incontinence is common in pregnancy. Fetal compression of the bladder plus large volumes of urine due to suggested volume intake and increased glomerular filtration rates may contribute to this. Postpartum, vaginal birth, and changes in the laxity/strength of the pelvic floor may contribute to stress incontinence. Stress incontinence is characterized by the involuntary leaking of urine during stress or increases in abdominal and bladder pressure, such as coughing and sneezing. Bladder pressure at these times exceeds urethral pressure, allowing urine to leak through the urethra. Treatments for stress incontinence include pelvic floor exercises. By repeatedly contracting and relaxing the vagina and pelvic floor, leaking may decrease. In this patient's case, treating her cough with cough suppressants may additionally help with the urine leaks. In obese patients, 5 - 10% weight loss may also improve symptoms. Pessaries may be inserted into the vagina to increase urethral support. Urethral support can also be increased surgically by inserting a fascial sling or vaginal tape to support the urethra. Smoking cessation is laudable on many accounts. It may contribute to low birth weight in the baby and is risk factor for a variety of cardiovascular diseases in the mother. It is not currently considered a risk factor for incontinence. Episiotomy may be a risk factor for fecal incontinence, but it is not a known risk for urinary incontinence. Oxybutynin is an anticholinergic amine used in the treatment of neurogenic bladder and overactive bladder/urge incontinence. Reports of its use during pregnancy and lactation are not available (Micromedex). Duloxetine is used in the management of stress incontinence. It is a reuptake inhibitor of serotonin and noradrenaline. Its use is not suggested in pregnancy because it is category C and may have teratogenic effects.

A 22-year-old woman develops fever, rash, arthralgias, and decreasing urine output 2 weeks after completing a course of penicillin to treat streptococcal pharyngitis. Her physical exam is normal. She takes no other medications or supplements and has no other past history or symptoms. Her laboratory work results are as follows: Total white blood cell count 7.0x103/cmm Eosinophils elevated Serum creatinine 1.5 mg/dl Serum potassium 4.0 meq/l Serum bicarbonate 23 meq/l Fasting Blood glucose 80 mg/dl Urinalysis no casts, no bacterial growth, 2 red cells per high powered field, numerous white cells, trace protein Erythrocytes sedimentation rate elevated Question What diagnostic study will confirm her diagnosis? Answer Choices 1 Renal ultrasound 2 Renal biopsy 3 Renal gallium scan 4 Urine eosinophil count 5 24 hour urine for creatinine clearance

Renal biopsy Explanation This patient most likely has acute interstitial nephritis (AIN) secondary to her penicillin exposure. Of the listed choices, only renal biopsies yield information specific to acute interstitial nephritis. Renal biopsy is the criterion standard for diagnosing AIN. Lymphocytic and plasma cell infiltrates in the peritubular areas of the interstitium are noted. However, being an invasive procedure, it is not used in all patients, especially if the condition is mild or if the patient improves rapidly after removing the offending cause. Renal ultrasound may show slight increases in renal size and cortical echogenicity in AIN, but this may also occur with other renal conditions. A clinician may order this test to evaluate for other forms of injury (such as acute renal obstruction) in the evaluation of acute kidney injury. 24-hour urine samples are used to assess urine output, daily protein, electrolyte excretion, and creatinine clearance, a measure of renal filtering ability. Currently, calculations of creatinine clearance are made using the modification of diet in renal disease (MDRD) equation. Here, a 24-hour urine collection will tell us how well her kidneys are working, but it will not show the cause of their decline in function. Elevated Urinary eosinophils may be found in a variety of other diseases, including pyelonephritis and prostatitis. The positive predictive value of urine eosinophils for diagnosing AIN is low (Kodner). Gallium scans have limited predictive value for diagnosing AIN (Markowitz). Cortical necrosis (i.e., secondary to ischemia) unrelated to AIN and other diseases may cause similar patterns of uptake as AIN. If the patient was still on the medication suspected to cause the problem, it should be discontinued and never again used. This alone may cause resolution of her illness in a couple of weeks. Her symptoms, urine output, volume status, serum creatinine, and electrolytes should be monitored to evaluate for the need for dialysis. She can be started on prednisone therapy for 2 weeks, to be tapered thereafter.

A 63-year-old woman presents to you with a 5-year history of stage-3 chronic kidney disease. She states that she has not been very good about following her provider's orders, and wants to know what things she can do to help her condition. Question What is the appropriate dietary management for this patient? Answer Choices 1 Salt, water, and protein restriction, potassium supplementation, and magnesium restriction 2 Salt, water, and protein restriction, with phosphorus, potassium, and magnesium restriction 3 Salt and water restriction, with magnesium supplementation, and potassium restriction 4 Salt, water, and protein restriction, with phosphorus and magnesium supplementation 5 Salt, water, and protein restriction, with magnesium and phosphorus restriction

Salt, water, and protein restriction, with phosphorus, potassium, and magnesium restriction Explanation The correct answer is restriction of salt, water, protein, phosphorus, and magnesium, as well as potassium. Some studies have shown that protein restriction will slow the progression to end-stage renal disease. Overload of sodium and water can lead to congestive heart failure and edema. Phosphorus and magnesium restriction is needed, as hyperphosphatemia and hypermagnesemia can be seen in chronic renal failure; this is due to decreased excretion of phosphate and magnesium. The other answers are not correct, as potassium supplementation could cause a hyperkalemic state: it should be avoided in chronic renal failure unless otherwise indicated. Phosphorus and magnesium should be restricted as indicated above.

A 65-year-old man presents with flank pain, blood in his urine, and an unexplained weight loss. His past medical history is significant for numerous infections, kidney stones, cigarette use, and alcohol use. On physical exam, there is a palpable abdominal mass, as well as a low-grade fever. Diagnostic tests determine that he has cancer. Question What puts this patient at risk for the development of his particular type of cancer? Answer Choices 1 Eschericia coli infection 2 Urolithiasis 3 Smoking 4 Interstitial nephritis 5 Schistosoma haematobium infection

Smoking Explanation This patient has renal cell carcinoma. Smoking is a risk factor for many neoplasms, including renal cell carcinoma. There is a classic triad associated with renal cell carcinoma: hematuria, flank pain, and a palpable abdominal mass; however, the classic triad is not seen in most patients. Other presenting symptoms include a fever and weight loss. Cells from the proximal convoluted tubule are the most common cells that renal carcinoma arises from. There is an increased incidence of renal cell carcinoma with von Hippel-Lindau disease. Escherichia coli infection, urolithiasis, and interstitial nephritis are not known risk factors. Schistosoma haematobium infection is associated with bladder tumors.

The patient is a 35-year-old woman who presents as a new patient with urinary frequency, urgency, dysuria, and suprapubic discomfort for several months. Repeated urinalysis and clean catch urine cultures ordered by her primary care physician have been unremarkable. The urologist does not find any significant physical exam findings and decides to perform a cystoscopy under IV sedation. Findings include velvety red patches known as Hunner's ulcers, and a bladder biopsy is negative for cancer. Passive hydrodistention of the bladder is performed at the time of the cystoscopy and is found to provide the patient with minimal relief from her symptoms following the procedure. Question What medication would be an appropriate next step in this patient's treatment? Answer Choices 1 Ciprofloxacin (Cipro) 500mg BID x 7 days 2 Sodium Pentosanpolysulfate (Elmiron) 100mg TID 3 Bisacodyl (Dulcolax) 5mg once daily 4 Hydrocodone (Vicodin) 5/500 q 4-6 hours prn 5 Acetaminophen/Aspirin/Caffeine (Excedrin) 1-2 tablets daily

Sodium Pentosanpolysulfate (Elmiron) 100mg TID Explanation The scenario is describing a patient with interstitial cystitis (IC). Patients with IC have a 10:1 female to male ratio and are typically in the third decade of life. Symptoms usually include urinary frequency, nocturia, urgency, and bladder or pelvic pain. Physical examination is usually unremarkable and helpful at ruling out other causes of the patient's symptoms. The urinalysis and urine culture are usually unremarkable, which also rules out other differential diagnoses. Cystoscopy with hydrodistention under sedation is often used to diagnose IC by both the appearance of the bladder and the bladder capacity (not usually over 350cc). Hunner's ulcers seen during cystoscopy with hydrodistention are pathognomonic for interstitial cystitis, although they do not have to be present for a patient to have this diagnosis (only present in 5-10% of cases). The hydrodistention can also help to relieve symptoms, and can be an effective treatment for many patients with IC. However, if symptoms persist, then other treatment options are warranted. Altering diet and avoiding foods and beverages that are bladder irritants can be helpful in improving symptoms in patients with IC. Beyond these measures, there are various medications that can offer relief. Elmiron stands alone in its class of medications, but is similar to a class of medications called low molecular weight heparins. It prevents the irritation of the bladder wall that is the cause behind the patient's symptoms. This medication is prescribed 100mg TID and is a first-line treatment. It is the best choice of those listed as potential answers. Ciprofloxacin (Cipro) is an antibiotic commonly used to treat urinary tract infections (UTI). While UTI would have been high on the list of differential diagnoses for this patient, it was ruled out by the negative urinalysis and urine culture. Bisacodyl (Dulcolax) is a medication commonly used to treat constipation and would therefore not be an appropriate treatment for this patient. Hydrocodone (Vicodin) and acetaminophen/aspirin/caffeine (Excedrin) are both commonly used to treat pain. Hydrocodone is often prescribed to patients with IC, as chronic opioid use is not uncommon due to the occasional extreme nature of the pelvic pain. However, it would not be the next best treatment and is essentially masking symptoms and not treating the IC. Excedrin is a pain reliever, but it contains caffeine. Caffeine is a bladder irritant and should be avoided by patients with IC, as it can potentiate the symptoms.

A 65-year-old Caucasian man presents for a routine physical. He states that he is concerned about the development of prostate cancer. His history is significant for benign prostatic hyperplasia (BPH), for which he underwent a transurethral resection of the prostate (TURP) 3 years ago. His social history is significant for a 50 pack-year history of smoking, and he worked for 40 years as a coal miner. His father died of prostate cancer at age 76. What factor would most likely contribute to his risk of developing prostate cancer? Answer Choices 1 Race 2 History of BPH 3 Smoking history 4 Family history 5 Occupational history

The correct response is family history. Several risk factors have been identified for prostate cancer, including race and nationality. African-Americans are twice as likely to develop prostate cancer as are Caucasian Americans, and the incidence is higher in North America and western Europe than in other areas of the world. Age is the single largest risk factor, with rates of prostate cancer increasing rapidly after the age of 50. High fat diets and a sedentary lifestyle have also been linked to prostate cancer. Men with first-degree relatives with prostate cancer are twice as likely to develop prostate cancer as are other men, and the risk is even higher if multiple relatives are affected. Other factors, such as a history of smoking and occupational exposures, have not been conclusively linked to prostate cancer. Benign prostatic hypertrophy arises in cells in a different area of the prostate gland, and a history of BPH does not increase the risk of developing prostate cancer.

A 23-year-old woman presents with dysuria and left flank pain. She is 27 weeks pregnant. Physical examination reveals a temperature of 38°C (100.4°F); there is tenderness with percussion over the left costovertebral angle. Urinalysis shows 15 to 20 WBC's/hpf and 15 to 20 bacteria/hpf. What is the most appropriate management plan? Answer Choices 1 Schedule renal ultrasound 2 Order urine culture and arrange follow-up in 24 hours 3 Obtain IV pyelogram 4 Treat with oral trimethoprim/sulfamethoxazole 5 Admit for IV antibiotics

The patient should be admitted for IV antibiotics. Urinary tract infections in pregnant patients are associated with significant risks greater than in non-pregnant patients. Mechanical pressure on the ureters and bladder by the enlarging uterus, progesterone-mediated relaxation of smooth muscle, incomplete bladder emptying, and increased urinary tract volume contribute to the increased incidence of urinary tract infections during pregnancy as well as the increased severity of associated complications. Approximately 5% to 7% of pregnant women have asymptomatic bacteriuria. Untreated bacteriuria is associated with a high incidence of prematurity and fetal wastage as well as a 20% to 40% incidence of acute maternal pyelonephritis. Patients with uncomplicated bacteriuria should be treated with ampicillin, amoxicillin, nitrofurantoin, or a cephalosporin. However, pregnant patients with pyelonephritis (i.e., fever, chills, and flank tenderness) are at increased risk for sepsis and preterm labor, and they require hospitalization for aggressive therapy with parenteral antibiotics. Sulfonamide antimicrobial agents increase serum bilirubin levels, thereby increasing the risk of neonatal kernicterus; they should be avoided during the 3rd trimester.

A 62-year-old man is being evaluated in an internal medicine clinic for persistent hematuria. He denies dysuria, urgency, and frequency. The isolated hematuria was first discovered 4 months ago on a routine urinalysis for a workplace physical. A repeat urinalysis 6 weeks later again showed hematuria, confirmed by microscopic evaluation. The remainder of his urinalysis was within normal limits. His past medical history is remarkable for COPD and obesity. He has smoked 1.5 ppd x 45 years. He uses inhaled medications for his COPD and has NKDA. His physical exam was significant for a temperature of 100.5 degrees F. Abdominal exam elicited mild tenderness in the left upper quadrant, but no masses were palpable (although his exam was limited by his obesity). Chest and abdominal CTs with contrast were ordered. The report showed a normal chest and a left renal mass enhanced by radiocontrast, suggestive of renal cell carcinoma (RCC). No metastatic disease was noted on imaging. A renal biopsy is pending. Question Assuming the biopsy supports the diagnosis of renal cell carcinoma, what referral would be most appropriate for this patient? Answer Choices 1 Medical oncologist for traditional cytotoxic chemotherapy 2 Radiation oncologist for external beam radiation therapy 3 Urologic surgeon for radical nephrectomy 4 Urologist for retrograde pyelogram 5 Hospice for palliative care only

Urologic surgeon for radical nephrectomy Explanation The most appropriate intervention for this patient is referral to a urologic surgeon for radical nephrectomy, which is the "standard treatment for localized RCC and provides a reasonable chance for cure."1 The nephrectomy serves to diagnose, stage, and treat the cancer. "Surgical removal of the kidney is the most accurate method to definitively diagnose and to stage RCC."2 A referral to a medical oncologist for traditional cytotoxic chemotherapy is not recommended, as "none of the cytotoxic chemotherapy agents have been shown to improve survival in renal cell carcinoma."2 Immunotherapy (interleukon or interferon) is considered a reasonable addition to nephrectomy in patients with metastatic RCC, but there is no current evidence suggesting this patient has metastatic disease. RCC is considered a radiation-resistant tumor, so external beam radiation is rarely used as a primary treatment. Radiation may have a role in palliative treatment in special circumstances. Referral to a urologist for a retrograde pyelogram would not address the RCC. This type of imaging allows visualization of the bladder, ureters, and pelvicaliceal collecting system by administering contrast through a catheter to flow up toward the kidneys. This type of imaging may be helpful in the diagnosis of urethral strictures, trauma, and reflux, but adds no additional information and certainly no treatment value for this patient with RCC. A referral to hospice for palliative care only suggests there are no reasonable treatments for this patient and that he is expected to die within 6 months. However, 5-year survival rates are around 66% for stage I RCC, and this patient should be offered the referral for treatment.

A 65-year-old woman presents with a complaint of blood in her urine, intermittently for the last month. The patient denies any fever, chills, flank pain, or dysuria. Social history is positive for tobacco use (45 pack years), but patient reports stopping her tobacco use last year. What is the most likely cause of her hematuria? A urinary tract infection (UTI) B bladder cancer C renal calculi D pyelonephritis

bladder cancer B Hematuria in women older than 60 years is consistent with a bladder malignancy. Bladder cancer causes episodic, gross hematuria that is usually painless. Cigarette smoking is a risk factor that also increases the incidence of bladder cancer. Painful hematuria associated with suprapubic discomfort or dysuria (or both) is more indicative of cystitis or calculi. Pyelonephritis is associated with chills, fever, and flank pain.

The most common cause of nephrotic syndrome in children is A post-streptococcal glomerulonephritis B minimal change disease C diabetes mellitus D NSAIDs E polycystic kidney disease

minimal change disease B The most common cause of nephrotic syndrome in children is minimal change disease. Diffuse injury to the capillaries is the underlying cause, resulting in significant proteinuria, edema, hypoalbuminemia, and hyperlipidemia. It accounts for 65% of cases of nephrotic syndrome in children; however, 10% of adults with nephrotic syndrome have minimal change disease. Treatment is with corticosteroids for 2 to 4 weeks, dietary sodium restriction, and sometimes diuretics to reduce the edema. Relapse and lack of response to corticosteroids can occur. If the latter occurs, renal biopsy is indicated to rule out other causes of the nephrotic syndrome, such as focal glomerulosclerosis and membranoproliferative glomerulonephritis.

A 2-year-old boy presents with a firm, painless mass in his right testicle. It is determined that he has an endodermal sinus tumor. What tumor marker is most likely to be elevated? Answer Choices 1 Alpha-fetoprotein 2 5 hydroxytryptamine 3 Gastrin 4 Vasoactive intestinal peptide (VIP) 5 Acid phosphatase

AFP Explanation An endodermal sinus tumor is also called a yolk sac tumor, infantile embryonal carcinoma, embryonal adenocarcinoma of the prepubertal testis, or orchioblastoma. The presence of alpha-fetoprotein is very typical. Alpha-fetoprotein is seen with a variety of tumors, such as hepatocellular carcinoma, pancreatic carcinoma, testicular tumors, and others. 5-hydroxytryptamine is serotonin. Serotonin is an indolamine. The precursor for serotonin is tryptophan. Serotonin is a neurotransmitter. Serotonin is also the precursor to melatonin. As a tumor marker, 5-hydroxytryptamine (serotonin) is the major product seen with carcinoid tumors. Gastrin is a hormone that is ordinarily secreted by cells within the stomach. Specifically, gastrin is secreted by G cells, which are in the stomach antrum. Gastrin stimulates acid secretion of the stomach. Pathologically, gastrin can be produced by pancreatic islet cells tumors. Vasoactive intestinal peptide is sometimes abbreviated as VIP. Vasoactive intestinal peptide can be seen with islet cell tumors and pheochromocytoma. The prostate has the enzyme acid phosphatase. Acid phosphatase is actually a group of enzymes, which can be found in a few other tissues as well. With the development of prostate cancer, elevated serum acid phosphatase can be seen.

A 62-year-old woman is being treated for chronic congestive heart failure. She has been put on hydrochlorothiazide therapy. Her serum electrolyte levels are being monitored and show a persistent hypokalemia. The addition of what to her therapeutic regimen would be most appropriate? Answer Choices 1 Acetazolamide 2 Amiloride 3 Furosemide 4 Indapamide 5 Mannitol

Amiloride Explanation Hydrochlorothiazide is a thiazide diuretic often used in congestive heart failure. It causes increased excretion of sodium and chloride and loss of potassium. Thus, hypokalemia is an important side effect. Amiloride is a potassium-sparing diuretic. Its diuretic effect is not very potent, and hence it is good to use in combination with other diuretics. Acetazolamide is a carbonic anhydrase inhibitor. It causes a mild diuresis, a marked elevation of urinary pH, and a significant loss of potassium. Furosemide is a loop diuretic. It has a rapid onset of action and is a potent diuretic. However, it also causes potassium depletion and would only worsen the hypokalemia. Indapamide is a thiazide analog with a long duration of action. Mannitol is an osmotic diuretic and would not be recommended in this patient.

A 62-year-old male presents with complaints of numbness in his hands and feet, with occasional foot drop, memory disturbance, fatigue, paleness, anorexia, nausea, and weight loss. He has a known history of diabetes and hypertension. Which of the following conditions is most likely responsible for these symptoms?

Chronic Renal Failure Explanation: Chronic renal disease is associated with functional disturbances in all organ systems, including the central nervous system. Renal disease promotes CNS complications including neuropathies and neuromuscular irritability, along with systemic symptoms. The symptoms are typically progressive if the underlying renal disease is not addressed. Although other conditions promote similar neuropathies, such as diabetes, they are differentiated by the level of involvement, progression, and associated symptoms. With Guillain Barre, an acute polyradiculoneuropathy would be expected to progress, and have associated weakness. Cerebrovascular accidents are not typically accompanied by generalized systemic symptoms, and a middle cerebral artery occlusion would be expected to have contralateral hemiparesis and hemisensory deficit.

A 68-year-old man presents with scrotal swelling; he has had the swelling for the past few months. It is not bothering him, but his wife wants him examined. His history is not significant for any other GU history or symptoms. A genitourinary examination reveals a right hydrocele. Question What would the examination findings describe? Answer Choices 1 Fluid collection in the scrotum that transilluminates 2 Solid lesion in the scrotum that does not transilluminate 3 Fluid collection in the scrotum that does not transilluminate 4 Dilated veins in the scrotum that transilluminate 5 Cystic mass in the scrotum that transilluminates

Explanation A hydrocele is a collection of fluid between layers of the tunica vaginalis that surrounds the testicle. The typical presentation of a hydrocele is a painless scrotal swelling that can worsen throughout the day. When a flashlight or penlight is held behind the scrotum in a dark room, the light will transmit through the fluid; this is known as transillumination. A hydrocele is not usually bothersome, and patients will usually only complain if it is large enough to get in the way. Hydroceles can be treated surgically, but this is usually reserved for the most bothersome cases. A solid lesion in the scrotum that does not transilluminate describes how a testicular mass may present on physical examination. A hydrocele is not solid and does transilluminate. While a hydrocele is a fluid collection in the scrotum, it usually does transilluminate. There are no genitourinary findings that would involve a fluid collection in the scrotum that would not transilluminate. Dilated veins in the scrotum that transilluminate typically represent a varicocele rather than a hydrocele. The dilated veins are often described as feeling like a 'sack of worms' on physical examination. A cystic mass in the scrotum that transilluminates typically represents a spermatocele. A spermatocele is usually freely movable and located both superior and posterior to the testicle.

Which condition is suggested by urethritis, arthritis, and conjunctivitis? A chlamydial infection B gonococcal infection C reactive arthritis D tertiary syphilis

Reactive arthritis Both chlamydia and gonorrhea infections can result in urethritis. Gonococci can disseminate to the joints and cause septic arthritis. Chlamydia is typically asymptomatic but can cause chronic conjunctivitis in adolescents and young adults. Reactive arthritis (also known as Reiter syndrome) is a result of an untreated chlamydia infection, and although typically characterized, in texts, by the triad of urethritis, arthritis, and conjunctivitis, all of the symptoms may not be present or not identified at the time of presentation. Tertiary syphilis is characterized by neurologic and cardiovascular disease, gumma, auditory and ophthalmic involvement, and cutaneous lesions.

A mother brings her 2-year-old boy for the evaluation of frequent febrile urinary tract infections (UTIs); the infections have been present since the birth. He has just finished a 10-day prescribed trimethoprim/sulfamethoxazole (Bactrim) course, and he now has no problems. The mother thinks that main reason for the frequent UTIs is because a boy is neither interested nor willing to use the toilet; he is almost always wet. Physical examination today is unremarkable. He is not circumcised. You ordered dipstick urine analysis and perform ultrasound that came back normal. Question What will be your next step? Answer Choices 1 Plan voiding ureterocystogram 2 Order antibiotics prophylaxis 3 Arrange circumcision 4 Address dysfunctional voiding 5 Perform radionucleotide cystography

Plan voiding ureterocystogram Explanation Urinary tract infections (UTIs) are common in children and may cause permanent kidney damage. Urinary tract anomalies are risk factors for UTIs, and you should search for them in a boy of this age. While voiding cystourethrography (VCUG) is not recommended routinely after the first UTI, it should be performed if there is a recurrence, particularly a recurrence of febrile UTIs. Kidneys and bladder ultrasound are performed in younger children (3 - 5 years of age) as the initial step to evaluate anatomy, but they cannot relieve vesicoureteral reflux (VUR). Recommendations regarding antimicrobial prophylaxis still lack evidence, both for and against regardless, in this case, your approach should be based on the diagnosis of the presence of eventual urinary tract anomalies. You should tell the patient's mother that circumcision reduces UTIs, especially in high-risk boys; however, it is more important to exclude the presence of anatomical abnormalities, which put this boy at an even bigger risk. Voiding dysfunction is defined as daytime voiding disorders in children who do not have neurologic, anatomic, obstructive, or infectious abnormalities of the urinary tract. It is recommended that toilet training begin when a child is 18 months old and shows the interest. A child's interest usually appears around 24 - 25 months. Daytime dryness is usually achieved by 3 years of age. Dysfunctional voiding can lead to VUR, accidental urinary leakage, and UTIs; however, it is too early to think about the presence of dysfunctional voiding in this child. Both radionuclide cystography and voiding cystourethrography are used in detecting and grading vesicoureteral reflux. While VCUG is suggested for both girls and boys, radionuclide cystography is suggested only for girls because voiding cystourethrography is needed for adequate anatomic imaging of the urethra and bladder in boys.

A 63-year-old Caucasian woman 3 weeks status post total thyroidectomy presents with "twitching". The patient has noted significant bilateral muscle cramps as well as not being able to feel objects in her hands appropriately. She is also experiencing muscle spasms. Physical examination reveals a blood pressure of 90/50 mm Hg in both the left and right arms (with the patient sitting); there is a positive Chvostek sign, and there is also a positive Trousseau sign. ECG reveals a prolonged QT interval. Lab results are drawn, and they are as follows: Sodium 139 mEq/L (135 - 145 mEq/L) Potassium 4.0 mEq/L (3.5 - 5.0 mEq/L) Calcium 7.3 mg/dL (8.5 - 10.5 mg/dL) Magnesium 1.8 mEq/L (1.5 - 2.5 mEq/L) Question What is the most appropriate first line intervention for this patient? Answer Choices 1 Oral magnesium supplement 2 Intravenous sodium replacement 3 Intravenous calcium replacement 4 Oral calcium supplement 5 Oral potassium supplement

The correct response is intravenous calcium replacement. The patient described in the above scenario is having signs and symptoms consistent with hypocalcemia. This electrolyte deficiency is most likely due to the fact that she had a total thyroidectomy 3 weeks ago; this has led to what is termed acquired hypoparathyroidism. The parathyroid glands are primarily responsible for secreting parathyroid hormone (PTH). PTH's primary actions include increasing osteoclastic activity, renal tubular reabsorption of calcium, stimulation of the synthesis of 1,25-dihydroxycholecaldiferal by the kidneys. PTH also inhibits the absorption of phosphate and bicarbonate by the renal tubules; all of these actions lead to the increase of serum calcium. Removal of the thyroid and inadvertently the removal of the parathyroid glands (and PTH) has led to hypocalcemia. Acquired hypoparathyroidism is most commonly seen following surgical thyroidectomies and could be a transient or permanent condition. Signs and symptoms of hypocalcemia typically are not seen until serum levels fall below 7.5 mg/dL or lower. Patients may experience paresthesias, hyperreflexia, tetany, muscle spasms, muscle cramps, or even seizures. Physical examination may reveal a positive Chvostek sign (facial spasms following percussion on the facial nerve) and a positive Trousseau sign (carpopedal spasms of the hand by inflammation of a sphygmomanometer above systolic BP for several minutes.) Refractory hypotension may also be found. Severe cases of hypocalcemia may also lead to prolonged QT intervals on ECG, which potentially can lead to torsades de pointes. Oral supplementation of calcium is utilized in patients who are asymptomatic; however, our patient is displaying significant signs and symptoms. For this reason, the answer choice of intravenous calcium replacement is suitable at this point in time. Oral magnesium, oral potassium, and intravenous sodium supplementation are all inappropriate treatments at this time; these electrolytes are still in acceptable ranges.

A 62-year-old woman presents with bright red gross hematuria for the past 2 months. She states it is painless, but persistent. She denies other symptoms. She has no chronic medical problems. Upon further questioning, she admits to a 50-pack year smoking history, and she states she is currently retired from her job in a rubber factory. Vital signs are within normal limits, and physical exam is normal. Urine dipstick only shows too numerous to count RBCs, and urine cultures are negative. Question What is the most likely diagnosis? Answer Choices 1 Acute cystitis 2 Bladder cancer 3 Renal cell carcinoma 4 Urethritis 5 Ureteral calculi

bladder cancer Hematuria is the most common presenting sign of urinary tract cancer, and bright red gross hematuria is usually of lower urinary tract origin. Silent or painless hematuria suggests tumor or renal parenchymal disease; therefore, the clinical picture points to cancer, with bladder cancer as the most likely diagnosis. Smoking and exposure to industrial dyes or solvents (like in a rubber plant) are risk factors for bladder cancer. Bladder cancer is the 2nd most common urologic cancer, and the mean age at diagnosis is 65 years. It is more common in men than women (2.7:1), and 98% of primary bladder cancers are epithelial malignancies (majority urothelial cell carcinomas). Ordering cytology of the urine sample is often helpful with higher grade and stage bladder cancers. Patients can become anemic with chronic blood loss, so a CBC is justified. Diagnosis is made by cystoscopy with biopsy. Acute cystitis typically presents with irritative voiding symptoms (frequency, urgency, dysuria) and suprapubic discomfort in addition to possible hematuria. Urinalysis will show pyuria, bacteriuria, and varying degrees of hematuria. Urine cultures will show specific organisms. Renal cell carcinoma can present with flank pain, hematuria, persistent back pain, and an abdominal mass; also, it can be found incidentally on CT scan. RCC is more common in men than women (2:1), and it has a peak incidence in the 6th decade of life. This could be a possible option for diagnosis, but the significant history of smoking and previous work history points more toward bladder cancer. Urethritis is inflammation of the urethra that presents with urethral discharge, dysuria, and itching. Urethritis is most often caused by a STD. This patient does not have a history of unprotected sexual intercourse or any other symptoms that would indicate this diagnosis. Ureteral calculi can present with hematuria, but it also typically presents with flank or abdominal pain as well. If the stone is in the ureter, it often causes some hydroureter with or without hydronephrosis, both of which cause some pain or discomfort. The patient has no past history of forming stones.

At a yearly history and physical examination of a male patient, digital rectal exam reveals an enlarged prostate. You would more likely suspect benign prostatic hypertrophy, rather than prostate cancer, if the patient's history also included what presentation? Answer Choices 1 68-year-old with a free prostate specific antigen of 5% 2 48-year-old African American who is asymptomatic 3 52-year-old with a prostate specific antigen of 10 ng/mL 4 62-year-old with a brother treated for prostate cancer 5 72-year-old Caucasian who complains of a poor urinary stream

72-year-old Caucasian who complains of a poor urinary stream Explanation The male patient most likely to have benign prostatic hypertrophy (BPH) is the 72-year-old Caucasian who complains of a poor urinary stream. BPH occurs in 90% of men >70 years old. It occurs in the periurethral zone of the prostate and usually presents with lower urinary symptoms (LUTS) that suggest obstruction (i.e. hesitancy, weak stream, intermittent stream, straining, incomplete emptying, post-void leaking) or irritation (i.e. nocturia, frequency, urgency). Risk factors for prostate cancer include men >50 years old or African American men >45 years old or a first-degree relative with prostate cancer. Prostate cancer most often develops in the peripheral zone of the prostate. Early prostate cancer is usually asymptomatic, but locally advanced prostate cancer may encroach on the central transition zone of the prostate and present with irritative urinary symptoms (i.e. nocturia, frequency, urgency). Laboratory findings suggestive of prostate cancer/BPH include: An elevated prostate specific antigen (PSA) >4 ng/mL: sensitivity of this value for prostate cancer is 57-79% In BPH, level of PSA is generally below 10 ng/dl, A rise in PSA >0.75 ng/mL per year would suggest prostate cancer

The effect of steroid therapy is evaluated in an 8 year-old African-American boy being treated for fatigue and generalized edema following a "bad cold". His vitals are T 37 C, pulse 90/min, RR 20/min and BP 110/70. Physical exam reveals presence of mild periorbital edema and marked peripheral edema in hands and feet with the remainder of exam within normal limits. Lab values include dipstick urine protein 3+; urine protein 50 mg/m2/hr (<40 mg/m2/hr) Specific gravity 1030 (1008-1020) Urine protein/creatinine ratio 2.0/gm creatinine (< 0.2/gm creatinine) Serum albumin 3.9 (5.9-8.0 gm/dl) Cholesterol 250 (112-247 mg/dl) Remainder of laboratory values including BUN and plasma creatinine are within normal limits. Question What additional treatment should be initiated in this patient to decrease the risk of chronic kidney disease? Answer Choices 1 Diuretics 2 Spironolactone 3 ACE inhibitor 4 Beta blocker 5 Mixed alpha + beta antagonist

ACEI Explanation The pediatric patient described is apparently suffering from nephrotic syndrome. Glomerular disease induced proteinuria is the most common cause of nephrotic syndrome in children due to damage to the glomerular filtration barrier resulting in leakage of plasma proteins into the glomerular ultrafiltrate. Signs and symptoms in children include edema, urine protein: creatinine ratio > 0.2/gm creatinine; heavy proteinuria (urine protein >40 mg/m2/hr), hypoalbuminemia, and hyperlipidemia. The nephrotic range of proteinuria in children is higher than in adults (> 40 mg/m2/hr). Angiotensin-converting enzyme inhibitors and angiotensin receptor blockers not only lower blood pressure but have that additional benefit of slowing the progression of kidney disease even in patients with normal blood pressure. Although this child is normotensive and is already receiving steroid treatment, the next best additional treatment, therefore, is an ACE inhibitor or an angiotensin receptor blocker (ARB) to decrease the proteinuria and GFR decline in order to reduce the risk of chronic kidney disease. Diuretic is incorrect. Although a diuretic would provide symptomatic relief for the edema, it would not prevent the development of chronic kidney disease in this patient. Spironolactone is incorrect. Spironolactone is an inhibitor of renal aldosterone effects such as sodium and water retention and would help combat the edema in this patient but would not decrease the risk of chronic kidney disease. Beta blocker is incorrect. Beta blockers can be useful in the treatment of hypertension but this patient has not yet developed hypertension. Mixed alpha and beta antagonist is incorrect. The use of a mixed alpha and beta antagonist drug is not indicated in this normotensive patient and would likely not reduce the risk of chronic kidney disease.

A 51-year-old male patient presents to your family practice office complaining of genital discomfort with dysuria. His digital rectal exam reveals an enlarged, tender prostate. His prostate-specific antigen (PSA) returns elevated with a value of 11.1 mg/mL, which you fractionate, and this reveals approximately 75% free PSA. His urinalysis reveals moderate white cells and trace blood. What would be your next step in treating this patient? A Begin him on 6 weeks of doxycycline to treat his prostatitis and when resolved, repeat his PSA level. B Immediately refer him to a urologist for prostate biopsy to rule out prostate cancer. C Immediately refer him to a urologist for cystoscopy to rule out bladder cancer and perform a computed tomography (CT) scan of the abdomen and pelvis in the interim. D Order a stat testicular sonogram to rule out torsion. E Order a CT scan of the abdomen and pelvis.

Begin him on 6 weeks of doxycycline to treat his prostatitis and when resolved, repeat his PSA level. This patient has signs and symptoms consistent with prostatitis. Additionally, while his PSA is elevated, this is common in prostatits as well as prostate cancer, and his free PSA is of a percentage that prostate cancer is unlikely. However, it would be prudent to recheck his PSA after treatment and resolution of his symptoms to confirm that an underlying cancer is not smoldering.

A 61-year-old man with known cirrhosis presents with a 1-week history of "puffy" ankles and increased shortness of breath. A week prior to symptom onset, he had traveled on vacation, where he engaged in walking, sightseeing, and eating out. He admits more dyspnea with lying down and with increased exertion. His shoes feel snug, and he notes a definite line from wearing socks. The patient denies chest pain, leg pain, fevers, claudication, nausea, headache, lethargy, and hemoptysis. His past medical history is remarkable for cirrhosis and a history of alcoholism. He is awaiting a liver transplant. He had a liver biopsy, but no other surgeries. He takes no medications, has no allergies, and has been abstinent of alcohol for 9 months. He lives with his wife, works as an electrician, and smokes cigarettes at 1 pack-per-day. Vitals are normal, including oxygen saturation. On physical exam, the patient appears in no acute distress and with normal mental status. His physical exam is remarkable for mild jugular venous distention, 2-3+ edema in lower extremities, and mild dullness to lung percussion. No hepatomegaly or ascites is noted. This patient's laboratory results are shown in the chart. Complete blood count (CBC)- Normal Chest x-ray- Mild bilateral effusions Urinalysis- Normal Beta-natriuretic peptide (BNP)- Normal Complete Metabolic Panel (CMP)- Abnormal as indicated below: Test Result Normal range Units Sodium 126 134-144 mmol/L Potassium 3.4 3.4-4.9 mmol/L Chloride 100 100-109 mmol/L HCO3 26 20-31 mmol/L Glucose 98 70-99 mg/dL Bun 18 7-18 mg/dL Creatinine 1.1 0.6-1.2 mg/dL Calcium 8.8 8.8-10.5 mg/dL Albumin 2.6 3.5-5.0 g/dL Total Protein 5.9 6.4-8.2 g/dL AST (SGOT) 112 15-37 U/L ALT (SGPT) 108 5-43 U/L Alk Phosphatase 158 50-136 U/L Total Bilirubin 1.9 0.1-1.2 mg/dL eGFR >60 >60 mL/min/1.73m2 Osmolality 265 285-293 Mmol/kg H2O Question What is the most appropriate intervention for this patient's fluid and sodium status? Answer Choices 1 Administer intravenous hypertonic saline solution 2 Advise the patient to contact his gastroenterologist for an appointment 3 Begin intravenous or oral diuresis 4 Evaluate for possible pulmonary embolus 5 Initiate hemodialysis

Begin intravenous or oral diuresis This patient is in a hypervolemic hypotonic hyponatremic state. The hypervolemia can be identified primarily by history (edema and dyspnea) and physical exam (jugular venous distention, edema, and pulmonary effusion). The hypotonia (low serum osmolality) and hyponatremia most likely are due to a "dilution" effect, in which this patient's overall fluid status is high, but the sodium levels are normal or even elevated. This patient has hypoalbuminemia (common with chronic liver disease) and increased extracellular fluid, which is causing his mild pulmonary effusions and dyspnea. The most appropriate treatment for this patient is to begin intravenous or oral diuresis. Diuretics should be used with caution in patients with cirrhosis, but his pulmonary edema must be addressed. It would be potentially harmful to this patient to administer intravenous hypertonic saline solution, especially as his overall sodium level is likely normal to high. Hypertonic saline should only be used in acutely ill patients, with careful monitoring. Without prior records, it is difficult to assess any potential worsening in this patient's chronic cirrhosis. While it would be acceptable to advise the patient to contact his gastroenterologist for an appointment in addition to other interventions, it would not address his current hypervolemic hyponatremia or his pulmonary edema. With a complaint of shortness of breath, especially in a smoker, it could be rationalized to evaluate for a possible pulmonary embolus. However, he does not meet the Wells criteria for a PE (no suspected DVT, no tachycardia, no immobilization, no history of DVT/PE or malignancy, no hemoptysis, and alternative diagnoses are more likely than PE). Further evaluation for PE is not indicated. With severe electrolyte abnormalities, or those that do not respond to diuresis, it would be reasonable to initiate hemodialysis. This patient's abnormalities are not severe, and he is mildly symptomatic. A trial of diuresis is indicated before considering hemodialysis.

A 68-year old man presents due to urinary hesitancy. He describes difficulty initiating his urinary stream as well as reduced force, intermittent and slow flow. These symptoms have become progressively worse over the past 12 months. The patient is otherwise in good health and has no significant medical history. On exam, the abdomen is soft with no organomegaly and the external genitalia are unremarkable. Digital rectal examination reveals an enlarged, smooth prostate. To reduce his symptoms, the patient is given a prescription for finasteride. Question What explanation describes the correct mechanism of action of this agent? Answer Choices 1 Blockade of the enzymatic conversion of testosterone to dihydrotestosterone 2 Relaxation of prostatic smooth muscle cells 3 Inhibition of androgen synthesis 4 Blockade of prostatic DHT receptors 5 Blockade of the enzymatic conversion of testosterone to estrogen

Blockade of the enzymatic conversion of testosterone to dihydrotestosterone Explanation Benign prostatic hypertrophy (BPH) is a condition that arises as a result of proliferation of stromal cells in the periurethral glands of the prostate. Consequently, whereas cancer most commonly arises in the outer layer of the prostate, BPH is a condition associated with the inner portion or transition zone. BPH is a condition that accompanies aging. By age 60, approximately 50% of men have microscopic evidence of BPH and by age 85, about 85% of men will have BPH. The exact cause of BPH remains unclear; however, it is most likely the result of changes in the local hormonal milieu of the prostate. Dihydrotestosterone (DHT) is produced in the prostate via conversion of testosterone by the 5-alpha reductase enzyme. DHT is known to promote stromal proliferation. It is hypothesized that elevated estrogen levels that accompany aging increase the density of DHT receptors in the prostate, leading to hyperplasia. In addition, the density of alpha-1 adrenergic receptors, which contract smooth muscle cells in the prostate, also appears to increase with age, contributing to the obstructive urinary symptoms that typically bring patients with BPH to clinical attention. Microscopic prostatic hyperplasia in and of itself is not dangerous; there is no known association between BPH and prostate cancer. However, prostatic enlargement around the urethral canal can cause urinary outflow obstruction. Clinically, patients report urinary hesitancy, decreased force of urination, interrupted urinary stream, and a feeling of incomplete bladder emptying. Recurrent urinary tract infections may also occur because of residual urine pooling. In severe cases, hydronephrosis may result. In patients with BPH, digital rectal examination reveals lateral and/or median lobe prostatic enlargement. However, prostate size correlates poorly with obstructive symptoms and urodynamic studies are the most useful means of quantifying the severity of BPH. Peak flow rates under 10 ml/sec and residual urine volumes of greater than 150 ml are indicators of significant obstruction. There are 2 main categories of pharmacological agents used to treat BPH: alpha-1 adrenergic antagonists and 5-alpha reductase inhibitors. Alpha-1 adrenergic antagonists (e.g., terazosin, doxazosin, tamsulosin) relax prostatic smooth muscle by blocking the action of alpha-1 adrenergic agonists, reducing obstructive symptoms. By contrast, 5-alpha reductase inhibitors (e.g., finasteride) reduce DHT levels by blocking enzymatic conversion of testosterone. It should be noted that finasteride also lowers prostate specific antigen (PSA) levels, which may complicate the use of PSA as a screening test for prostate cancer.

A patient presents to your family practice office with classic renal colic with his pain being a 6 on a 1 to 10 scale, with 10 being the worst pain imaginable. You refer your patient for a stat computed tomography (CT) scan of the kidneys. Your radiologist calls and advises that the CT reveals that your patient has a 4 mm stone at the ureterovesical junction (UVJ) on the left side without evidence of hydronephrosis. What should you advise your patient? A Go directly to the emergency department for admission. B Go directly to the emergency department for hydration and pain management. C Return to the office, administer ketorolac IM, give a prescription for an opioid, increase fluids, strain their urine, go to the emergency department if the pain worsens, and return in 48 hours for a reevaluation. D Go home, take four Motrin IB tablets every eight hours for pain, and go to the emergency department if the pain worsens. E Refer the patient for immediate lithotripsy.

C Return to the office, administer ketorolac IM, give a prescription for an opioid, increase fluids, strain their urine, go to the emergency department if the pain worsens, and return in 48 hours for a reevaluation. C Nonsteroidal anti-inflammatory drugs (NSAIDs) in general, and ketoralac specifically more so than other, causes constriction of the renal afferent arteriole, reducing pressure on a kidney stone and providing significant pain relief. Generally speaking a stone of less than 5 mm, especially at the UVJ, will pass relatively rapidly (within 48 hours) and the patient only needs to strain their urine to collect the stone and receive analgesia.

A 35-year-old man presents with right flank pain that has progressively worsened over the past 12 hours and is now radiating into his right groin and testicle. He currently rates the pain as an 8/10 and complains of some nausea but no vomiting. He denies ever having this type of pain previously. He states that he thinks he had some blood in his urine at his last void, but he did not notice any prior to that. The patient is unable to sit still during the interview and refuses the portion of the physical exam where CVA tenderness is assessed; he also refuses any palpation of the abdomen or testicle on the right side. Vital signs include a temperature of 99.2, and BP is 156/84 mm Hg; RR is 12, and oxygen sat 99% on room air. Question What is the best study to assess this patient's condition? Answer Choices 1 CT scan of abdomen and pelvis without contrast 2 MRI of abdomen and pelvis 3 Ultrasound of kidneys 4 Kidney, ureters, bladder X-ray 5 Intravenous urogram

CT scan of abdomen and pelvis without contrast Explanation This patient most likely suffers from urolithiasis. These patients typically present with unilateral renal colic that often radiates to the ipsilateral groin or testis and hematuria. These patients generally cannot sit still secondary to the pain and discomfort. They can also exhibit guarding, nausea, and vomiting in some cases. A stone protocol (non-contrast) CT scan has become the standard initial workup of patients with suspected stone. This study is especially useful in the emergency department, since it can quickly and effectively diagnose urolithiasis. A CT scan gives the most information (location and size of stone, hydronephrosis, any anatomical variations, etc.) to the urologist who will likely see the patient. Stones are more common in men than women (3:1), and initial presentation is typically in the 3rd of 4th decade of life. MRIs are used more often in assessing soft tissues; they are not typically useful in urolithiasis cases. Ultrasound of the kidneys can reveal hydronephrosis and cystic or solid lesions. Stones can sometimes be assessed as well, but small stones are occasionally not visible. A kidney, ureter, bladder X-ray will show approximately 90% of stones. Uric acid stones are not visible on plain film, and some stones are difficult to see because of size or location. A KUB is a viable option, but not the best study to order. Intraveneous urogram is most useful after a thorough bowel preparation, so a non-emergent study will typically yield more information than an emergent study. However, if high-grade obstruction is suspected, emergent intervention with intraveneous urogram may be warranted. These patients can have an elevated temperature and a CT scan can show high-grade obstruction, so that is still the best initial study.

A 24-year-old man presents with gradual onset of scrotal pain. The pain is constant, and it lessens with elevation of the testicles. The patient developed a fever this morning. He has been voiding small amounts frequently, and he is also experiencing dysuria. The patient has a negative past medical history and past surgical history. He does not smoke, drink alcohol, or abuse illicit drugs. He is currently sexually active with a new partner. Examination reveals edema of the left testicle; there is tenderness to palpation. Cremasteric reflex is positive. Question What is the most appropriate intervention at this time? Answer Choices 1 Biopsy of the testicle 2 Testicular ultrasound 3 Insertion of foley catheter 4 Ciprofloxacin and doxycycline 5 Ceftriaxone and doxycycline

Ceftriaxone and doxycycline The patient is most likely suffering from acute epididymitis, so he should be treated with a single dose of IM ceftriaxone along with a 10-day course of oral doxycycline. Epididymitis is inflammation of the epididymis, the tube that connects the testicle to the vas deferens. Epididymitis is typically the result of infection. In younger, sexually active man, it is usually caused by Chlamydia trachomatis or Neisseria gonorrhoeae. In older man, epididymitis is associated with Gram-negative rods. Given the patient's age and sexual history, he should be treated with ceftriaxone, which will cover gonorrhea, as well as doxycycline, which is active against chlamydia. Biopsy of the testicle is an incorrect response. The patient has no identifiable mass on examination, only testicular edema. A testicular ultrasound is not indicated at this time. Insertion of a foley catheter is an incorrect response. Ciprofloxacin and doxycycline is an incorrect response. If the patient had been older and did not have sexual risk factors for sexually transmitted epididymitis, then ciprofloxacin might have been required to cover for Gram-negative rods. However, in this case, ceftriaxone and doxycycline will better cover the likely causative agents.

A 25-year-old sexually active man notices that he has burning and pain while urinating. He also notices some urethral discharge. He sees you in your office for a consultation, and you order several laboratory tests. One of the tests that you order is a Gram stain and culture on a sample of the discharge. The results are negative, and gonorrhea is ruled out. After reviewing all the lab results, you tell your patient that he has nongonococcal urethritis (NGU). What is the most likely cause of this patient's nongonococcal urethritis? Answer Choices 1 Chlamydia psittaci 2 Chlamydia trachomatis 3 Ureaplasma urealyticum 4 Trichomonas vaginalis 5 Pneumocystis jiroveci

Chlamydia trachomatis Chlamydia are bacteria. Chlamydia was once thought to be protozoa as well as viruses, but it is neither; they are bacteria in the order Chlamydiales. Chlamydia is obligate intracellular organisms. Characteristic of chlamydia infections is the development of inclusion bodies. Chlamydia trachomatis cause approximately 50% of all cases of nongonococcal urethritis in males. Chlamydia trachomatis causes more nongonococcal urethritis in boys/men than Trichomonas vaginalis does. Urethritis is an inflammation of the urethra. It is classified as either gonococcal urethritis (caused by Neisseria gonorrhoeae) or nongonococcal urethritis (caused by something other than Neisseria gonorrhoeae). Common causes of nongonococcal urethritis are Chlamydia trachomatis, Trichomonas vaginalis, and Ureaplasma urealyticum. Chlamydophila psittaci does not cause nongonococcal urethritis. Chlamydophila psittaci causes a systemic illness called psittacosis, parrot fever, or ornithosis. Psittacosis is acquired from birds. Trichomonas vaginalis is a protozoan. Trichomonas vaginalis can cause nongonococcal urethritisin males.It also causes trichomoniasis vaginitis, sometimes called trichomoniasis. Ureaplasma urealyticum is considered a mycoplasma. Ureaplasma urealyticum is in the family Mycoplasmataceae and the genus Ureaplasma. Bacteria in the genus Ureaplasma require urea; therefore, Ureaplasma urealyticum is found primarily in genitourinary tract. Ureaplasma urealyticum is a common cause of nongonococcal urethritis. Pneumocystis jiroveci (formerly called Pneumocystis carinii) is a fungus, not a protozoan. In an immunosuppressed host, Pneumocystis jiroveci can cause pneumonia. Pneumocystis jiroveci does not cause nongonococcal urethritis.

A 67-year-old man presents with a subacute onset of lower urinary tract symptoms. He is unable to discuss his past medical history or current medications. An initial genitourinary workup is started, and a microscopic urinalysis reveals granular and waxy casts. Question With what disease process are his results closely associated? Answer Choices 1 Chronic renal disease 2 High urinary protein nephrotic syndrome 3 Glomerulonephritis 4 Pyelonephritis 5 Acute tubular necrosis

Chronic renal disease Explanation The correct response is chronic renal disease. Casts are cylindrical structures, consisting of clumps or clusters of cells or material that can form in the renal distal and collecting tubules of the kidney. Casts form when the pH of the urine is acidic and when the urine is very concentrated. Casts dislodge from the kidney and can be seen in the urine. In order to see casts, urine must be visualized under low power on a microscope. There are various types of casts that can be characterized into acellular versus cellular casts; each category can be further characterized, and the various casts can be associated with various disease processes. Granular casts are the 2nd most common type of cast and result from the breakdown of cellular material. They are most often indicative of chronic renal disease, but can also be seen if a patient has just vigorously exercised. Waxy casts are also indicative of advanced renal disease, specifically indicating a more chronic issue. Fatty casts are the result of the breakdown of lipid-rich epithelial cells; they are pathognomonic for high urinary protein nephrotic syndrome. High urinary protein nephrotic syndrome does not lead to the formation of granular casts. Nephritic syndromes, urinary tract injury, glomerulonephritis, and vasculitis can all result in red blood cell casts. Whenever there are red blood cells within a cast, there is a strong indication for glomerular damage from a number of different disease processes. Glomerulonephritis does not lead to the formation of granular casts. If white blood cells are seen within a cast, this is an indication that there is an inflammation or infection of the kidney known as pyelonephritis. Various other inflammatory states can also result in white blood cell casts. Pyelonephritis does not lead to the formation of granular casts. Acute tubular necrosis, cytomegalovirus, hepatitis, and toxic ingestion can all result in epithelial cell casts. These casts are the result of desquamation of the renal tubule cells into the collecting system. Acute tubular necrosis does not lead to the formation of granular casts.

A 35-year-old woman presents with a 24-hour history of fever, right flank pain, vomiting, dysuria, and hematuria. A urinalysis reveals large amounts of red and white blood cells, as well as leukocyte esterase, and a subsequent urine culture was positive for >100,000 Escherichia coli. The patient denies having a history of renal calculi, and a bedside renal ultrasound does not show any stones or hydronephrosis bilaterally. She is diagnosed with a right-sided pyelonephritis and admitted for pain control, hydration, and IV antibiotic therapy. Her symptoms, including her fever, abate quickly, and she is discharged after 48 hours of being afebrile. Question Which prescription would she most likely receive upon discharge? Answer Choices 1 Motrin 800 mg PO q 8 hours prn pain 2 Nitrofurantoin 100 mg PO BID x 7 days 3 Ciprofloxacin 500 mg PO BID x 14 days 4 Levofloxacin 500 mg PO daily x 7 days 5 Amoxicillin 500 mg PO BID x 14 days

Explanation Ciprofloxacin 500 mg PO BID x 14 days is the correct answer. Patients with pyelonephritis who are sick enough to be treated as an inpatient receive IV antibiotics until they have been afebrile for 24 - 48 hours. They also need to be able to tolerate oral hydration and oral medications before being discharged. Upon discharge, they will be given a prescription for antibiotics that will complete at least 2 weeks of antibiotic treatment. Ciprofloxacin has good coverage for E. coli urinary tract infections, and given at 500 mg PO BID x 14 days would be an appropriate choice as long as sensitivities from her culture showed Ciprofloxacin to have sensitivity. Motrin 800 mg PO q 8 hours prn pain is not the correct answer. While motrin is a good choice for patients who may still have some discomfort related to the pyelonephritis, it is not the most likely prescription for this patient to receive. She had become asymptomatic prior to discharge, so there would not be any reason to prescribe anything to help with pain or discomfort at that time. In addition, she is more likely to receive treatment for her infection than for pain. Nitrofurantoin 100 mg PO BID x 7 days is not the correct answer for several reasons. While nitrofurantoin is often an appropriate antibiotic for E. coli urinary tract infections, it does not achieve tissue levels reliable enough for pyelonephritis treatment. In addition, only 7 days of treatment does not add up to at least 2 weeks total of antibiotics. Levofloxacin 500 mg PO daily x 7 days is not the correct answer. While levofloxacin is an appropriate antibiotic to treat E. coli pyelonephritis, 7 days of treatment is not long enough in this situation. Amoxicillin 500 mg PO BID x 14 days is not the correct answer. While 14 days of antibiotics is a good length of time, amoxicillin does not have good coverage for E. coli, so it would not be a good choice to treat an E. coli related pyelonephritis.

An 8-year-old boy is evaluated for persistent bed wetting. He has never been continent, averaging 2 - 3 episodes of bedwetting per week. His urological evaluation revealed a normal bladder and urethra, with no neurological problems. Lately, his problem has been a source of much embarrassment; he is unable to attend camp or sleepovers due to fear of wetting his bed. He has tried multiple interventions, including lifestyle changes, alarm systems, and reward systems. His physical exam shows no abnormalities. His parents are keen on a rapid resolution to his problems, and they insist treatment be initiated. Question What is the best therapy? Answer Choices 1 Continued lifestyle changes 2 Desmopressin 3 Imipramine 4 Continue alarm system 5 Corticosteroid

Explanation Desmopressin is the correct answer. Desmopressin is an analogue of anti-diuretic hormone, which reduces the production of urine at night. It is effective and has a rapid onset of action. The intra nasal form has been removed from the market in favor of the oral tablet. Desmopressin is indicated in enuresis not responding to lifestyle changes or alarms, when patients/parents want rapid improvement, or in children in whom alarm systems are ineffective. Continued lifestyle changes is incorrect. Lifestyle changes include fluid restriction after 5 pm, restriction of caffeinated drinks, and making the child urinate before bed. In this child, enuresis is pathological; it is persistent after the age of 5, and medical intervention is indicated due to the failure of lifestyle measures. Imipramine is incorrect. Imipramine is an effective drug, but is avoided in children due to its numerous side effects, such as dry mouth, constipation, and drowsiness. Alarm system is incorrect. Alarm systems are effective; however, they have already been tried in this child. Therefore, a different form of therapy is indicated. Coricosteroids is incorrect. They are not used in the management of enuresis.

A 14-year-old boy presents due to embarrassment after an incident in school: while undressing before the class in physical education, the other boys laughed at him because of his "underdevelopment". His personal and family history is non-contributing. Question If puberty has started, what would you expect to notice? Answer Choices 1 Pubertal growth spurt 2 Increase in size of testes 3 Increase in body mass 4 Increase in bone accretion 5 Elongation of penis

Explanation In boys, age of 13 and a half years is the upper limit of normal onset of puberty (to simplify, 14 years is often used as the upper limit). The initial sign is usually an increase in the size of the testes to more than 2.5 cm in their longest diameter, excluding the epididymis. Testicular size reaches adult size about 6 years after the onset of puberty. Increase in the speed of growth (pubertal growth spurt) is the initial sign of puberty in girls (although breast development is usually the first sign of puberty reported). In boys, a growth spurt usually follows the increase in size of testes. Changes in body composition (lean body mass, skeletal mass and body fat) usually follow changes in testicular size. They happen several years earlier in girls, who also have an earlier growth spurt and weight gain. Critical periods of bone accretion occur during infancy and puberty in both sexes. In puberty, peak of bone mineralization is registered after the peak height velocity. After testicles have enlarged and developed for about 1 year, the length of the penis increases; this is followed by an increase of the breadth of the shaft of the penis and the enlargement of the glans and corpora cavernosa.

A 2-month-old infant is being seen for a routine examination by his pediatrician. However, his mother admits to not following recommendations and has not had him seen by the pediatrician since hospital discharge. During the genitourinary examination, the pediatrician cannot palpate the testis on either side of the scrotum. The pediatrician is concerned that the infant has bilateral cryptorchidism (or undescended testes), but needs to make sure testes are present somewhere above the scrotum. Question What laboratory testing and imaging study combination would the pediatrician order? Answer Choices 1 Luteinizing hormone, follicle stimulating hormone, and testosterone levels followed by CT scan 2 Luteinizing hormone, follicle stimulating hormone, and testosterone levels followed by ultrasonography 3 Human chorionic gonadotropin stimulation test and testosterone levels followed by ultrasonography 4 Human chorionic gonadotropin stimulation test and testosterone levels followed by X-ray 5 Human chorionic gonadotropin stimulation test and testosterone levels followed by CT scan

Explanation Luteinizing hormone (LH), follicle stimulating hormone (FSH), and testosterone levels followed by ultrasonography is the correct answer. In male infants under the age of 3 months, LH, FSH, and testosterone levels are helpful in determining whether there are testes present. Ultrasonography has a sensitivity of 76%, a specificity of 100%, and an overall accuracy of 84% when diagnosing undescended testes that are nonpalpable on examination. MRI would have also been a good imaging choice, as it has a sensitivity of 86%, a specificity of 79%, and an overall accuracy of 85%. However, ultrasonography is both easier and a lower risk to perform on infants and children, making it the more popular choice in many cases. Luteinizing hormone, follicle stimulating hormone, and testosterone levels followed by CT scan is not the correct answer. CT scan findings in children when used in the diagnosis of nonpalpable undescended testes are historically not reliable. Therefore, the imaging study of CT scan makes this answer incorrect. Human chorionic gonadotropin (HCG) stimulation test and testosterone levels followed by ultrasonography is not the correct answer. The HCG stimulation test is done by administering 2000IU of HCG daily for 3 days and checking testosterone levels pre- and post-stimulation. This helps to determine the presence or absence of testicular tissue. However, this test is reserved for infants over 3 months of age. Ultrasonography is the correct imaging study, but the answer is incorrect due to the laboratory test being incorrect. Human chorionic gonadotropin stimulation test and testosterone levels followed by x-ray is not the correct answer. The HCG stimulation test is done by administering 2000IU of HCG daily for 3 days and checking testosterone levels pre- and post-stimulation. This helps to determine the presence or absence of testicular tissue. However, this test is reserved for infants over 3 months of age. X-ray is not used at all in the diagnosis of cryptorchidism. Both facets of this answer choice are incorrect. Human chorionic gonadotropin stimulation test and testosterone levels followed by CT scan is not the correct answer. The HCG stimulation test is done by administering 2000IU of HCG daily for 3 days and checking testosterone levels pre- and post-stimulation. This helps to determine the presence or absence of testicular tissue. However, this test is reserved for infants over 3 months of age. CT scan findings in children when used in the diagnosis of nonpalpable undescended testes are historically not reliable. Both facets of this answer choice are incorrect.

A 10-year-old boy is brought to the clinic by his mother. She noted that his face is swollen, and he told her that his urine was cloudy and reddish. He has a history of falling and abrading the skin of his right thigh 2 weeks ago. The next day, the skin became red, hot, and tender; the infection was treated with a topical antibiotic ointment. The cellulitis gradually healed. You suspect acute glomerulonephritis. What organism is the most likely cause of the disease? Answer Choices 1 Streptococcus pyogenes (group A beta-hemolytic) 2 Streptococcus agalactiae (group B) 3 Enterococcus faecalis 4 Peptostreptococcus 5 Streptococcus pneumoniae

Explanation Streptococcus pyogenes (group A beta-hemolytic) cause 3 types of diseases: 1) pyogenic diseases, such as pharyngitis and cellulitis, 2) toxigenic diseases, such as scarlet fever and toxic shock syndrome, and 3) immunologic diseases, such as rheumatic fever and acute glomerulonephritis. Glomerulonephritis occurs especially following skin infections. Streptococcus pneumoniae are Gram-positive lancet-shaped cocci arranged in pairs (diplococci) or short chains. On blood agar, they produce alpha-hemolysis. Virulence factors of Pneumococci are polysaccharide capsules. Pneumococci cause pneumonia, bacteremia, meningitis, and infections of the upper respiratory tract, such as otitis and sinusitis. Mortality rate is high in elderly, immunocompromised (especially splenectomized), and/or debilitated patients. They should be immunized with the polyvalent polysaccharide vaccine. Peptostreptococci grow under anaerobic or microaerophilic conditions and produce variable hemolysis. Peptostreptococci are members of the normal flora of the gut and female genital tract and participate in mixed anaerobic infections of the abdomen, pelvis, lungs, and brain. Streptococcus agalactiae (group B streptococcus) colonize the genital tract of some women and can cause neonatal meningitis and sepsis. They are usually bacitracin-resistant. Enterococcus faecalis (group D streptococcus), formerly known as Streptococcus faecalis, are part of the normal flora in the gut. They can cause urinary, biliary, and cardiovascular infections.

A 56-year-old African-American man presents with urinary hesitancy, frequency, and nocturia. He has to get up and urinate 3 to 4 times per night, and he is not sure if he empties his bladder completely. He states that his symptoms have been getting worse over the past 2 years. His urinary stream is weaker than it was a 1 year ago. He denies blood in his urine, and there is no history of urinary tract infections, dysuria, or pain. He is otherwise in a good state of health, and he has no significant past medical or surgical history. Currently, he takes no medications, and he has no known drug allergies. On review of systems, you discover that his father and brother died of prostate cancer in their 50's. The remainder of the history and ROS is non-contributory. Physical Examination: Vital signs are stable and he is afebrile. General physical exam is unremarkable. Genital exam reveals a circumcised penis with no lesions or discharge. There is no inguinal adenopathy. Testicles are descended bilaterally with no lesions, masses, or hernias. Rectal exam reveals a smooth prostate with no nodules or tenderness. Diagnostic investigations: Urinalysis is normal and Prostate-Specific Antigen (PSA) test is within normal range for age. (For men aged 50 to 59 years, the normal reference range is 0 to 3.5 ng/mL) After emptying 250 mL of urine, the post-void residual urine volume is 50 mL. Question What is the most appropriate intervention? Answer Choices 1 Finasteride 2 Surgical therapy 3 Terazosin 4 Transrectal ultrasound with prostate biopsy 5 Urine culture and sensitivity

Explanation Terazosin is the correct response. Given the history, physical exam, and negative PSA, you have enough information to make the diagnosis of symptomatic benign prostatic hyperplasia (BPH). No further diagnostic studies are necessary. Benign prostatic hyperplasia (BPH), also known as benign prostatic hypertrophy, is a histologic diagnosis characterized by proliferation of the cellular elements of the prostate. A classic history is usually sufficient to make the diagnosis. Clinical manifestations include urinary hesitancy, urinary frequency, urgency, nocturia (awakening at night to urinate), decreased or intermittent force of stream, and/or a sensation of incomplete bladder emptying. Treatment: Depending on the patient's preferences, the next step is to begin treatment; in most cases, medical therapy is initiated first. If the symptoms do not significantly interfere with the patient's life, he may choose to wait and refuse treatment once he is reassured that he does not have a life-threatening illness. This decision would be medically acceptable in this case. If he selects treatment, management begins with a selective α1-receptor blocker, such as doxazosin or terazosin. A medication specific for α 1A-receptor subtype, such as tamsulosin (Flomax®), may be used in patients who cannot tolerate traditional α1-receptor blockers. If medical therapy fails or if a patient has severe BPH with ongoing obstruction, retention of large volumes of urine, or recurrent urinary tract infections, surgical therapy should then be considered. The most commonly performed surgery is transurethral resection of the prostate. Because of his family history, the patient in this case has an increased risk of prostate cancer; however, transrectal ultrasound with prostate biopsy is not indicated. This diagnostic procedure should be reserved for suspicion of prostate cancer. Based on this patient's family history and because he is African-American (African-Americans have a 50% higher incidence of and mortality from prostate cancer in comparison with Caucasians) a healthy index of suspicion is astute nonetheless. Given this patient's classic BPH presentation and the normal PSA, prostate cancer is low probability at this time. That being said, caution must be exercise when using PSA as a diagnostic tool to rule in or rule out prostate cancer. The USPSTF recommends against PSA-based screening for prostate cancer. In addition, the negative prostate exam on rectal probing, while classically taught to be important, adds no additional information in most cases; currently, it is not recommended by the U.S. Preventive Services Task Force. No evidence in the case points towards the need for urine culture and sensitivity. Finasteride is a 5 α-reductase inhibitor. If the patient does not receive sufficient relief from maximum doses of a α1-receptor blocker, it may be added. However, it may take up to 6 months for a 5 α-reductase inhibitor to result in a noticeable difference in symptoms. Thus, finasteride is not a first-line treatment. The full therapeutic benefit of a α1-receptor blocker, on the other hand, will be apparent within 4-6 weeks. Post-void residual is a diagnostic tool used to determine if a patient with BPH will benefit from scheduled bladder catheterizations. A post-void residual >200 mL is associated with an increased risk of urinary tract infections. Scheduled catheterizations are usually reserved for cases in which medical or surgical interventions do not correct the problem; they are also used when medical and surgical interventions are contraindicated.

A man with suspected acute renal failure provides a urine specimen that demonstrates high urine osmolality, low sodium urine, and high BUN-to-creatinine ratio. The urine microscopic examination reveals few red blood cells and white blood cells with no casts. What is the most likely explanation? Answer Choices 1 Postrenal cause 2 Prerenal cause 3 Glomerulonephritis 4 Acute tubular necrosis 5 Interstitial nephritis

Explanation The clinical picture is suggestive of a postrenal cause. Common etiology includes obstruction of the urinary tract. Initial urine findings include high urine osmolality, low sodium urine, and a high BUN-to-creatinine level. The microscopic examination may be normal, but it also may contain red blood cells, white blood cells, or crystals. A prerenal cause would have hyaline casts present. Glomerulonephritis would have dysmorphic red blood cells and red blood cell casts on microscopic examination. Acute tubular necrosis would have pigmented granular casts and renal tubular epithelial cells on microscopic examination. Interstitial nephritis would have pyuria (including eosinophiluria), white blood cell casts, and white blood cells on microscopic examination. Patients may also present with fever and a transient maculopapular rash.

The mother of a 2-year-old Caucasian boy explains that the boy has had a 4-day history of fever and cries during urination. She has been giving him acetaminophen, but the fever keeps coming back. She also says that his urine "smells funny." Family history is positive for a 4-year-old brother with Down syndrome. His temperature is 39°C. Urinalysis and culture reveal urinary tract infection. A VCUG and a renal bladder ultrasonography reveal vesicourethral reflux (VUR). Question What is a risk factor for VUR in this boy? Answer Choices 1 His gender 2 Acetaminophen treatment 3 His age 4 His ethnicity 5 Family history of Down syndrome

Explanation Ethnicity is a significant risk factor for vesicourethral reflux (VUR) because it is 10 times more common in Caucasians than African-Americans. VUR is the retrograde flow of urine from the bladder towards the kidneys due to a dysfunctional vesicoureteric junction. It may occur alone or in association with other urological conditions like posterior urethral valves. Although prevalence is more common in male neonates, VUR is 5 times more common in female children compared to male children after the age of 1 year. Acetaminophen treatment and family history of Down syndrome have no association with VUR.

Your patient is a 42-year-old woman with a 1-year history of amenorrhea and galactorrhea. She also has been experiencing fatigue, somnolence, and easy bruising. She takes Metformin for type II diabetes and an ACE inhibitor for hypertension. On examination, you find an obese woman with peripheral edema (ankles and hands), hirsutism, and diabetic retinopathy; the rest of the examination is normal. Her prolactin levels are 144.8 ng/mL (normal range is 4.8 to 23.3 ng/mL); repeated prolactin is 110 ng/mL. TSH and free T4 are normal. Question What should be your next step in regards to making a diagnosis? Answer Choices 1 Obtain serum testosterone levels 2 Perform kidney function tests 3 Schedule pituitary MRI 4 Perform visual field testing 5 Evaluate coagulation status

Explanation The correct response is that you should perform kidney function tests. Your patient has galactorrhea, amenorrhea, and signs and symptoms that suggest a renal insufficiency (fatigue, somnolence, easy bruising, peripheral edema); the insufficiency was probably caused by long-standing diabetes (diabetic retinopathy develops after a long history of diabetes). Chronic renal failure elevates prolactin by decreasing peripheral clearance of the hormone. Evaluation of hyperprolactinemia should include a review of medications, including estrogen therapy, and history of fertility or gonadal dysfunction. Elevated prolactin levels can result in secondary hypogonadism. In general, signs and symptoms of hyperprolactinemia are due to either the excess hormone secretion (i.e., galactorrhea and amenorrhea) or local compression (e.g., new-onset or persistent headache, dizziness, visual changes, and vision loss). Since your patient has no signs of any of them, biochemical tests should be ordered before the imaging. Laboratory evaluation should include a repeat serum prolactin test, measurements of TSH and free T4, and a pregnancy test. If the results come back normal and if other diagnoses are excluded, the most likely diagnosis is a prolactinoma. A pituitary MRI should only be obtained in such cases. Elevated prolactin levels can result in secondary hypogonadism. Serum testosterone levels should be checked in men with galactorrhea. Visual field testing can be performed in individuals with specific visual complaints, especially loss or impairment of peripheral vision.

A 23-year-old presents with frequent, painful urination and lower back pain. History and physical exam reveals suprapubic pain, no evidence of systemic illness or fever, and no history of hospitalization. A presumptive diagnosis of an urinary tract infection (UTI) is made, and a urine culture yields lactose-fermenting beta-hemolytic Gram-negative rods. What is the most likely etiology of the UTI? Answer Choices 1 Escherichia coli 2 Klebsiella pneumoniae 3 Enterobacter spp. 4 Proteus mirabilis 5 Pseudomonas sp.

Explanation The most common cause of an uncomplicated UTI is Escherichia coli, responsible for up to 85% of outpatient cases. Proteus sp. and Pseudomonas sp. are most often the etiologic agents of UTIs in hospitalized patients. Pseudomonas does not ferment lactose. Klebsiella pneumoniae and Enterobacter spp. are rarely beta-hemolytic.

A 24-year-old woman presents with severe diarrhea that she has been experiencing for 3 days, with no medical complaints before then. She now feels dizzy upon standing, her tongue is dry, and her eyes appear glazed. Her serum sodium concentration is 130 mEq/L. Which of the following findings is most likely? Answer Choices 1 Decreased serum aldosterone 2 Increased serum atrial natriuretic peptide 3 Increased intravascular volume 4 Increased serum ADH concentration 5 Urine osmolality less than serum osmolality

Explanation The patient has obvious symptoms of dehydration. She also has a low serum sodium concentration, which shows that the body's drive to conserve water supersedes that of maintaining an adequate sodium concentration. ADH makes the normally impermeable collecting tubules permeable to water. Hence ADH promotes water reabsorption in the collecting tubules, so urine volume is decreased, and urine osmolality is increased.The patient's condition can be readily explained in view of an increased level of serum ADH. Serum aldosterone will be increased, as the body will also be trying to conserve sodium. Atrial natriuretic peptide will be decreased for the same reason.

A 3-year-old girl is brought to the clinic by her mother; she tells you that the child has not been eating well over the past month and has developed swelling in the abdomen. On exam, the child has a smooth abdominal mass that is the size of a baseball on the left side. Vital signs reveal a blood pressure of 134/82 mm Hg, temperature of 99.8 degrees F, and respirations of 16 breaths per minute. Urinalysis shows only 1+ red blood cells, and CBC and CMP are within normal limits. Question What is the most likely diagnosis? Answer Choices 1 Multicystic dysplastic kidney disease 2 Splenomegaly 3 Lymphoma 4 Wilms' tumor 5 Renal cell carcinoma

Explanation Wilm's tumors account for 95% of all urinary tract malignancies in childhood. The median age of diagnosis is 3 years. These tumors are usually a solitary, unilateral mass. Children typically present with an enlarging smooth abdominal mass confined to one side of the abdomen. Other associated symptoms include: abdominal pain, fever, hypertension (60%), and hematuria (25%). Multicystic dysplastic kidney disease is more common on the left side and is more common in boys. 60% of unilateral multicystic dysplastic kidneys involute in the first 3 years of life. Therefore, instead of getting bigger to reveal a palpable mass, these typically get smaller over time in children. Splenomegaly in children is usually indicative of hepatic or hematologic disease. This child's CBC and CMP are within normal limits, making this diagnosis unlikely. Lymphoma often presents with painless, firm lymphadenopathy typically confined to 1 or 2 lymph node areas (usually the supraclavicular and cervical nodes). Mediastinal lymphadenopathy is also a common presentation. It can manifest with coughing and shortness of breath. Abdominal masses are not a common presentation of lymphoma, and CBC is typically abnormal in these patients. Renal cell carcinoma accounts for only 3% of childhood urinary tract malignancies. Wilms' tumor is much more likely to be the diagnosis in a child.

A 2-year-old boy has a solid mass of tumor on the lower pole of his right kidney. Histopathological studies show whorls of mesenchymal cells and clusters of disorganized epithelial cells rather than renal corpuscles and renal tubules, which confirm the diagnosis of Wilms tumor. Question What is true regarding treatment/prognosis of this diagnosis? Answer Choices 1 If tumor penetrates kidney capsule, post-treatment survival is less than 50%. 2 Combination chemotherapy is appropriate after surgery. 3 Bilateral involvement is incurable. 4 There are no associated congenital abnormalities. 5 The tumor is not radiosensitive.

Explanation Wilms tumor is one of the earliest examples where adjuvant systematic chemotherapy and surgical reduction of the tumor is employed with successful outcome even for advanced disease. Even with metastases to liver and lungs, or with bilateral tumors, patients still have better than a 50% chance of survival. Some cases of Wilms tumor are associated with mutations in the WT-1 gene, a tumor suppressor gene that is also involved in the development of the urogenital system. Consequently, Wilms tumor is commonly associated with developmental defects in the gonads. The tumor is radiosensitive, and advanced cases can be treated with radiotherapy, albeit with a high complication rate in young children.

A 17-year-old high school basketball player had been hospitalized with a throat infection, fever, and a question of pneumonia. She had been taking a number of antibiotics and her physician noted edema and an elevated blood pressure. Ten days after being discharged, she began to note blood in her urine. You order an urinalysis and red blood cells and protein are noted in her urine. What is the most likely diagnosis? Answer Choices 1 Chronic Renal Failure 2 Nephrolithiasis 3 Cystitis 4 Glomerulonephritis 5 Nephrotic Syndrome

Explanation The clinical picture is suggestive of glomerulonephritis. Signs and symptoms of glomerulonephritis include hematuria, proteinuria, edema, and hypertension usually occurring 7 to 10 days after the onset of acute pharyngitis. Chronic renal failure is most commonly caused by untreated or poorly-controlled diabetes mellitus and untreated or poorly-controlled hypertension. This is no indication of untreated or poorly-controlled diabetes mellitus and untreated or poorly-controlled hypertension in this patient. Nephrolithiasis, or kidney stones, would present with flank pain, ± fever, hematuria, and oliguria. There is no indication of flank pain or oliguria in this patient. Cystitis or bladder infection or inflammation would present with suprapubic pain, dysuria, nocturia, odd or foul smelling urine, an increase in urinary frequency, and no fever. On urinalysis, a cloudy appearance and white blood cells would be seen on microscopic examination. These symptoms are not present in this patient. Nephrotic syndrome presents with massive proteinuria, > 3.5g per 24 hour urine, hypoalbumenemia, edema, and hyperlipidemia. Oval fat bodies may be seen on urinalysis. These symptoms, with the exception of edema, are not present in this patient.

A 40-year-old woman presents to the emergency room with flank pain and fever of 102°F. Upon interview of the patient, she also noted that her urine output has decreased over the last few days. On further inquiry, the patient stated that she has been trying to lose weight by increasing her protein in her diet, exercising and decreasing her normal fluid intake. On urinalysis, red blood cells are noted. What is the most likely diagnosis? Answer Choices 1 Acute renal failure 2 Nephrolithiasis 3 Cystitis 4 Chronic renal failure 5 Nephritic syndrome

Explanation The clinical picture is suggestive of nephrolithiasis(renal stones). Most patients with renal stones present with flank pain and hematuria with or without fever. There are numerous causes of renal stones but dehydration favors stones formation and a high protein diet can predispose a patient to stone formation. Acute renal failure can be associated with renal stones. Renal stones are categorized as a postrenal cause of acute renal failure. But with acute renal failure, a rapid rise in BUN and creatinine would be seen on serum chemistries, which are not seen in this patient. Cystitis or bladder infection or inflammation would present with suprapubic pain, dysuria, nocturia, odd of foul smell to urine, and an increase in urinary frequency, and no fever. On urinalysis, a cloudy appearance and white blood cells would be seen. These symptoms are not present in this patient. Chronic renal failure is most commonly caused by untreated or poorly-controlled diabetes mellitus and untreated or poorly-controlled hypertension. This is no indication of untreated or poorly-controlled diabetes mellitus and untreated or poorly-controlled hypertension in this patient. Nephritic syndrome is characterized by proteinuria, hematuria, azotemia, red blood cell casts, oliguria, and hypertension.

A 55-year-old Hispanic woman presents to establish care. She recently went to a health fair, where she had some basic serum chemistries drawn. Her serum creatinine was 1.5 mg/dl. On a questionnaire she completed in your waiting room, she noted that she has no known past medical history, except for occasional muscular aches, for which she takes indomethacin (about 2 or 3 times in the past 8 months). There is no family history of renal disease. Her BP is 142/82 mm Hg, and her body mass index is 31 kg/m2. Question What additional study would be most helpful in determining the cause of her elevated creatinine? Answer Choices 1 Complete blood count 2 Renal ultrasound 3 Fasting serum glucose level 4 Urinalysis to evaluate for the presence of hematuria 5 Urine culture

Fasting serum glucose level Explanation The most likely cause of chronic kidney disease in the United States is diabetes, so a history of elevated blood sugars is important to know. This patient is noted to be somewhat overweight, putting her at higher risk for diabetes. A fasting serum glucose level would be useful to rule of diabetes mellitus. Complete blood count may reveal anemia related to chronic kidney disease, but would not be useful in identifying the cause of kidney dysfunction. Chronic urinary infections can lead to scarring of the renal interstitium and to chronic kidney disease. It is important to ask about this risk, particularly if other risk factors for kidney disease are not noted. A urine culture can be used to rule out urinary tract infection. However, in this patient with no signs or symptoms of urinary tract infection, urine culture is not likely to be a useful diagnostic test. Hematuria can be caused by a variety of disorders including glomerular disease, acute and chronic infections, nephrolithiasis, renovascular disease, cystic kidneys, and urogenital cancers. It is prudent to ask about hematuria. Still, these disorders are less common than diabetic and hypertensive nephropathies, so they are not the most likely cause of her elevated creatinine. Renal ultrasound can identify renal scarring, tumors, kidney stones, and other structural abnormalities that could be causing kidney dysfunction, but ultrasound is not a usual first-line screening test.

Your patient is a 65-year-old man presenting with a 3-week history of progressive leg swelling, dyspnea on exertion, and increased thirst. A couple days ago, he started having nausea, headache, vomiting, and his gait became unsteady. He does not have chest pain or shortness of breath at rest. His medical history is significant for hypertension, type II diabetes, and chronic kidney failure. His medications include a beta blocker/alpha-1 blocker (carvedilol), aspirin, angiotensin-converting enzyme (ACE) inhibitor (lisinopril), biguanide (metformin), and a loop diuretic (furosemide). Physical examination reveals bilateral pitting edema up to his knees, swelling in the face, peripheral pulses 2+ in all extremities, and an unsteadiness during Romberg testing with both eyes open and closed. Auscultation reveals bilateral crackles at the lung bases, and the rest of his physical examination is not contributing. Laboratory findings are: Test Results Reference Range (males) Sodium Potassium Chloride CO2 Creatinine Blood urea nitrogen Fasting Glucose Serum osmolality 125 mEq/L 3.1 mEq/L 97 mEq/L 22 mmol/L 2.6 mg/dL 40 mg/dL 189 mg/dL 270 mOsm/kg H2O 135-145 mEq/L 3.5-5.0 mEq/L 95-108 mEq/L 20-32 mmol/L 0.7-1.4 mg/dL 7-30 mg/dL <110 mg/dL 280-295 mOsm/kg H2O Total protein Albumin Total bilirubin Alkaline phosphatase Alanine aminotransferase Aspartate aminotransferase 6.0 g/dL 2.9 g/dL 1.1 mg/dL 90 IU/L 34 IU/L 37 IU/L 6.0-8.5 g/dL 3.5-5 g/dL 0.1-1.3 mg/dl 40-120 U/L 0-35 U/L 0-37 U/L Urine Sodium <10 mEq/L Variable (depending on volume status) Red blood cell count White blood cell Hemoglobin Platelets 4 000 000/uL 6.8 000 /mm 11.0 g/dL 300 000/mm3 4,7 000 000- 6,1 000 000/uL 3.8000-10.8000/mm3 13.8-17.2 g/dL 150 000-450 000/mm3 Question What is the most probable cause of his neurological symptoms and signs? Answer Choices 1 Hyponatremia 2 Hypokalemia 3 Hypervolemia 4 Hyperglycemia 5 Anemia

Hyponatremia Explanation Nausea, headache, vomiting, unsteady gait, and Romberg showing vestibular disfunction are most probably caused by hyponatremia, the most common electrolyte abnormality in clinical practice. Hyponatremia is defined as a serum sodium concentration less than 135 mEq/L and is severe when the serum sodium level falls below 125 mEq/L. It is often relatively asymptomatic. Hyponatremia manifests mostly by neurological symptoms secondary to cerebral edema. When the serum sodium level drops, water moves from the extracellular space into cells and causes cellular swelling. Brain swelling is probably the first to manifest, given the fact that the brain contains around 80% water and has no space within a skull to accommodate swelling. Early symptoms of cerebral edema are nausea, headache, and vomiting. In the acute cases (those that happen over several hours), hyponatremia may cause acute cerebral edema, with seizures, brain herniation, respiratory failure, coma, and death. However, hyponatremia that develops over several days gives a chance for the brain to adapt to the excess water by extruding osmolytes (potassium, chloride, and small organic molecules), thus equalizing the osmolality between the intra- and extracellular fluid. Therefore, the symptoms of chronic hyponatremia are less severe, even when laboratory shows severe hyponatremia, slightly decreased cognitive functions, mild nausea, and mild gait and balance disturbances. Physical examination in patients with hyponatremia is generally not specific and usually relates to the underlying condition - in this case, probably renal failure. Hypokalemia will not present with neurological symptoms. Mild hypokalemia is often without symptoms, and severe hypokalemia, with serum potassium concentrations of 2.5-3 mEq/L, causes mostly muscle symptoms, such as weakness, pain, cramps, constipation (disturbance in the smooth muscles function), and, when severe, can cause flaccid muscle paralysis. Volume overload, primarily salt and water, will manifest as subcutaneous edema, ascites, and/or pulmonary edema, presenting with the symptoms of the failure of those organs. Your patient has volume overload, but neurological symptoms in hypervolemia with hyponatremia are also caused by sodium disbalance. Hypervolemia with hyponatremia is most commonly caused by chronic heart failure, liver, or kidney disease. Hyperglycemia of 189 mg/dL is unlikely the cause of progressive leg swelling, dyspnea on exertion, increased thirst, nausea, headache, vomiting, unstaeady gait, and balance disturbances. Severe anemia may present with dypnea, thirst, and dizziness, but anemia in this case is very unlikely to be the cause and is not the cause of progressive leg swelling and neurological symptoms and signs.

An 83-year-old man presents to his urologist's office with "problems down there." He is a poor historian, but his wife accompanies him and helps with his review of systems. She does not report that he has any voiding issues and reports that he has never had to see a urologist before now. Genitourinary examination is suggestive of phimosis. Question What physical examination finding would support this diagnosis? Answer Choices 1 Entrapment of the foreskin behind the glans penis in an uncircumcised male patient 2 Erythema and edema of the glans penis in an uncircumcised male patient 3 Erythema and edema of the phallus in a circumcised male patient 4 Erythema and edema of the glans penis in a circumcised male patient 5 Inability to retract the foreskin over the glans penis in an uncircumcised male patient

Inability to retract the foreskin over the glans penis in an uncircumcised male patient Explanation Inability to retract the foreskin over the glans penis in an uncircumcised male patient is the correct answer. Phimosis can be the result of recurrent infections or irritation, advanced age, diabetes, and poor hygiene. Patients can experience painful erections, recurrent balanitis, and voiding difficulties. Treatment can be with topical steroids followed by gradual retraction of the foreskin or circumcision. Entrapment of the foreskin behind the glans penis in an uncircumcised male patient is not the correct answer. This is a description of a patient with paraphimosis. Typically, this results in retraction of the foreskin for medical or hygiene purposes that is not followed by properly pulling the foreskin back in place over the glans penis. This can be very painful, and needs to be manually reduced as soon as possible to prevent necrosis of the glans penis. A dorsal slit or circumcision may be necessary if manual reduction is not possible. Erythema and edema of the glans penis in an uncircumcised male patient is not the correct answer. This description most closely describes balanitis and does not address the placement or status of the foreskin, which is the main concern with phimosis. Erythema and edema of the phallus in a circumcised male patient is not the correct answer, as patients who are circumcised cannot experience phimosis due to their lack of foreskin. Erythema and edema of the glans penis in a circumcised male patient is not the correct answer, as patients who are circumcised cannot experience phimosis due to their lack of foreskin.

A 55-year-old woman presents with a 2-month history of gross hematuria. She states she has no pain with urination, but the hematuria is persistent. On questioning, she states that she has had a 1-month history of some progressively worsening left flank pain and persistent back pain. The pains are not debilitating, but they are nagging. She has no chronic medical problems. She admits to a 50-pack/year smoking history, and she states she is currently retired from her job as a teacher. Vital signs are within normal limits, and physical exam reveals a left side abdominal mass. Urine dipstick only shows too numerous to count RBCs, and urine cultures are negative. CT scan of the abdomen and pelvis with and without contrast reveals a 4.2 cm solid enhancing lesion in the left renal parenchyma. Question What is the most effective treatment for this patient? Answer Choices 1 Radiation of the kidney 2 Chemotherapy 3 Left nephrectomy 4 Hormonal therapy 5 Observation

Left Nephrectomy Explanation This patient likely has renal cell carcinoma of the left kidney. Surgery is most effective treatment for primary RCC. Partial nephrectomies are common in peripheral tumors that are less than 4 cm in size, but total nephrectomies are the treatment of choice in larger or more centrally located tumors. Radiation preoperatively and/or postoperatively has not been shown to change overall survival with RCC. Chemotherapy currently available is not beneficial in RCC. Hormone therapy, including progesterone therapy, has not been shown to be effective in treatment of RCC. Observation is a viable option in elderly patients and poor surgical candidates if the tumor is less than 3 cm in size. These patients are often followed with CT scans every 6 months.

A 55-year-old man is admitted to the hospital with abdominal pain. He has a past medical history of hypertension. A CT scan with contrast reveals the presence of a small bowel obstruction. An NG tube is placed; IV narcotics are given for pain control; and IV fluids are started. A few days into his hospitalization, he develops confusion. Arterial blood gas is drawn, revealing the following: pH- 7.55 CO2- 42 HCO3- 28 Question What in the patient's history is contributing to his acid base disorder? Answer Choices 1 Anxiety 2 Hypertension 3 CT contrast administration 4 NG tube placement 5 IV narcotic administration

NGT Explanation NG tube placement is correct. The patient's ABG values indicate that he is suffering from metabolic alkalosis. Metabolic alkalosis is associated with elevated pH. The pH becomes elevated due to decreased hydrogen ion concentration or increased bicarbonate concentration. NG tube placement causes the loss of HCl from the stomach, and this is the most likely cause of metabolic alkalosis in this patient. Anxiety is incorrect. During anxiety attacks, patients may develop respiratory alkalosis due to hyperventilation. Carbon dioxide levels decrease during hyperventilation due to the patient's rapid breathing. Hypertension is incorrect. Hypertension itself is not associated with metabolic alkalosis. Certain medications, such as diuretics, may lead to metabolic alkalosis due to the associated fluid loss. However, there is no mention in the patient's history of diuretic use. IV narcotic administration is incorrect. The use of IV narcotics may cause CNS depression, which would lead to respiratory acidosis. Respiratory acidosis results from hypoventilation. CT contrast administration is incorrect. CT contrast administration may lead to renal failure in certain individuals. However, this would likely be associated with a metabolic acidosis and not metabolic alkalosis.

A 43-year-old Caucasian man with a 20-year history of bipolar disorder comes to your office for the first time. He presents with long term polyuria and polydipsia. He previously took lithium for mood stabilization for 15 years before initiating divalproex sodium therapy. He stopped using lithium because of the polyuria, but felt that the polyuria never fully subsided. His weight is stable, and he has no other urinary complaints. His blood pressure is 115/80 mm Hg and his physical exam is normal. His urinalysis shows no blood, cells, protein, glucose, nitrate, casts, or crystals. Question What is the most likely cause of his polyuria? Answer Choices 1 Central diabetes insipidus 2 Nephrogenic diabetes insipidus 3 Polyuria secondary to hyperglycemia 4 Polyuria following acute kidney injury 5 Polyuria secondary to polydipsia

Nephrogenic diabetes insipidus

A 43-year-old Caucasian man with a 20-year history of bipolar disorder comes to your office for the first time. He presents with long term polyuria and polydipsia. He previously took lithium for mood stabilization for 15 years before initiating divalproex sodium therapy. He stopped using lithium because of the polyuria, but felt that the polyuria never fully subsided. His weight is stable, and he has no other urinary complaints. His blood pressure is 115/80 mm Hg and his physical exam is normal. His urinalysis shows no blood, cells, protein, glucose, nitrate, casts, or crystals. Question What is the most likely cause of his polyuria? Answer Choices 1 Central diabetes insipidus 2 Nephrogenic diabetes insipidus 3 Polyuria secondary to hyperglycemia 4 Polyuria following acute kidney injury 5 Polyuria secondary to polydipsia

Nephrogenic diabetes insipidus This patient's symptoms of excessive production of urine (polyuria) is most likely caused by nephrogenic diabetes insipidus secondary to lithium use. Lithium impairs the distal water reabsorption in the collecting ducts, mediated by vasopressin (ADH), leading to the production of large quantities of dilute urine. Unfortunately, lithium use, even for as short a period as 1 year, can lead to irreversible damage of the renal tubules (via down-regulation and production of receptors and channels responsible for water reabsorption). Treatments for lithium-induced nephrogenic diabetes insipidus include A) amiloride, a distal-tubule acting diuretic which competes with lithium for access to ion channels and thus prevents the lithium-induced polyuria, and B) hydrochlorothiazide with a low-salt diet, in order to effectively decrease the quantity of urine produced. Depakote can be substituted for lithium,but as mentioned above, lithium may cause irreversible tubular damage. In central diabetes insipidus, decreased levels of ADH are produced by the posterior pituitary. Nephrogenic, not central diabetes insipidus is the expected complication of lithium use. ADH is usually released in response to increases in serum osmolality, and/or decreases in arterial volume. Non-osmotic causes such as nausea in post-opertive setting can also elicit ADH release. Head injury, granulomas, and other central nervous system abnormalities can lead to impaired ADH production and release. Central and nephrogenic diabetes are distinguished by a water deprivation test as described in the following table: Disorder Response to ADH Treatment Central DI >50% increase in urine osmoles Vasopressin, DDAVP, Decreased salt intake Nephrogenic DI <10% increase in urine osmol. Diuretics, Removal of offending medications, Decreased salt intake Primary polydipsia <10% increase in urine osmol. Fluid restriction Central diabetes insipidus is treated by the administration of vasopressin (DDAVP), intra-nasally, orally, or intravenously. The differential diagnosis for polyuria also includes the presence of excess serum solutes, ie: glucose and mannitol than can be reabsorbed by the kidney tubules. Hyperglycemia is an unexpected cause of this patient's polyuria since no glucose was found in this urinalysis. Following acute kidney injury (previously termed acute renal failure), damaged renal tubules may temporarily have difficulty concentrating and diluting urine. The benign urine (lack of blood, protein, cells and casts), makes this cause of polyuria unlikely. Polyuria secondary to polydipsia is intuitive. Unlike patients with diabetes insipidus, patients with primary polydipsia (also termed psychogenic polydipsia) will have decreased urine production when they decrease their fluid intake, will increase their plasma ADH levels, and concentrate their urine appropriately in response to decreased fluid intake. Primary polydipsic patients are also less prone to abnormalities of sodium. Note that polyuria is not equivalent to urinary urge, the sudden need to void, or to urinary frequency, voiding in the order of 7-15 times/day.

A 42-year-old woman presents for an evaluation of back pain. She states that she has a history of recurrent UTIs and that on occasion, she has noted that her urine appears red. Upon physical examination, she demonstrates CVA tenderness and a palpable abdominal mass. Her vital signs include blood pressure of 145/90 mmHg, respiratory rate of 16 breaths/min, and heart rate of 72 bpm. You order a urinalysis which has the following abnormal results: leukocyte esterase 1+, protein trace, and blood 3+. The microscopic examination reveals 5-10 WBCs/hpf and 20-30 RBCs/hpf. An abdominal ultrasound demonstrates multiple, bilateral fluid collections within the kidneys. Question What, if any, is the most likely underlying renal pathology? Answer Choices 1 Normal kidneys 2 Renal cell carcinoma 3 Goodpasture's syndrome 4 Polycystic kidney disease 5 Medullary sponge kidneys

Polycystic Kidney Disease The clinical picture is suggestive of acute pyelonephritis in the setting of polycystic kidney disease. Abdominal or flank pain, blood in the urine, recurrent UTIs, and hypertension are common symptoms of polycystic kidney disease. Cystic lesions on abdominal ultrasound are a hallmark of the disorder. These lesions would not be present in a patient with normal kidneys. A solid renal mass is most often seen in the context of renal cell carcinoma. Goodpasture's syndrome is defined as a combination of glomerulonephritis and pulmonary hemorrhages. The disease is usually preceded by an upper respiratory tract infection. Patients usually present with dyspnea and hemoptysis. This is not seen in this patient. Medullary sponge kidney is a common and benign disorder presenting at birth and is not usually diagnosed until the 4th of 5th decade. Common findings include gross hematuria, recurrent UTIs, and renal stones. The kidneys will have irregular enlargements of the medullary and interpapillary collecting ducts giving a "Swiss cheese" appearance. This is not the finding present on this patient's abdominal ultrasound.

A 7-year-old girl presents for a 1-day history of bloody urine. The grossly bloody urine scared both the girl and her parents, but she denies dysuria and frequency. No trauma or sexual abuse has occurred. The parents deny recent fever in the patient, but note that she had a fever for a few days accompanying a sore throat. She was given acetaminophen at an appropriate dosage for her weight, and about 3 days of some leftover amoxicillin; both the fever and pharyngitis then resolved. Her past medical history is unremarkable for any chronic illnesses. Her only medication is a multivitamin, and she has NKDA. She has had no surgeries, and family history is unremarkable for urinary tract disorders or any bleeding disorders. On physical exam, she appears interactive and in no apparent distress; she is well-nourished, non-obese, and perhaps mildly edematous. Vitals are a temperature of 99.0° F and a BP of 138/85 mm Hg; P is 98, and RR is 20. No rashes are found. Cardiac exam reveals normal rate and rhythm; there are no murmurs or rubs. On abdominal exam, her abdomen is non-distended, non-tender, and without masses or hepatosplenomegaly. She has no CVA tenderness. A urinalysis is performed; the significant findings are as follows: Protein 2+ Glucose Negative Ketones Negative Blood 4+ Nitrites Negative Leukocyte esterase 1+ Microscopic analysis is performed, which confirms the presence of red blood cells (RBCs) that are dysmorphic and indicate RBC casts. Blood tests are ordered, which reveal a complete blood count within normal limits and a complete metabolic panel with elevated creatinine. The patient's antistreptolysin O level is elevated, and her serum complement level is low. Question What is the most likely diagnosis for this patient? Answer Choices 1 Acute cystitis 2 Analgesic abuse nephropathy 3 Diabetic nephropathy 4 Postinfectious glomerulonephritis 5 Renal cell carcinoma

Postinfectious glomerulonephritis Explanation Of the listed choices, this pediatric patient most likely has postinfectious glomerulonephritis (PIGN), or poststreptococcal glomerulonephritis. The diagnosis is supported by both her history and the test results. "An estimated 5 to 10% of patients with streptococcal pharyngitis" may develop this condition.1 Though the test is not specific, the antistreptolysin O, along with the history of pharyngitis, help establish a recent streptococcal infection for this patient. PIGN falls under the broader category of nephritic syndromes characterized by hematuria and RBC casts, with possible proteinuria, edema, hypertension, and elevated serum creatinine. Treatment for PIGN is supportive, and normal renal function returns for the vast majority of patients. While the hematuria and presence of leukocyte esterase are consistent with acute cystitis, the rest of the history and findings are not. The classic presentation of acute cystitis includes dysuria and frequency. The remainder of the patient's presentation (i.e., edema, elevated BP, RBC casts, and serum tests) is inconsistent with acute cystitis. The leukocyte esterase is not specific for infection; in this case, it indicates inflammation in the urinary tract. Analgesic abuse nephropathy is a chronic tubulointerstitial nephritis "caused by cumulative lifetime use of large amounts (i.e., ≥ 2 kg) of certain analgesics."1 Though this patient was dosed with acetaminophen, her use was in small amounts and only for a few days. The history can rule out this diagnosis. Diabetic nephropathy is an important diagnosis to keep in mind when evaluating renal dysfunction, as it is the most common cause of nephrotic syndrome. The diagnosis includes a history of diabetes and demonstrated proteinuria, which can be confirmed by various methods. This patient has no history of diabetes, and even if she presented with elevated glucose, diabetic nephropathy would not be expected to develop for many years. Renal cell carcinoma (RCC) is suggested by persistent hematuria, especially in individuals over age 50 or in those at high risk. The serum findings in this patient are not consistent with the diagnosis of RCC. CT, MRI, and sometimes renal biopsy help confirm the diagnosis for RCC.

A 55-year-old man presents with increased urinary frequency; he is also experiencing difficulty starting and maintaining a steady stream, nocturia, and the inability to achieve an erection for the past few months. During rectal examination, you note a non-tender, non-enlarged prostate with an isolated left posterior lobe nodule. What option is the most appropriate? Answer Choices 1 Refer the patient for a transrectal ultrasound of the prostate and order a PSA level 2 Order a serum acid phosphatase 3 Reassure the patient and schedule a follow-up appointment in 6 months 4 Initiate prazosin and schedule a follow-up appointment in 4 weeks 5 Initiate norfloxacin therapy for 7 days and schedule a follow-up in 2 weeks

Refer the patient for a transrectal ultrasound of the prostate and order a PSA level Explanation The clinical picture is suggestive of prostate cancer. Signs and symptoms include those similar to benign prostatic hypertrophy (BPH) such as urinary frequency, urinary hesitancy, nocturia, hematuria, and difficulty achieving erections. Most patients are asymptomatic. Prostate cancer may be manifested as focal nodules or areas of induration within the prostate. Modern transrectal ultrasound provides high-definition images of the prostate. It also guides biopsy. Measurement of the PSA is useful in detecting and staging prostate cancer. Serum acid phosphatase would not be useful since it is found in many organs. A prostatic acid phosphatase would be more specific in diagnosing prostate cancer. Since a nodule was found in this patient, it needs to be investigated further, and reassuring the patient would be considered inappropriate medical practice. Prazosin is an alpha-blocker used in the treatment of BPH. BPH usually results in a smooth, firm, elastic enlargement of the prostate, which is not seen in this patient. Norfloxacin is an antibiotic used in the treatment of urinary tract infections. Signs and symptoms of a UTI include painful urination, increased frequency, and an odd smell to the urine. Urinalysis may show positive leukocyte esterase, nitrates, and blood, with WBCs and bacteria on microscopic examination. None of these symptoms/reults are indicated in this patient.

A 55-year-old woman presents with a 2-month history of gross hematuria. She states she has no pain with urination, but the hematuria is persistent. On questioning, she states that she does have some progressively worsening left flank pain and persistent back pain over the past month. The pains are not debilitating, but they are nagging. She has no chronic medical problems. She admits to a 50-pack year smoking history, and she states she is currently retired from her job as a teacher. Vital signs are within normal limits, and physical exam reveals a left side abdominal mass. Urine dipstick only shows too numerous to count RBCs, and urine cultures are negative. Question What is the most likely diagnosis? Answer Choices 1 Acute cystitis 2 Bladder cancer 3 Renal cell carcinoma 4 Urethritis 5 Ureteral calculi

Renal cell carcinoma Explanation Hematuria is the most common presenting sign of urinary tract cancer. Silent or painless hematuria suggests tumor or renal parenchymal disease. Renal cell carcinoma can present with flank pain, hematuria, persistent back pain, and an abdominal mass; also, it can be found incidentally on CT scan; therefore, the clinical picture points to cancer with renal cell carcinoma as the most likely diagnosis. Smoking is a risk factor for renal cell cancer. RCC is more common in men than women (2:1), and it has a peak incidence in the 6th decade of life. Bladder cancer often presents with gross hematuria, but it is most typically painless. The left flank pain and back pain are more characteristic of RCC. Bladder cancer is the 2nd most common urologic cancer, and the mean age at diagnosis is 65 years. It is more common in men than women (2.7:1), and 98% of primary bladder cancers are epithelial malignancies (majority urothelial cell carcinomas). Acute cystitis typically presents with irritative voiding symptoms (frequency, urgency, dysuria) and suprapubic discomfort in addition to possible hematuria. Urinalysis will show pyuria, bacteriuria, and varying degrees of hematuria. Urine cultures will show specific organisms. Urethritis is inflammation of the urethra that presents with urethral discharge, dysuria, and itching. Urethritis is most often caused by a STD. This patient does not have a history of unprotected sexual intercourse or any other symptoms that would indicate this diagnosis. Ureteral calculi can present with hematuria, but it also typically presents with flank or abdominal pain as well, but no abdominal mass. If the stone is in the ureter, it often causes some hydroureter with or without hydronephrosis, both of which cause some pain or discomfort. The patient has no past history of forming stones.

A 62-year-old man is hospitalized because of a 1-week history of extreme malaise and painful skin changes. At the beginning, it was "just a rash", but it then developed into clusters of clear vesicles; new vesicles appear while old ones dry and crust over. His past medical history is significant for diabetes mellitus type II, for which he is on the diet-controlled. You find the rash distributed over his body, and you make a diagnosis of disseminated herpes zoster. Acyclovir IV is introduced, together with the supportive therapy, but after 2 days, the patient starts experiencing nausea and swelling; his urine output dramatically decreased with laboratory studies showing elevated BUN and creatinine. Question What is the most probable mechanism of renal failure in this patient? Answer Choices 1 Renal tubular obstruction 2 Endothelial injury 3 Hypersensitivity reaction 4 Interstitial nephritis 5 Kidney infection

Renal tubular obstruction Explanation Acyclovir precipitates in renal tubules because it is poorly soluble in urine. In that way, it causes the obstruction of renal tubules and acute renal failure. Endothelial injury is characterized by reduced vasodilation, a proinflammatory state, and prothrombic properties; it may be associated with hypertension and diabetes, particularly in type II diabetes with insulin resistance. However, in a patient with acute kidney failure who has been exposed to the high dose of parenteral therapy with a poorly soluble nephrotoxic drug, endothelial injury should not be your initial choice. Renal tubular cells dysfunction due to hypersensitivity reaction to drugs or by infection will cause acute interstitial nephritis. It is often associated with obstruction or reflux, so that you can include this in your list of differential diagnoses. However, the development of acute renal failure during the therapy with acyclovir makes renal tubular obstruction more likely. In acute interstitial nephritis, renal tubular cells dysfunction is caused primarily by a hypersensitivity reaction. When caused by an allergic reaction, the symptoms of acute tubulointerstitial nephritis are fever, rash, and enlarged kidneys. Besides, acyclovir-induced crystalluria that causes mechanical tubular obstruction is the better option in this case. Kidney infection with acute renal failure is not a probable diagnosis in a patient that has no back pain and no signs of a urinary tract infection.

A 29-year-old man has been trying to have a child with his wife for the past 3 years; they have not met with any success. The wife was thoroughly evaluated for infertility; the workup revealed no abnormalities. Examination of his inguinal region reveals an ill-defined tortuous swelling that increases with standing and coughing. Ultrasonography is suggestive of being a varicocele. His initial semen analysis reveals mild oligospermia. Question What is the best next step in management? Answer Choices 1 Embolization of dilated veins 2 Surgical mesh repair 3 Low dose testosterone 4 Repeat semen analysis 5 Surgical excision of dilated veins

Repeat serum analysis Explanation Repeat semen analysis is the correct answer. The results of a single semen analysis are inadequate in making a diagnosis of infertility. For a multitude of reasons, counts vary over time. 2 or 3 separate counts should be taken at least 2 - 4 weeks apart. Despite the presence of a varicocele, if a repeat semen analysis is normal and the varicocele remains asymptomatic, there is no reason for operative intervention. Embolization of dilated veins is incorrect. Embolization is an option in treatment of varicocele. The dilated veins of the pampiniform plexus are embolized using interventional radiology. It is used in the presence of a symptomatic varicocele or if infertility is confirmed. Surgical mesh repair is incorrect. Mesh repair is used in the management of a hernia, not in a varicocele. Low dose testosterone is incorrect. It may be an option if infertility is due to hypogonadism due to low testosterone. It is not an initial step in management. Surgical excision of dilated veins is incorrect. If the varicocele is symptomatic or if infertility is confirmed, surgical excision is an option.

A 33-year-old African American woman develops a loss of appetite and has a few episodes of vomiting. She reports constipation and lethargy. She thinks that her urine output has increased. Her history is significant for the presence of sarcoidosis. On physical examination, some muscle weakness is present. Laboratory tests are ordered and are pending. An EKG is done and shows a shortened QT interval. Question Based on the EKG changes, what electrolyte abnormality would be expected? Answer Choices 1 Hypercalcemia 2 Hyperkalemia 3 Hypocalcemia 4 Hypokalemia 5 Hyponatremia

The four causes of a short QT interval: Hypercalcemia Digitalis Thyrotoxicosis Increased sympathetic tone Explanation Hypercalcemia refers to an elevated calcium level. A shortened QT interval is seen with hypercalcemia. The time from the beginning of the QRS complex to the ending of the T wave is the QT interval. Sarcoidosis can be associated with hypercalcemia. Symptoms of hypercalcemia include anorexia, vomiting, constipation, and polyuria. Muscle weakness can be seen. None of the other electrolyte abnormalities listed are associated with sarcoidosis. Hyperkalemia refers to an elevated potassium level. Hyperkalemia can result in peaked T waves on EKG. The T wave is a reflection of ventricular repolarization. There can be widening of the QRS complex as well. The QRS complex is seen when the ventricle depolarizes. Hypocalcemia refers to a depressed calcium level. There would be a prolonged QT interval with hypocalcemia. The time from the beginning of the QRS complex to the ending of the T wave is the QT interval. Hypokalemia refers to a depressed potassium level. Hypokalemia can result in a depressed ST segment and the presence of U waves. The time from the end of the QRS complex to the beginning of the T wave is the ST segment. Hyponatremia refers to a depressed sodium level. Hyponatremia is not associated with EKG changes.

The picture below is a view of the inferior aspect of the penis and scrotum of a 2-month-old male infant. The mother brought the infant to a pediatric clinic because, during micturation, urine ran from the opening at the bottom of the midline groove instead of from the tip of the penis. Question What is the most likely diagnosis? Answer Choices 1 Epispadias 2 Cleft scrotum 3 Cryptorchism 4 Hypospadias 5 Bifid penis

This is an example of hypospadias. It occurs frequently (about 1/300 male infants) and is caused by inadequate midline fusion of the urogenital folds in male embryos; consequently, there is incomplete formation of the penile urethra. Hypospadias often results from inadequate production of androgens by the testes or inadequate receptors for the hormones. The external urethral orifice may be located anywhere along the ventral midline aspect of the glans penis, shaft of the penis, scrotum, or perineum. Epispadias is an abnormal urethral orifice on the dorsal side of the penis. Cleft scrotum is due to failure of fusion of the labialscrotal swellings. Cryptorchism is either unilateral or bilateral failure of descent of the testes into the scrotum. A bifid penis is often associated with bladder exstrophy.

A 1-month-old infant boy presents with a 101.5 fever. He was a full term baby born via vaginal delivery. His mother was GBS (group B strep) positive and treated adequately. Since birth he has been exclusively breastfed and has been gaining weight slowly. He has no URI symptoms and is active and feeding well. He receives a sepsis workup and is admitted for 48-hour sepsis rule out. At 24 hours, his urine culture grows e. coli. His other cultures are negative, and he is sent home on antibiotics. Question What further testing should be done as an outpatient when he has recovered? Answer Choices 1 Monthly urine cultures 2 No testing required until he has a second UTI 3 CT of the pelvis 4 MRI of the abdomen 5 Voiding cystourethrogram

Voiding Cystourethrogram Explanation Any boy who has a urinary tract infection should undergo an initial urethral catheterization to collect a urine sample as the infant or child is unable to void upon request. A sensitivity test could also be ordered, so that the health care provider can test the bacteria to confirm which medication is best for treating the UTI infection. Immediately following the clearing of infection, additional testing may be recommended to rule out abnormalities in the urinary tract. Kidneys can be damaged with repeated infections. The age of the child and the type of urinary infection will determine the types of tests that will be ordered. As this child is less than 2 months, a voiding cystourethrogram should be ordered to confirm that hydronephrosis, scarring, obstructive uropathy, or masses are not associated with the urinary tract infection. As an individual test cannot detect all that needs to be known about the urinary tract that could prove useful, multiple tests might be necessary. Such tests could include: Kidney and bladder ultrasound, computerized tomography scan, magnetic resonance imaging, radionuclide scan, or urodynamics.

A 63-year-old man presents with a 6-month history of symptoms of urinary frequency, hesitancy, and nocturia. In addition, burning dysuria has occurred on 2 occasions, requiring treatment with antibiotics. He has a 1-year history of angina pectoris, for which he takes occasional nitroglycerin. On physical examination, the blood pressure is 130/90; heart rate is 90/min and regular, and an enlarged prostate is palpable per rectum. Laboratory data, including EKG, yields no contraindication to surgery; he is operated upon for a transurethral resection of the prostate. Anesthesia and surgery are uneventful, and blood loss is minimal. 6 hours postoperatively, he experiences a shaking chill, a temperature of 40 degrees C, and his blood pressure is 90/60 mm Hg. What is the most likely diagnosis? Answer Choices 1 Gram-negative bacteremia 2 Myocardial infarction 3 Postoperative bleeding 4 Arrhythmia 5 Lobar pneumonia

gram negative bacteremia Explanation The correct response is Gram-negative bacteremia. Some degree of enlargement of the prostate is extremely common from the age of 50 onwards, but this type of enlargement often produces either minor symptoms, or no symptoms at all. However, benign hypertrophy of the gland results in elongation and tortuosity of the prostatic urethra, and the median lobe may become a large, rounded, swelling overlying the posterior aspect of the internal urinary meatus. Here, it can act like a ball valve, producing urinary obstruction. The deranged anatomy in the region of the internal meatus, may allow urine into the prostatic urethra. The urine in this situation sets up a desire to micturate and this produces one of the most common symptoms of prostatism, namely, frequency. This is particularly worrisome to the patient at night, as it interferes with his sleep. The obstruction, and instrumentation to relieve it, predisposes to urinary infection. The obstruction to the outflow of the bladder may result in renal failure and uremia. Gram-negative enteropathogens are the most common cause of urinary tract infections and intra-abdominal sepsis, especially post-operatively, in the acute abdomen. Septicemia causes high fever, shivering, headache, and rapid breathing; it may progress to delirium, coma, and death. Myocardial infection gives rise to chest pain, which is usually of greater severity and duration than in angina, and is associated with nausea, vomiting, sweating, and extreme distress. The patient may be cold and clammy with tachycardia, hypotension, cyanosis, and mild pyrexia (Postoperative bleeding may lead to hypotension and hypovolemic shock, unless fluid volume is rapidly replenished. Arrhythmias may give rise to tachy- and bradycardias, which are sometimes felt as palpitations. They may also present with their hemodynamic consequences: dyspnea, angina, collapse, or 'funny turns'. Corresponding EKG changes are diagnostic. Pneumonia is relatively slow in onset, with symptoms of systemic upset, fever, pleuritic pain, cough, and green sputum (may be scanty at first, or, 'rusty' in color, if due to pneumococcal). On examination, there will be signs of consolidation, or just localized crepitations. Tachypnea is a valuable sign, especially in the elderly, in whom there is high index of suspicion.

A 54-year-old man presents to his family physician with complaints that he has very large urine output and that he is constantly thirsty. On questioning, he states that his urine output is many liters per day and that he awakens at night to urinate. His history is also remarkable for a 25-year history of a bipolar disorder, treated effectively with lithium. His lab results are as follows: TEST RESULTS REFERENCE RANGE BUN 17 mg/dL 10-20 mg/dL Calcium 9.9 mg/dL 8.5-10.5 mg/dL Potassium 4.2 mEq/L 3.5-5.0 mEq/L Sodium 149 mEq/L 135-145 mEq/L Glucose (fasting) 109 mg/dL 65-110 mg/dL The results of his urinalysis are as follows: TEST RESULTS REFERENCE RANGE Urine dipstick blood negative negative Urine dipstick glucose negative negative Urine dipstick ketones negative negative Urine dipstick protein negative negative 24 hour urine protein 124 mg/24 hour <150 mg/24 hour Urine osmolality 40mOsm/kg 50-1400mOsm/kg Urine specific gravity 1.001 1.001-1.035 Urine pH 6.2 4.5-8.5 Question A water deprivation test with exogenous vasopressin administration is done under meticulous supervision. His urine osmolality does not increase. What organ is most involved in the pathophysiology of this condition? Answer Choices 1 Kidney 2 Pituitary 3 Bladder 4 Ureter 5 Urethra

kidney A known side effect of lithium is nephrogenic diabetes insipidus. Nephrogenic diabetes insipidus can be seen in a significant minority of the patients taking lithium. Nephrogenic diabetes insipidus results when the collecting duct of the kidney does not respond to antidiuretic hormone (ADH). In complete nephrogenic diabetes insipidus, the kidney is unresponsive to ADH. Therefore, exogenous vasopressin (ADH) will not increase the urine osmolality. The pituitary gland secretes numerous hormones. The posterior lobe of the pituitary is the neurohypophysis. One of the hormones secreted by the posterior pituitary is antidiuretic hormone (ADH). The production of the posterior pituitary hormones actually occurs in the hypothalamus. Antiduiretic hormone (ADH) is also known as arginine vasopressin. It works on the collecting tubules of the kidney to conserve water. When the pituitary does not secrete ADH, the condition is called central diabetes insipidus. The urinary bladder is a muscular sac. The bladder stores urine and it is not responsible for concentration or production of urine. Pathology of the bladder does not result in nephrogenic diabetes insipidus. The ureter is a tube that transmits urine from the kidney to the bladder. The ureter is not responsible for concentration or production of urine. Pathology of the ureter does not result in nephrogenic diabetes insipidus. The urethra is a tube that transmits urine from the bladder out of the body. The urethra is not responsible for concentration or production of urine. Pathology of the urethra does not result in nephrogenic diabetes insipidus.

A 54-year-old man notices that he has very large urine output and he is constantly thirsty. In addition to urinating large volumes during the day, he awakens at night to urinate. His history is remarkable for a 25-year history of a bipolar disorder, which is treated effectively with lithium. His lab results are as follows: TEST RESULTS REFERENCE RANGE BUN 17 mg/dL 10 - 20 mg/dL Calcium 9.9 mg/dL 8.5 - 10.5 mg/dL Potassium 4.2 mEq/L 3.5 - 5.0 mEq/L Sodium 149 mEq/L 135 - 145 mEq/L Glucose (fasting) 109 mg/dL 65 - 110 mg/dL The results of his urinalysis are as follows: TEST RESULTS REFERENCE RANGE Urine dipstick blood Negative Negative Urine dipstick glucose Negative Negative Urine dipstick ketones Negative Negative Urine dipstick protein Negative Negative 24 hour urine protein 124 mg/24 hour <150 mg/24 hour Urine osmolality 40mOsm/kg 50-1400mOsm/kg Urine specific gravity 1.001 1.001 - 1.035 Urine pH 6.2 4.5 - 8.5 Question A water deprivation test with exogenous vasopressin administration is done under meticulous supervision. His urine osmolality does not increase. What is most likely the case? Answer Choices 1 Syndrome of inappropriate ADH secretion 2 Central diabetes insipidus 3 Diabetes mellitus 4 Nephrogenic diabetes insipidus 5 Psychogenic polydipsia

nephrogenic diabetic insipidus Explanation A known side effect of lithium is nephrogenic diabetes insipidus. This side effect is very common. Nephrogenic diabetes insipidus results when the collecting duct of the kidney does not respond to the antidiuretic hormone (ADH). Antiduiretic hormone (ADH) is also known as arginine vasopressin. It is secreted from the pituitary. It works on the collecting tubules of the kidney to conserve water. Nephrogenic diabetes insipidus is characterized by renal resistance to ADH. This results in a large output of dilute urine. There will be polyuria and polydipsia. Hypernatremia would be present. In contrast to central diabetes insipidus, plasma ADH level would be elevated and there is no response to the administration of vasopressin. Nocturia can be present. In complete nephrogenic diabetes insipidus, the kidney is unresponsive to ADH. Therefore, exogenous vasopressin (ADH) will not increase the urine osmolality. The syndrome of inappropriate ADH secretion (SIADH) is due to excess ADH (antidiuretic hormone) or an ADH like substance. The syndrome of inappropriate ADH secretion is characterized by concentrated urine. There will be an accompanying serum hyponatremia. Hyponatremia can cause central nervous system symptoms (obtundation, seizure, and coma). Central diabetes insipidus is characterized by a lack of antidiuretic hormone (ADH) secretion from the pituitary. Because of the deficiency of ADH, there is an inability of the kidney to concentrate the urine. This results in a large urine output of dilute urine. There will be polydipsia and polyuria. Nocturia can be present. Hypernatremia would be present. In contrast to nephrogenic diabetes insipidus, there is an increase in urine osmolality in response to the administration of vasopressin. Diabetes mellitus is due to an absolute or relative insulin deficiency. Untreated, it is characterized by polyuria, polydipsia, and polyphagia, and there will be an elevated blood glucose and glucosuria. Glucosuria is the presence of glucose in the urine. Psychogenic polydipsia is a psychological condition. A patient with psychogenic polydipsia drinks an enormous amount of water. Because of this large water intake, there is polyuria. However, with psychogenic polydipsia nocturia is usually absent. There will also be hyponatremia.

A 40-year-old otherwise healthy woman, nulligravida, presents with involuntary loss of urine that occurs after drinking a small amount of water, when washing the dishes, when hearing water running, and sometimes for no reason discernable to the patient. It is preceded by suddenly feeling the need to urinate and happens both during the day and at night. Urine analysis and culture, pelvic, gynecological, and neurological examinations are normal. Cystometric studies show residual volume of 45 mL (normal) with involuntary detrusor contractions starting already with 200 mL. Question What will be your therapeutic approach? Answer Choices 1 Obtain psychiatric consultation 2 Antibiotic trial 3 Oxybutinin 4 Neostigmine 5 Kegel exercise

oxybutinin Your patient most probably suffers from idiopathic hypertonic incontinence. Her bladder is best described as instable, spastic, or overactive. The most common cause of urge incontinence is involuntary and inappropriate detrusor muscle contractions (normally there are no detrusor contractions during cystometry studies). Cholinergic receptors are present mostly in the detrusor muscle, and their stimulation enhances urination. Therefore, their inhibition with use of an anticholinergic such as oxybutynin will prevent involuntary urgency and incontinency for no apparent reason. Psychiatric consultation will probably not prevent involuntary activity of the detrusor. It can be performed after the organic cause is excluded. Patients with normal urine analysis do not need antibiotics. You will not use neostigmine because it is an inhibitor of acethylcholinesterase. Acethylcholinesterase inhibitors such as neostigmine indirectly stimulate receptors in the detrusor and enhance urination. They could worsen her symptoms. Kegel exercises with voluntary contractions of pubococcygeus muscle will help the patient with some forms of stress incontinence. Your patient is nulligravida and has no signs of problems with pelvic relaxation.

A 15-year-old adolescent boy is seen in your office at 11:30 A.M. with a complaint of left scrotal pain and swelling; it started at 7 A.M. that same day, which is when he woke up. He recalls no trauma; when questioned, he says that he has never had intercourse. He has been feeling nauseated, and he vomited once. Physical examination demonstrates a well-nourished, well-developed boy, appearing moderately uncomfortable. Vital signs are normal. with the exception of a temperature of 37.9 degrees centigrade orally. Pain assessment score (Wong-Baker scale) is 6/10. He is Tanner Stage III puberty. The remaining physical examination is normal, except for the following findings: The left testicle is approximately 1.5 times the size of the right testicle. The skin is diffusely erythematous. Due to tenderness when touched, it is difficult to palpate the scrotum. Cremasteric reflex is absent. There are small, soft, pea-sized lymph nodes in both inguinal areas. Penis is circumcised and appears normal. Scrotal ultrasonography with Doppler ultrasound demonstrates decreased blood flow to the testis. Question What is the most likely diagnosis? Answer Choices 1 Acute idiopathic scrotal edema 2 Epididymo-orchitis 3 Testicular torsion 4 Torsion of testicular appendage 5 Varicocele

testicular torsion Explanation Testicular torsion in the adolescent boy is a urologic emergency, the most common cause of acute scrotal swelling and pain, and the most common cause of testicular loss. Torsion occurs in 1:4000 and occurs most commonly on the left side in the United States. The cause is a congenital anomaly that occurs in approximately 12% of boys/men, in which the tunica vaginalis is attached too high, allowing the testicle to rotate freely on the spermatic cord and vascular pedicle in the tunica vaginalis. Approximately 40% of boys/men have the anomaly bilaterally. Testicular torsion usually occurs between 12 and 18 years of age with the peak age of 14. It may occur up to 30 years of age, and it is found in infants and occasionally neonates at the time of birth. Up to 50% of patients may have had prior episodes of mild intermittent testicular pain that has resolved spontaneously, due to intermittent torsion and spontaneous derotation. Associated symptoms may include nausea and vomiting (20%), fever (16%), abdominal pain (20 - 30%), and urinary frequency (4%). Physical examination may demonstrate a horizontal position of the testis, and it may be elevated compared to the uninvolved side. The cremasteric reflex is usually absent, but its presence does not rule out testicular torsion. Elevation of the scrotum does not relieve the pain. The diagnosis is a clinical diagnosis. Because it consumes precious time, ultrasound examination of the testis with color flow Doppler should only be ordered when the diagnosis is uncertain and can determine if there is blood flow to the testis. The studies are 86% sensitive and 100% specific in making the diagnosis if the only criterion is decreased blood flow. Radionuclide scans are 90 - 100% accurate in identifying decreased blood flow. Rapid diagnosis is critical; if surgical intervention is provided within 6 hours of onset, the salvage rate for the testis is 80 - 100%; after 6 hours, the salvage rate is approximately 0%. Acute idiopathic scrotal edema is uncommon, but presents acutely with the average age of presentation 6 years. 90% of patients have a unilateral presentation. The scrotal skin is red and tender, but the testis appears to be normal. The redness tends to extend off the scrotum onto the perineum or onto the penis. This tends to resolve spontaneously in 48-72 hours and leaves no sequelae. Doppler ultrasound, if done, demonstrates good blood flow to the testis with peritesticular edema and fluid in the scrotal wall. Laboratory examination is normal except for occasional eosinophilia. Acute epididymitis and/or orchitis is not a common pediatric diagnosis. It was first described in 1956. The onset tends to be more gradual, generally over a few days, with fever and dysuria. Elevation of the scrotum may reduce discomfort. The cause may be viral, such as adenovirus, mumps, or Epstein-Barr virus, or bacterial. Bacterial infection is often associated with structural changes in the urinary tract. Urinalysis and urine culture may be helpful in establishing the diagnosis. Typical treatment is with rest, analgesia, and antibiotics if there is concern about a bacterial etiology. If a bacterial cause is identified, urinary tract imaging should be performed. There have been rare reports of acute epididymitis progressing to testicular infarction. Torsion of the appendix testis may present similarly to testicular torsion. Tenderness is usually localized to the upper portion of the testis and, typically, a blue dot is seen on the scrotal skin resulting from the venous congestion in the appendix testis. This is a self-limited condition and does not require surgical intervention. There are 5 appendages to the testis, all of which serve no function. If one twists or infarcts, symptoms result. Pain is less intense than with testicular torsion, and the cremasteric reflex is usually present. Varicocele occurs in 10 - 15% of males, 16% of adolescents, and 20 - 40% of men evaluated for infertility. First described in adolescents in 1885, the most common age of presentation is adolescence and early adulthood. They are caused by incompetent or absent valves of the spermatic veins, resulting in dilatation of the veins of the pampiniform plexus. Rarely are they caused by compression of the renal vein by a tumor, an aberrant renal artery, an obstructed renal vein. Doppler ultrasonography can demonstrate retrograde blood flow. They are most common on the left side, are usually asymptomatic, but may present with vague scrotal discomfort and swelling. Of those with symptoms, 2% have intratesticular Varicocele and these are more common on the right side. The typical physical finding is the bag of worms within the scrotal sac. They may be missed on physical examination if the supine position, so the patient should be examined in a standing position. Patients should be referred to urologists for further evaluation and to discuss options for treatment which sometimes, but not always, requires surgery.

What tissue is lined with simple cuboidal epithelium? Answer Choices 1 Small intestine 2 Trachea 3 Lung alveoli 4 The distal convoluted renal tubules 5 The renal corpuscle

the distal convoluted renal tubules The correct response is the distal convoluted renal tubules. The small intestine is lined with simple columnar epithelium. The trachea is lined with pseudostratified ciliated epithelium. Both lung alveoli and the renal corpuscle (Bowman's capsule) are lined with simple squamous epithelium.


Set pelajaran terkait

World geography south/east Asia review

View Set

Lewis Med-Surg Ch. 33 CAD and Acute Coronary Syndrome

View Set

Chapter 1, MKTG Chapter 1, Chapter 16 - Practice Problems, MKTG 351 Ch.20, MKT Exam 4, mktg ch 15, Chapter 2, MKT321_CH20, MKT 350 Test 4, Mktg 351 Test 3 Ch 20, MKTG 3, Marketing Segmentation, Chap 19 Practice Questions, MKT 350 quiz 4, Ch. 12 marke...

View Set

Pharmacology Exam 1 - Multiple Choice

View Set